ORTHOPEDIC MCQS ONLINE QUESTION BANK H2E

ORTHOPEDIC MCQS ONLINE QUESTION BANK H2E

The diagnosis of chronic exertional compartment syndrome is confirmed by measuring a resting pressure of more than    mm Hg, 1-minute postexercise pressure of more than    mm Hg, or a 5-minute postexercise pressure more than    mm Hg.

 

1) 10 mm Hg, 20 mm Hg, 15 mm Hg

3) 15 mm Hg, 30 mm Hg, 20 mm Hg

2) 30 mm Hg, 30 mm Hg, 30 mm Hg

5) 10 mm Hg, 10 mm Hg, 10 mm Hg

4) 5 mm Hg, 10 mm Hg, 5 mm Hg

 

Compartment pressure measurement and clinical history allow for an accurate diagnosis of chronic exertional compartment syndrome. The diagnosis is confirmed by measuring a resting pressure of more than 15 mm Hg, 1-minute postexercise pressure of more than 30 mm Hg, or a 5-minute postexercise pressure more than 20 mm Hg.Correct Answer: 15 mm Hg, 30 mm Hg, 20 mm Hg

 

 

3033. (2068) Q7-2487:

The recommended treatment for patients with chronic exertional compartment syndrome is:

 

1) Stretching and strengthening program

3) Fasciotomy

2) Activity modification

5) Steroid injections to reduce muscle swelling

4) Ultrasound

 

Unless patients are willing to avoid the offending activity, nonoperative management of chronic exertional compartment syndrome is unsuccessful. The surgical management involves fasciotomy of the affected compartments.Correct Answer: Fasciotomy

 

 

3034. (2069) Q7-2489:

Generally, the results after fasciotomy in carefully selected patients with acute compartment syndrome have resulted in improvement of symptoms in what percentage of patients:

 

1) 10% to 20%

3) 50% to 70%

2) 25% to 50%

5) 100%

4) 80% to 95%

 

Some studies have shown better results after release of the anterior compartment when compared to release of the deep posterior compartment. One study reported 96% excellent results after release for anterior compartment symptoms compared to 25% excellent (65% satisfactory) results after release for deep posterior compartment symptoms. Generally, the results after fasciotomy in carefully selected patients have resulted in improvement of symptoms in 80% to 95% of patients.Correct Answer: 80% to 95%

 

 

 

 

 

Slide 1

The magnetic resonance image in this distance runner (Slide) reveals:

 

1) Iliopsoas tendinitis

3) Pelvic stress fracture

2) Hip flexor injury

5) Subtrochanteric stress fracture

4) Femoral neck stress fracture

 

The magnetic resonance image reveals a compression side femoral neck stress reaction. A compression side femoral neck stress reaction should be treated with restricted weight bearing to a pain free level. If the reaction/fracture progresses to more than 50% involvement of the neck, it can be treated with percutaneous screw fixation to avoid the disastrous complication of a displaced femoral neck stress fracture.Correct Answer: Femoral neck stress fracture

 

 

3036. (2071) Q7-2491:

 

 

 

Slide 1

The following arthroscopic photograph (Slide) of a right knee intercondylar notch reveals:

 

1) A loose body

3) A posterior cruciate ligament tear

2) An anterior cruciate ligament tear

5) Posterior meniscofemoral ligament tear

4) A meniscal tear

 

Isolated posterior cruciate ligament (PCL) injuries are treated conservatively, whereas combined PCL injuries/grade 3 PCL injuries are typically managed operatively.Correct Answer: A posterior cruciate ligament tear

 

 

 

 

 

Slide 1

The femoral tunnel position in a right knee (Slide) is:

 

1) Appropriate for posterior cruciate ligament reconstruction

3) Too anterior for ACL reconstruction

2) Appropriate for anterior cruciate ligament (ACL) reconstruction

5) Too posterior for ACL reconstruction

4) Too vertical for ACL reconstruction

 

The appropriate position for femoral tunnel placement during ACL reconstruction is 10 or 10:30 in the right knee and 1:30 or 2:00 in the left knee.Correct Answer: Appropriate for anterior cruciate ligament (ACL) reconstruction

 

 

3038. (2194) Q7-2621:

In which of the following positions in baseball are athletes most frequently injured:

 

1) Pitcher

3) Infielder

2) Catcher

5) Base runner

4) Outfielder

 

Pitchers are more prone to injury than any other position in baseball.Correct Answer: Pitcher

 

 

3039. (2195) Q7-2622:

During pitching, upon landing normal lower extremity rotation of the lead leg is:

 

1) Neutral

3) 45° internal rotation

2) 15° internal rotation

5) 45° external rotation

4) 15° external rotation

 

During normal pitching mechanics, the lead leg should land in approximately 15° of internal rotation.Correct Answer: 15° internal rotation

 

 

3040. (2196) Q7-2623:

During pitching, which of the following best describes the position of the arm during ball release:

 

1) 60° to 80° shoulder abduction, 40° to 60° shoulder external rotation, and 20° elbow flexion

3) 80° to 100° shoulder abduction, 40° to 60° shoulder external rotation, and 0° elbow flexion

2) 80° to 100° shoulder abduction, 60° to 80° shoulder external rotation, and 20° elbow flexion

5) 80° to 100° shoulder abduction, 60° to 80° shoulder external rotation, and 0° elbow flexion

4) 80° to 100° shoulder abduction, 40° to 60° shoulder external rotation, and 20° elbow flexion

 

During pitching, the ball is released with the arm positioned in 80° to 100° shoulder abduction, 40° to 60° shoulder external rotation, and 20° elbow flexion.Correct Answer: 80° to 100° shoulder abduction, 40° to 60° shoulder external rotation, and 20° elbow flexion

 

 

 

 

 

 

Slide 1

The magnetic resonance image (MRI) of a 16-year-old high school football player who sustained a knee injury during a game is presented (Slide). He reports mild swelling at the time of injury but does not recall hearing a "pop." He has attempted to return to football but is unable to make side-to-side movements. On clinical examination, no difference in anterior or posterior laxity is appreciated when comparing the injured knee to the uninjured knee. What is the most appropriate initial management:

 

1) Open reduction internal fixation of the tibial plateau fracture

3) Medial patellofemoral ligament reconstruction

2) Percutaneous fixation of the tibial plateau fracture

5) Functional rehabilitation with progressive return to play

4) Medial collateral ligament repair

 

The MRI demonstrates an isolated injury to the medial collateral ligament with an associated lateral tibial plateau bone contusion. Appropriate initial management consists of functional rehabilitation with progressive return to play.Correct Answer: Functional rehabilitation with progressive return to play

 

 

3042. (2198) Q7-2625:

A 20-year-old male weightlifter presents with shoulder pain of insidious onset that occurs predominantly with bench press and military press activities. Clinical examination reveals tenderness on the superior aspect of the shoulder and pain with cross body adduction. No weakness or instability is noted. The most likely diagnosis is:

 

1) Osteolysis of the distal clavicle

3) Small rotator cuff tear

2) Acromioclavicular joint arthritis

5) Glenoid labrum tear

4) Microinstability of the glenohumeral joint

 

The most likely diagnosis is osteolysis of the distal clavicle. Osteolysis of the distal clavicle is a phenomenon that affects weightlifters and is exacerbated by bench press exercises.Correct Answer: Osteolysis of the distal clavicle

 

 

 

 

 

 

Slide 1

A magnetic resonance image (MRI) of the right foot of a 16-year-old female cross-country runner is presented. The patient complains of progressive pain in her right foot. Recommended initial management should include:

 

1) Excisional biopsy

3) Resumption of activities as tolerated

2) A period of nonweight bearing with further workup including nutritional and endocrine evaluation

5) Incisional biopsy

4) Open reduction internal fixation

 

The MRI demonstrates a stress fracture of the second metatarsal. Appropriate initial management includes a period of nonweight bearing, as well as assessment of risk factors for additional stress fractures such as poor nutrition and amenorrhea.Correct Answer: A period of nonweight bearing with further workup including nutritional and endocrine evaluation

 

 

3044. (2200) Q7-2627:

 

 

 

Slide 1

A magnetic resonance image (MRI) of both legs of a 32-year-old male marathon runner with progressive distal right leg pain is shown (Slide). Recommended initial management includes:

 

1) Unreamed intramedullary nailing

3) A period of nonweight bearing

2) Incisional biopsy

5) Reamed intramedullary nailing

4) Resumption of activities as tolerated

 

The MRI demonstrates a stress fracture of the tibia. Appropriate initial management is a period of nonweight bearing.Correct Answer: A period of nonweight bearing

 

 

 

 

 

Slide 1

A magnetic resonance image (MRI) of the left hip of a 26-year-old female triathlete with progressive pain is shown (Slide). Recommended initial management includes:

 

1) Percutaneous fixation with cannulated screws

3) Incisional biopsy

2) Resumption of activities as tolerated

5) A period of nonweight bearing with further workup including nutritional and menstrual history

4) Fixation with a dynamic hip screw

 

The MRI demonstrates an incomplete compression-sided femoral neck stress fracture. Appropriate initial management is a period of nonweight bearing as well as assessment of risk factors (e.g., poor nutrition and amenorrhea) for additional stress fractures. Tension-sided and complete femoral neck stress fractures should be treated with fixation to avoid fracture displacement and to promote union.Correct Answer: A period of nonweight bearing with further workup including nutritional and menstrual history

 

 

3046. (2202) Q7-2629:

 

 

 

Slide 1

A 19-year-old collegiate level volleyball player injured her right thumb during a game. A magnetic resonance image (MRI) through the metacarpophalangeal joint is shown (Slide). What is the diagnosis:

 

1) Extensor tendon disruption

3) Volar plate disruption

2) Complete unar collateral ligment tear

5) Ulnar collateral ligament sprain

4) Incomplete ulnar collateral ligament tear

 

The MRI reveals a complete disruption of the ulnar collateral ligament of the metacarpophalangeal joint of the thumb.Correct Answer: Complete unar collateral ligment tear

 

 

 

 

 

Slide 1

A 19-year-old collegiate level volleyball player injured her right thumb during a game. A magnetic resonance image (MRI) through the metacarpophalangeal joint is shown (Slide). What is the appropriate initial management:

 

1) Thumb spica cast for 6 weeks

3) Ulnar collateral ligament reconstruction with palmaris longus graft

2) Therapy to regain mobility and function

5) Excisional biopsy

4) Ulnar collateral ligament repair

 

The MRI reveals a complete disruption of the ulnar collateral ligament of the metacarpophalangeal joint of the thumb. The preferred treatment for complete disruptions of the ulnar collateral ligament is acute repair.Correct Answer: Ulnar collateral ligament repair

 

 

3048. (2204) Q7-2631:

Which of the following injuries is most often responsible for death in youth baseball:

 

1) Head injury

3) Pelvic injury

2) Abdominal injury

5) Spinal cord injury

4) Cardiac injury

 

Catastrophic injuries are rare in baseball. In the few deaths that have occurred, most have resulted from a blow to the chest resulting in cardiac injury. The gravest prognosis occurs with commotion cordis, which results from a direct blow to the chest causing dysrhythmia even if resuscitative care is immediately provided.Correct Answer: Cardiac injury

 

 

3049. (2205) Q7-2632:

 

 

 

Slide 1

A magnetic resonance image (MRI) of a 24-year-old, right-hand dominant minor league baseball pitcher is presented (Slide). He has undergone two shoulder arthroscopies within the past year. He currently complains of vague shoulder pain and a catching or locking sensation when attempting to pitch. Which of the following is most likely responsible for his symptoms:

 

1) Internal impingement

3) Recurrent labral tear

2) Loose foreign body with associated synovitis

5) Anterior shoulder instability

4) Rotator cuff tear

 

The MRI shows an intra-articular loose foreign body, presumably a bioabsorbable tack from a previous shoulder surgery. Multiple researchers have observed synovitic reactions from bioabsorbable implants in the glenohumeral joint.Correct Answer: Loose foreign body with associated synovitis

 

 

 

 

 

Slide 1

The magnetic resonance image (MRI) of a 29-year-old recreational bodybuilder is shown (Slide). He states that he felt a pop in the right shoulder girdle while "maxing out" on bench press. What is the most appropriate treatment:

 

1) Physical therapy to restore mobility and function

3) Acute repair of the injured structure

2) Sling usage for 6 weeks

5) Incisional biopsy of the lesion

4) Delayed repair of the injured structure

 

This patient has a rupture of the pectoralis major muscle. Although delayed repair can yield acceptable results in young, active patients, most surgeons advocate early repair for restoration of strength and cosmesis.Correct Answer: Acute repair of the injured structure

 

 

3051. (2207) Q7-2634:

 

 

 

Slide 1

A magnetic resonance image (MRI) of a 16-year-old high school football player who sustained a knee injury during a game is presented (Slide). He reports mild swelling at the time of injury but does not recall hearing a "pop." He has attempted to return to football, but he is unable to make side to side movements. On clinical examination, no difference in anterior or posterior laxity is appreciated when comparing the injured knee to the uninjured knee. What is the most likely diagnosis:

 

1) Anterior cruciate ligament injury

3) Medial patellofemoral ligament injury

2) Medial collateral ligament injury

5) Posterior cruciate ligament injury

4) Medial meniscal tear

 

The MRI demonstrates an isolated injury to the medial collateral ligament with an associated lateral tibial plateau bone contusion. Appropriate initial management consists of functional rehabilitation.Correct Answer: Medial collateral ligament injury

 

 

Slide 1

A magnetic resonance image (MRI) of a 15-year-old female volleyball player who twisted her knee is shown (Slide). Despite 6 weeks of rehabilitation, she has been unable to return to volleyball without having her knee give out. What is the most likely diagnosis:

 

1) Anterior cruciate ligament injury

3) Medial patellofemoral ligament injury

2) Medial collateral ligament injury

5) Posterior cruciate ligament injury

4) Medial meniscal tear

 

The MRI clearly shows disruption of the femoral attachment of the anterior cruciate ligament with characteristic joint effusion.Correct Answer: Anterior cruciate ligament injury

 

 

3053. (2209) Q7-2636:

 

 

 

Slide 1

A magnetic resonance image (MRI) of a 15-year-old female volleyball player who twisted her knee is presented (Slide). Despite 6 weeks of rehabilitation, she has been unable to return to volleyball without having her knee give out. If left untreated, which of the following structures is prone to injury with repetitive "giving way" episodes:

 

1) Lateral collateral ligament

3) Medial patellofemoral ligament

2) Medial collateral ligament

5) Posterior cruciate ligament

4) Menisci

 

The MRI clearly shows disruption of the femoral attachment of the anterior cruciate ligament with characteristic joint effusion. With repetitive "giving way" episodes, the menisci are vulnerable to injury, particularly in the adolescent population. Graf and colleagues reported that seven of eight skeletally immature patients with anterior cruciate ligament injuries treated nonoperatively went on to develop meniscal tears.Correct Answer: Menisci

 

 

Slide 1

A magnetic resonance image (MRI) of a 15-year-old female volleyball player who twisted her knee is shown (Slide). Despite 6 weeks of rehabilitation, she has been unable to return to volleyball without having her knee give out. What is the most appropriate treatment:

 

1) Anterior cruciate ligament reconstruction

3) Continued physical therapy

2) Posterior cruciate ligament reconstruction

5) Medial patellofemoral ligament reconstruction

4) Use of a knee brace

 

The MRI clearly shows disruption of the femoral attachment of the anterior cruciate ligament with characteristic joint effusion. The most appropriate treatment for the patient in this scenario is anterior cruciate ligament reconstruction.Correct Answer: Anterior cruciate ligament reconstruction

 

 

3055. (2211) Q7-2638:

 

 

 

Slide 1

A magnetic resonance image (MRI) of a 21-year-old female collegiate basketball player who felt her knee give out while playing is shown (Slide). What is the injured structure:

 

1) Lateral collateral ligament

3) Medial patellofemoral ligament

2) Medial collateral ligament

5) Posterior cruciate ligament

4) Meniscus

 

This patient dislocated her patella, and the MRI demonstrates the pathoanatomical lesion of traumatic patellar dislocation â disruption of the medial patellofemoral ligament. Initial management of these injuries is generally nonoperative, although some surgeons now advocate early repair or reconstruction of the medial patellofemoral ligament in the athletic population.Correct Answer: Medial patellofemoral ligament

 

Good prognostic signs for patients with arthroscopic treatment of degenerative arthritis of the knee incude:

 

1) Malalignment

3) Loose bodies

2) Advanced degenerative changes

5) Pending litigation

4) Chondrocalcinosis

 

Good prognostic signs for patients undergoing arthroscopic treatment of a degenerative knee include increased pain of acute onset, mechanical symptoms, recent effusion, loose bodies, normal alignment, isolated chondral flap/fracture, and meniscal tears. Poor prognostic signs include malalignment, ligamentous instability, pending litigation, and complete loss of the joint space.Correct Answer: Loose bodies

 

 

3057. (2213) Q7-2640:

The reported 3- to 5-year success rate of a chondral lesion microfracture technique in the knee is:

 

1) 10% improvement

3) 40% improvement

2) 25% improvement

5) 95% improvement

4) 75% improvement

 

Steadman and colleagues reported a 75% improvement at 3- to 5-year follow-up using the microfracture technique with arthroscopic awls.Correct Answer: 75% improvement

 

 

3058. (2214) Q7-2641:

Which of the following is not a high-risk stress fracture:

 

1) Navicular

3) Anterior tibial diaphysis

2) Femoral neck

5) Proximal 5th metatarsal

4) Posteromedial tibia

 

Posteromedial tibial stress fractures are not considered high-risk fractures and can be treated with relative rest and staying at an asymptomatic level of activity. Complete fracture, nonunion, or other complications are rare after posteromedial tibial stress fractures.Correct Answer: Posteromedial tibia

 

 

3059. (2516) Q7-2987:

Meniscus repair is most successful for tears in which of the following zones:

 

1) Red-blue zone

3) Red-white zone

2) Red-red zone

5) White-red zone

4) White-white zone

 

The microvascular anatomy of the meniscus was initially defined by Arnoczky and Warren in 1982. They demonstrated that the perimeniscal capillary plexus supplies the most peripheral region of the meniscus with blood and that this blood supply is centrally insufficient. Their finding accurately predicted that peripheral tears would have a better chance of healing than more central tears. Subsequently, the menisci were divided into three regions based upon the blood supply. These three regions are the red-red zone (peripheral third), the red-white zone (middle third), and the white-white zone (central third). The zone names correlate with the rich peripheral and poor central blood supply.Correct Answer: Red-red zone

 

Which of the following does not improve meniscal repair outcomes in avascular zones:

 

1) Concomitant anterior cruciate ligament (ACL) reconstruction

3) Synovial abrasion

2) Fibrin clot

5) Hyalgan injections

4) Vascular access channels

 

Several techniques have been implemented to improve the outcome of meniscus repair. Patients undergoing concomitant ACL reconstruction have better results than patients with an ACL tear that is not addressed or those with no ACL tear. Fibrin clot is known to improve healing due to growth factors present in the clot. Synovial abrasion and vascular access channels are two less commonly used techniques that augment meniscal healing. Hyalgan injections have no proven benefit for meniscal healing.Correct Answer: Hyalgan injections

 

 

3061. (2518) Q7-2989:

Which tear pattern is most responsive to meniscus repair:

 

1) Radial

3) Parrot beak

2) Complex

5) Horizontal cleavage

4) Longitudinal

 

Longitudinal tears are the most responsive tear pattern to successful repairs. Longitudinal tears occur in the plane of the radial orientation of the meniscal infrastructure. Other patterns of tears disrupt this radial orientation, resulting in high stress across the repair site and difficulty in reconstituting lost meniscal function.Correct Answer: Longitudinal

 

 

3062. (2519) Q7-2990:

Which head of the pectoralis major muscle most likely remains intact with a partial rupture:

 

1) Sternocostal

3) Clavicular

2) Sternoclavicular

5) Superior

4) Costal

 

There are two heads of the pectoralis major â the clavicular head inserts anteriorly and the sternocostal head inserts posteriorly. The clavicular head remains intact in a partial rupture.Correct Answer: Clavicular

 

 

3063. (2520) Q7-2991:

In 1948, T.J. Fairbank, MD, published a landmark article on which of the following topics:

 

1) Classification of articular cartilage injuries

3) Short-term radiographic changes after total meniscectomy

2) Long-term radiographic changes after total meniscectomy

5) The effect of anterior cruciate ligament reconstruction on arthritis

4) Short-term radiographic changes after partial meniscectomy

 

T.J. Fairbank, MD, published a landmark article in 1948 on the long-term radiographic changes after total meniscectomy. The radiographic changes that now bear his name (Fairbank changes) refer to osteophyte formation, joint space narrowing, and flattening of the femoral condyles.Correct Answer: Long-term radiographic changes after total meniscectomy

 

Which of the following meniscus repair techniques results in the highest pullout strength:

 

1) Meniscus arrows

3) Vertical mattress sutures

2) Horizontal mattress sutures

5) Fibrin clot

4) Fibrin glue

 

Vertical mattress sutures have been proven to be the gold standard for comparison of all other meniscus repair techniques. The suture pattern yields the highest pullout strength for load to failure and cyclic loading testing.Correct Answer: Vertical mattress sutures

 

 

3065. (2522) Q7-2993:

Which of the following is the most important factor for meniscus repair success:

 

1) Location of tear

3) Age of patient

2) Pattern of tear

5) Stability of the knee

4) Age of tear

 

The location of the tear is the most important factor in meniscus repair. Tears in the peripheral red-red zone have the highest likelihood of healing, whereas tears in the central white-white zone are less likely to heal.Correct Answer: Location of tear

 

 

3066. (2523) Q7-2994:

Which structure makes up the anterior boundary of Hunterâs canal (adductor canal):

 

1) Vastus medialis

3) Sartorius

2) Vastus lateralis

5) Semitendinosus

4) Pectineus

 

Hunterâs canal (adductor canal) is bound anterolaterally by the vastus medialis, posterolaterally by the adductor longus, and medially by the sartorius.Correct Answer: Sartorius

 

 

3067. (2524) Q7-2995:

Which of the following structures is the major stabilizer of the knee to valgus stress at 30°:

 

1) Posterior oblique ligament

3) Superficial medial collateral ligament

2) Deep medial collateral ligament

5) Anterior cruciate ligament

4) Medial meniscus

 

The major stabilizer of the knee to valgus load at 30° of flexion is the superficial medial collateral ligament. The deep medial collateral ligament and the posterior oblique ligament also contribute to stability of the knee, but they are not as important as the superficial medial collateral ligament at 30°. The anterior cruciate ligament plays a secondary role in valgus stability; however, it functions mostly in full extension.Correct Answer: Superficial medial collateral ligament

 

Which of the following is the most commonly injured knee ligament:

 

1) Lateral collateral ligament

3) Anterior cruciate ligament

2) Medial collateral ligament

5) Falciform ligament

4) Posterior cruciate ligament

 

The medial collateral ligament is the most commonly injured ligament of the knee. Generally, these injuries are treated nonoperatively and many are overlooked or not diagnosed by a physician. Anterior cruciate ligament injuries are the second most common injury, but they most often result in surgical reconstruction. Posterior cruciate ligament injuries are less common but are under-recognized.Correct Answer: Medial collateral ligament

 

 

3069. (2526) Q7-2997:

Where is the most common site of a medial collateral ligament injury:

 

1) Tibial insertion

3) Mid substance

2) Femoral origin

5) Inferior pole of the patella

4) Fibular head

 

The most common site of a medial collateral ligament injury is the femoral origin. Midsubstance injuries occur less commonly. Injuries at the tibial insertion generate some controversy, and some authors advocate surgical repair, citing inferior results with conservative management.Correct Answer: Femoral origin

 

 

3070. (2527) Q7-2998:

Which of the following structures is tested with an external rotation recurvatum test:

 

1) Posterolateral corner

3) Medial collateral ligament

2) Anterior cruciate ligament

5) Medial meniscus

4) Posterior cruciate ligament

 

The external rotation recurvatum test evaluates the posterolateral corner. The test is performed with a patient in the supine position. The great toe of each foot is grasped and lifted as the patient is relaxed. The injured knee will hyperextend and drift into varus with a positive test.Correct Answer: Posterolateral corner

 

 

3071. (2528) Q7-2999:

Which of the following structures is most commonly injured in conjunction with a posterior cruciate ligament rupture:

 

1) Anterior cruciate ligament

3) Medial collateral ligament

2) Posterolateral corner

5) Patellar tendon

4) Lateral meniscus

 

The posterolateral corner is most often associated with posterior cruciate ligament injuries. Patients who fail conservative management of posterior cruciate ligament injuries often have an associated posterolateral corner injury.Correct Answer: Posterolateral corner

 

What is the most appropriate management for treatment of an isolated grade I medial collateral ligament (MCL) injury:

 

1) Anterior cruciate ligament reconstruction

3) Nonoperative treatment with 2 to 4 weeks in a cylinder cast, followed by range of motion and strengthening

2) Nonoperative treatment with an optional short course of immobilization, followed by range of motion and strengthening

5) Surgical repair of the MCL

4) Nonoperative treatment with immediate return to sport

 

Isolated grade I MCL injuries are best treated with a short course of immobilization (few days at most), followed by range of motion and strengthening as pain allows. Patients can generally return to sport within 2 weeks.Correct Answer: Nonoperative treatment with an optional short course of immobilization, followed by range of motion and strengthening

 

 

3073. (2530) Q7-3001:

What is the most appropriate treatment of an isolated grade II medial collateral ligament (MCL) injury:

 

1) Anterior cruciate ligament reconstruction

3) Nonoperative treatment with 4 to 6 weeks in a cylinder cast, followed by range of motion and strengthening

2) Nonoperative treatment with a short course of immobilization, followed by range of motion and strengthening. Patients can return to sport within 4 weeks.

5) Surgical repair of the MCL

4) Nonoperative treatment with immediate return to sport

 

Isolated grade II MCL injuries are best treated with a short course of immobilization (up to 1 week), followed by range of motion and strengthening as pain allows. Patients can generally return to sport within 4 weeks.Correct Answer: Nonoperative treatment with a short course of immobilization, followed by range of motion and strengthening. Patients can return to sport within 4 weeks.

 

 

3074. (2531) Q7-3002:

What is the major technical limitation of open meniscus repair:

 

1) Cannot use sutures for the repair

3) Cannot repair peripheral tears

2) Cannot place vertical mattress sutures

5) Poor long-term results

4) Cannot access central tears

 

The major limitation to open meniscus repair is the poor access to more central tears. The long-term results of meniscus repair have been good.Correct Answer: Cannot access central tears

 

 

3075. (2532) Q7-3003:

Which of the following is the main advantage of an all-inside meniscus repair compared to an inside-out repair:

 

1) Cheaper implants

3) Ease of use, less operative time, and no incision

2) Biomechanically superior repair

5) Better long-term results

4) Fewer implant complications

 

The main advantages to all-inside repair are ease of use, less operative time, and lack of an incision. Currently, there are still complications with implants, but these are improving over time. The vertical mattress suture of an inside-out repair is still superior to an all-inside implant biomechanically, but the newer generation of implants is improving. Although an inside-out technique is biomechanically superior, several studies indicate equivalent outcomes between all-inside and inside-out techniques.Correct Answer: Ease of use, less operative time, and no incision

 

Which adductor muscle is most commonly injured in a groin strain:

 

1) Adductor brevis

3) Adductor magnus

2) Adductor longus

5) Sartorius

4) Pectineus

 

The adductor longus is the most frequently injured of the adductor muscles. The adductor brevis and pectineus muscles are short; therefore, they are relatively protected. The adductor magnus has a larger muscle belly than the adductor longus but is still not injured as often. The sartorius crosses two joints and is the longest muscle-tendon unit in the body. The sartorius is often injured but less frequently than the adductor longus.Correct Answer: Adductor longus

 

 

3077. (2534) Q7-3005:

Where is the primary pathology site for athletic pubalgia:

 

1) Sartorius origin

3) Rectus abdominus insertion

2) Rectus abdominus origin

5) Ischial tuberosity

4) Inguinal canal

 

The primary pathology site of athletic pubalgia is believed to be the rectus abdominus insertion on the pubis. This is a diagnosis of exclusion and some authors question its existence. An athlete generally has pain in the lower abdominal and groin area. A hernia is not present and, generally, patients fail a course of conservative management of a groin strain before the diagnosis is made.

Patients generally have pain with palpation at the rectus abdominus insertion on the pubis. Surgical management involves an open Bassini hernia repair.Correct Answer: Rectus abdominus insertion

 

 

3078. (2535) Q7-3006:

Which of the following physical examination maneuvers is suggestive of piriformis syndrome:

 

1) Forced internal rotation of an extended thigh

3) Forced internal rotation of a flexed thigh

2) Forced external rotation of a flexed thigh

5) Forced internal rotation of a flexed knee

4) Forced external rotation of an extended thigh

 

Piriformis syndrome is an uncommon ailment in which patients generally complain of posterior buttock and thigh pain. A lumbar spine etiology must be ruled out first. Piriformis syndrome is often a diagnosis of exclusion. Few physical examination findings suggest the diagnosis. A patient may have pain with direct palpation over the piriformis muscle, but the more suggestive finding is pain elicited from forced internal rotation of an extended thigh.Correct Answer: Forced internal rotation of an extended thigh

 

 

3079. (2536) Q7-3007:

What is the name of the sign elicited with the test for piriformis syndrome:

 

1) Wilsonâs sign

3) Pellegrini-Stieda sign

2) McMurrayâs sign

5) Paceâs sign

4) Hoffmanâs sign

 

Paceâs sign is the name given to pain elicited with forced internal rotation of a flexed hip and is suggestive of piriformis syndrome if patient history correlates with the examination. Wilsonâs sign is associated with osteochondritis dissecans of the knee.

McMurrayâs sign is associated with meniscus tears of the knee. Pellegrini-Stieda sign refers to a radiographic finding of chronic medial collateral ligament injuries. Hoffmanâs sign is associated with cervical stenosis.Correct Answer: Paceâs sign

 

Which population of athletes most commonly presents with snapping hip syndrome due to the iliopsoas tendon:

 

1) Soccer players

3) Gymnasts

2) Football players

5) Bowlers

4) Long distance runners

 

Snapping iliopsoas tendon syndrome is most common in long distance runners. Other athletes may have groin pain, but the etiology generally differs. Soccer players and football players often have adductor strains due to the ballistic nature of the sport. Gymnasts may have several sources of groin pain including adductor strains, acetabular labral tears, or iliopsoas tendon syndrome. Soccer players may also have athletic pubalgia. The repetitive nature of long distance running seems to predispose those athletes to snapping iliopsoas tendon syndrome.Correct Answer: Long distance runners

 

 

3081. (2538) Q7-3009:

Which of the following muscles is the strongest flexor of the hip joint:

 

1) Rectus femoris

3) Adductor magnus

2) Sartorius

5) Biceps femoris

4) Iliopsoas

 

The iliopsoas is the strongest flexor of the hip joint. The sartorius and rectus femoris are also hip flexors. The biceps femoris extends the hip, and the adductor magnus mainly adducts the hip.Correct Answer: Iliopsoas

 

 

3082. (2539) Q7-3010:

Which type of injury is evaluated by Gaenslenâs test:

 

1) Iliopsoas tendinitis

3) Osteitis pubis

2) Acetabular labral tear

5) Sacroiliac sprain

4) Hamstring strain

 

Gaenslenâs test is performed by hyperflexing the uninjured hip while hyperextending the hip on the painful side. If this causes pain over the sacroiliac joint, then it is considered a positive test for a sacroiliac sprain.Correct Answer: Sacroiliac sprain

 

 

3083. (2540) Q7-3011:

Which of the following structures is compressed or occluded in quadrilateral space syndrome:

 

1) Anterior circumflex humeral artery

3) Radial nerve

2) Posterior circumflex humeral artery

5) Axillary artery

4) Profunda brachii artery

 

The quadrilateral (quadrangular) space is bound by the teres minor superiorly, teres major inferiorly, long head of the triceps medially, and the lateral head of the biceps laterally. The axillary nerve and posterior circumflex humeral vessels pass through this space. Quadrilateral space syndrome is defined by pain and paresthesias with the shoulder abducted, extended, and externally rotated, which transiently occludes the posterior circumflex humeral vessels.Correct Answer: Posterior circumflex humeral artery

 

Where is Voshellâs bursa located:

 

1) Between the superficial and deep medial collateral ligaments of the knee

3) Lateral to the greater trochanter of the hip

2) In the subscapularis fossa of the shoulder

5) Posterior to the medial head of the gastrocnemius of the knee

4) Superficial to the hamstring tendons of the knee

 

Voshellâs bursa is located between the deep and superficial medial collateral ligament of the knee.Correct Answer: Between the superficial and deep medial collateral ligaments of the knee

 

 

3085. (2668) Q7-3161:

Which of the following lends the greatest stability to the sternoclavicular joint:

 

1) Bony articular congruity

3) Costoclavicular ligament

2) Articular disk

5) Interclavicular ligament

4) Sternoclavicular capsular ligament

 

The sternoclavicular joint is a fairly incongruous diarthrodial joint consisting of the medial aspect of the clavicle, the sternum, and an intervening cartilaginous articular disk. The articular disk accommodates for some of the incongruence between the medial aspect of the clavicle and the sternum; however, the majority of stability of the sternoclavicular joint is imparted by the strong surrounding ligamentous complex. This ligamentous complex is comprised of the capsular ligament, the interclavicular ligament, the costoclavicular or rhomboid ligament, and the intra-articular disk ligment. Of these ligaments, the capsular ligament imparts the most stability to the joint.Correct Answer: Sternoclavicular capsular ligament

 

 

3086. (2669) Q7-3162:

What is the most common mechanism of anterior dislocation of the sternoclavicular joint:

 

1) A medially directed force applied to the lateral aspect of the externally rotated shoulder girdle

3) A medially directed force applied to the lateral aspect of the neutrally rotated shoulder girdle

2) A medially directed force applied to the lateral aspect of the internally rotated shoulder girdle

5) A superiorly directed force applied to the medial aspect of the clavicle

4) An inferiorly directed force applied to the medial aspect of the clavicle

 

The mechanism of sternoclavicular joint injury may be direct or indirect. Direct injury involves posteriorly directed forces applied to the medial aspect of the clavicle resulting in proximal clavicular fracture and/or posterior sternoclavicular dislocation. More commonly, indirect mechanisms are responsible for sternoclavicular joint injury. Indirect injury occurs when a medial directed force is applied to the lateral aspect of the shoulder. As this force is applied, if the shoulder girdle externally rotates, an anterior sternoclavicular dislocation can occur. If the shoulder internally rotates, then a posterior sternoclavicular joint dislocation may ensue.Correct Answer: A medially directed force applied to the lateral aspect of the externally rotated shoulder girdle

 

 

3087. (2670) Q7-3163:

What is the most common mechanism of posterior dislocation of the sternoclavicular joint:

 

1) An inferiorly directed force applied to the medial aspect of the clavicle

3) A medially directed force applied to the lateral aspect of the externally rotated shoulder girdle

2) A superiorly directed force applied to the medial aspect of the clavicle

5) A medially directed force applied to the lateral aspect of the neutrally rotated shoulder girdle

4) A medially directed force applied to the lateral aspect of the internally rotated shoulder girdle

 

The mechanism of sternoclavicular joint injury may be direct or indirect. Direct injury involves posteriorly directed forces applied to the medial aspect of the clavicle resulting in proximal clavicular fracture and/or posterior sternoclavicular dislocation. More commonly, indirect mechanisms are responsible for sternoclavicular joint injury. Indirect injury occurs when a medial directed force is applied to the lateral aspect of the shoulder. As this force is applied, if the shoulder girdle externally rotates, then an anterior sternoclavicular dislocation can occur. If the shoulder internally rotates, a posterior sternoclavicular joint dislocation may ensue.Correct Answer: A medially directed force applied to the lateral aspect of the internally rotated shoulder girdle

 

At what age does the medial clavicular physis close:

 

1) 8 years

3) 17 years

2) 13 years

5) 25 years

4) 20 years

 

The proximal clavicular physis is the last in the body to close with the proximal clavicular epiphysis uniting with the metaphysis usually between the 23rd and 25th year, possibly as late as the 31st year. This becomes important when evaluating sternoclavicular joint injuries in patients younger than 30 years as many of the suspected dislocations are actually proximal clavicular physeal fractures.Correct Answer: 25 years

 

 

3089. (2672) Q7-3165:

Which of the following is the most common pattern of sternoclavicular joint dislocation:

 

1) Superior

3) Inferior

2) Anterior

5) Posterior inferior

4) Posterior superior

 

Athletic-related injuries to the sternoclavicular joint range from sprains to fractures and/or dislocations. Most injuries are sprains involving stretching or incomplete tearing of the sternoclavicular ligaments. More significant ligamentous injury may result in sternoclavicular subluxation (transient loss of articular apposition of the sternoclavicular joint resolving spontaneously) or dislocation. More than 90% of dislocations occur in the anterior direction.Correct Answer: Anterior

 

 

3090. (2673) Q7-3166:

A 32-year-old man sustains an injury as a result of being on the bottom of a pile during a rugby game. He complains of pain at the medial aspect of his clavicle. He is having difficulty swallowing water and feels like his âthroat is closing.â Physical examination demonstrates obvious swelling at the medial aspect of the clavicle and sternoclavicular joint. His radial pulse in his ipsilateral hand is also slightly decreased compared to the contralateral radial pulse. What is the most likely diagnosis:

 

1) Anterior sternoclavicular joint dislocation

3) Inferior sternoclavicular joint dislocation

2) Posterior sternoclavicular joint dislocation

5) Medial clavicular physeal fracture

4) Superior sternoclavicular joint dislocation

 

Presentation of patients with sternoclavicular joint injuries varies with the type of injury sustained. Patients with sprains and subluxations of the sternoclavicular joint present with complaints of tenderness at the medial aspect of the clavicle and sternoclavicular pain with shoulder movement. The patient may sustain the injury through an indirect mechanism as a result of being on the bottom of a pile while participating in football or rugby.

 

Patients with anterior dislocation of the sternoclavicular joint usually present with an obvious deformity with the medial aspect of the clavicle protruding anteriorly. These injuries are usually sustained when the patient is on the bottom of a pile during football or rugby, and the ipsilateral shoulder girdle is externally rotated during simultaneous lateral compression.

 

Patients with posterior sternoclavicular joint dislocations often present with more serious symptoms. In addition to severe pain and an obvious deformity with the medial aspect of the clavicle protruding posteriorly, patients often complain of dyspnea, dysphagea, and throat pain. Additionally, signs of vascular compromise to the ipsilateral upper extremity may occur.

 

Correct Answer: Posterior sternoclavicular joint dislocation

 

Which of the following is the imaging modality of choice for evaluation of sternoclavicular joint injuries:

 

1) Simple radiography

3) Magnetic resonance imaging

2) Ultrasonography

5) Double contrast arthrography

4) Computed tomography

 

Assessment of the sternoclavicular joint using simple radiography is exceptionally difficult. Secondary imaging using computed tomography, which readily identifies fractures and dislocations involving the sternoclavicular joint, is the imaging modality of choice in evaluation of sternoclavicular joint injuries.Correct Answer: Computed tomography

 

 

3092. (2675) Q7-3168:

A 32-year-old man sustains an injury as a result of being on the bottom of a pile during a rugby game. He complains of pain at the medial aspect of his clavicle. He is having difficulty swallowing water and feels like his âthroat is closing.â Physical examination demonstrates obvious swelling at the medial aspect of the clavicle and sternoclavicular joint. His radial pulse in his ipsilateral hand is also slightly decreased compared to the contralateral radial pulse. After confirming the diagnosis with appropriate imaging studies, what is the most appropriate treatment:

 

1) Symptomatic treatment with progressive return to rugby

3) Figure-of-eight splint for 6 weeks

2) Sling for 2 weeks

5) Emergent open reduction

4) Emergent closed reduction

 

Acute posterior sternoclavicular joint dislocations should be emergently reduced in a controlled environment. Closed reduction under anesthesia is usually successful and is performed by placing a rolled sheet between the patientâs scapulae, having an assistant apply longitudinal and posterior traction to the upper extremity, and applying an anteriorly directed force to the ipsilateral clavicle using pointed reduction forceps percutaneously. Closed reduction is usually successful. Failing closed reduction, open reduction is used.Correct Answer: Emergent closed reduction

 

 

3093. (2676) Q7-3169:

Which best describes a type IIA distal clavicle fracture:

 

  1. The fracture is lateral to the coracoclavicular ligaments.

    3) The fracture line is between the conoid and trapezoid ligaments.

    2) The fracture involves intra-articular injury of the acromioclavicular joint.

    5) The fracture involves comminuted distal clavicle fracture.

    1. The fracture is medial to the coracoclavicular ligaments.

       

      A type IIA distal clavicle fracture occurs medial to the coracoclavicular ligaments. Type I is an extra-articular fracture lateral to the coracoclavicular ligaments. Type III is an intra-articular injury of the acromioclavicular joint.Correct Answer: The fracture is medial to the coracoclavicular ligaments.

       

       

      3094. (2677) Q7-3170:

      Distal one-third clavicle fractures constitute what percentage of all clavicle fractures:

       

      1) 15%

      3) 45%

      2) 30%

    2. 90%

    4) 60%

     

    Distal one-third clavicle fractures constitute 15% of all clavicle fractures. The most common location for a clavicle fracture is midshaft.Correct Answer: 15%

     

    What percentage of type II distal clavicle fractures results in nonunion with nonoperative management:

     

    1) 15%

    3) 45%

  2. 30%

    5) 90%

    4) 60%

     

    Few studies have looked at the nonunion rate for type II distal one-third clavicle fractures. These few studies, however, generally yield a 30% nonunion rate, the highest among any type of clavicle fracture.Correct Answer: 30%

     

     

    3096. (2679) Q7-3172:

    What is the most common type of discoid lateral meniscus:

     

    1. Humphrey

  3. Complete

    1. Wrisberg

      5) Partial

      4) Incomplete

       

      Discoid lateral menisci are divided into three types by the Watanabe classification system: Wrisberg, complete, and incomplete. Complete is the most common type. Incomplete and Wrisberg types are less common and approximately equal in incidence.

      Complete discoid meniscus implies that the abnormal meniscus covers the entire tibial plateau. Incomplete implies partial covering of the plateau. Wrisberg type is defined by a lack of attachment of the meniscotibial ligaments to the posterior horn of the lateral meniscus.Correct Answer: Complete

       

       

      3097. (2680) Q7-3173:

      The gravity subluxation test is used to evaluate:

       

      1) Anterior cruciate ligament deficiency

      3) Lateral patellar subluxation

      2) Posterior cruciate ligament deficiency

      5) Shoulder subluxation

      4) Medial patellar subluxation

       

      The gravity subluxation test is used to evaluate medial patellar subluxation. The test is performed with the patient in the lateral decubitus position with the affected extremity up. The thigh is passively abducted by the examiner. If the patella subluxates medially, then the test is suggestive. It is definitively positive if active quadriceps contraction fails to bring the patella back into the groove.Correct Answer: Medial patellar subluxation

       

       

      3098. (2681) Q7-3174:

      Detachment of the superior glenoid labrum:

       

      1) Diminishes anterior shoulder stability

      3) Increases anterior shoulder stability

      2) Diminishes inferior shoulder stability

      5) Diminishes global shoulder stability

      4) Increases inferior shoulder stability

       

      Biomechanical studies have shown that superior labrum anterior and posterior (SLAP) lesions allow increased anterior humeral head translation and decreased anterior stability of the glenohumeral joint. The superior labrum acts as a static stabilizer of the shoulder joint preventing anterior instability. Contraction of the long head of the biceps tightens the surrounding labrum and capsule at its attachment site, providing dynamic stability to the shoulder joint. SLAP lesions disrupt the static restraint of the labrum and decrease the effectiveness of the biceps to dynamically stabilize the shoulder. SLAP lesions diminish anterior stability primarily.Correct Answer: Diminishes anterior shoulder stability

       

      Superior labrum anterior and posterior (SLAP) lesions affect a shoulder in the throwing position by:

       

      1) Decreasing torsional rigidity

      3) Decreasing axial rigidity

      2) Increasing torsional rigidity

      5) Decreasing angular rigidity

      4) Increasing axial rigidity

       

      Detachment of the labrum in SLAP lesions causes a decrease in the torsional stiffness of the shoulder in the throwing position of abduction and external rotation. This highlights the fact that the superior labral complex plays an important role in the stability of the shoulder. Biomechanical studies have shown that SLAP lesions play a role in destabilizing the shoulder and cause increased strain on the inferior glenohumeral ligament in the throwing position. Increased biceps muscle force allows the abducted and externally rotated shoulder joint to withstand higher external rotation forces (torque).Correct Answer: Decreasing torsional rigidity

       

       

      3100. (2683) Q7-3176:

      Dynamic muscular stabilizers of the shoulder play an important role in stability. Which of the following is the most important dynamic stabilizer:

       

      1) The rotator cuff

      3) The coracobrachialis

      2) The labrum

      5) The biceps brachii

      4) The latissimus dorsi

       

      The rotator cuff is an important dynamic anterior stabilizer of the shoulder. The stability of the shoulder is maintained primarily by the shoulder musculature. The biceps brachii has been shown to act as a dynamic stabilizer of the shoulder, but the most important dynamic stabilizer is the rotator cuff.Correct Answer: The rotator cuff

       

       

      3101. (2684) Q7-3177:

      Which of the following is not true regarding the function of the biceps in the shoulder:

       

      1) The biceps helps prevent proximal migration.

      3) The biceps provides a dynamic stabilizing effect on the shoulder.

      2) The biceps plays an important role in anterior stability.

      5) The biceps provides most of the dynamic stability of the shoulder joint.

      4) The biceps increases the torsional stiffness of the shoulder in the throwing position.

       

      The rotator cuff is the most important dynamic stabilizer of the shoulder. The biceps acts as a depressor of the humeral head against proximal migration. Biomechanical studies have demonstrated the importance of the biceps in anterior stability as well as its role as a dynamic stabilizer. Studies have also shown that the biceps increases the torsional stiffness of the shoulder in the throwing position.Correct Answer: The biceps provides most of the dynamic stability of the shoulder joint.

       

       

      3102. (2685) Q7-3178:

      Which of the following statements is not true regarding the shoulder capsule:

       

      1) The rate of capsular stretch may determine where the capsular ligament is damaged.

      3) Anterior shoulder dislocations usually cause some degree of capsular stretching.

      2) Slower strain rates tend to cause disruption at the ligament insertion site.

      5) Capsular injury is a common arthroscopic finding in patients who sustain an acute shoulder dislocation.

      4) The shoulder capsule tends to be tight in patients with multidirectional instability.

       

      Biomechanical studies have shown that the rate of speed of injury may determine where the capsule is injured. Faster strain rates tend to cause more ligament intrasubstance injury, and slower strain rates tend to cause a failure at the ligament insertion site.

      Arthroscopic studies have documented that some degree of capsular injury is present in almost all patients after an acute shoulder dislocation. The shoulder capsule tends to be lax and patulous in patients with symptomatic multidirectional instability.Correct Answer: The shoulder capsule tends to be tight in patients with multidirectional instability.

       

      Which of the following is not considered a mechanism of injury for a superior labrum anterior and posterior (SLAP) tear:

       

      1) Traction injury from carrying, dropping, or lifting a heavy object

      3) Repetitive overhead throwing

      2) Compression force from a fall on an outstretched arm

      5) External rotation movement with returning a backhand in tennis

      4) Forceful biceps contraction with throwing a ball or spiking a volleyball

       

      The other mechanisms of injury have been reported in patients with documented SLAP tears. The series reported by Snyder and colleagues reported that a compressive force from a fall on an outstretched arm was the most common mechanism of injury.

      However in their study, 21% of the patients with a SLAP lesion reported an insidious onset to their symptoms. Maffet and colleagues reported that two-thirds of patients described a traction injury as the initial traumatic event. In their study, only 8% of patients sustained a fall on their outstretched arm. Most throwing athletes examined by Andrews and colleagues failed to report a distinct traumatic event. Thus, while an isolated injury may cause a labral injury, SLAP lesions also may occur as a result of the repetitive microtrauma associated with overhead motions.Correct Answer: External rotation movement with returning a backhand in tennis

       

       

       

      3104. (2687) Q7-3180:

      The belly press test is used in testing the integrity of which of the following structures:

       

      1) The abdominal muscles

      3) The subscapularis tendon

      2) The supraspinatus tendon

      5) The teres minor tendon

      4) The infraspinatus tendon

       

      The subscapularis is tested using the belly press test and lift-off test. The supraspinatus is tested using the Jobe test. The infraspinatus is tested with the external rotation lag sign and by testing external rotation strength with the arm at the side. The teres minor is tested using the horn blowerâs sign.Correct Answer: The subscapularis tendon

       

       

      3105. (2688) Q7-3181:

      The lift-off test is used in testing the integrity of which of the following structures:

       

      1) The abdominal muscles

      3) The subscapularis tendon

      2) The supraspinatus tendon

      5) The teres minor tendon

      4) The infraspinatus tendon

       

      The subscapularis is tested using the belly press test and lift-off test. The supraspinatus is tested using the Jobe test. The infraspinatus is tested with the external rotation lag sign and by testing external rotation strength with the arm at the side. The teres minor is tested using the horn blowerâs sign.Correct Answer: The subscapularis tendon

       

       

      3106. (2807) Q7-3305:

      Surferâs ear is characterized by:

       

      1) Injury of the ears while surfing

      3) Damage of ear cartilage

      2) Middle ear inflammation

      5) None of the above

      4) Exostosis of the external acoustic meatus

       

      Many sports have associated injuries, for instance there is the tennis elbow, skier's thumb, and boxer's ear. For the sport of surfing there is a condition known as surfer's ear. This injury involves an exostosis of the external acoustic meatus with obstructions of differing degrees.

       

      A study of 51 surfing professionals was conducted in Japan to determine the frequency of surfer's ear. Study participants surfed approximately 300 days per year and were an average of 26 years of age. It was found that 41 of the 51 surfers (80%) had surfer's ear. Nineteen of the 51 surfers (37%) were identified as having more than a 50% stenosis of the external acoustic meatus in 30 ears.

       

      Correct Answer: Exostosis of the external acoustic meatus

       

       

      3107. (2891) Q7-3392:

      Lateral epicondylitis of the elbow is more common than medial epicondylitis in all of the following populations except:

       

      1) Tennis players

      3) Plumbers

      2) Electricians

      5) Tailors

      1. Golfers

         

        Lateral epicondylitis is a pathological condition involving the origin of the common extensors (extensor carpi radialis longus, extensor carpi radialis brevis, extensor carpi digitorum communis, and extensor carpi ulnaris); hence, any activity, whether recreational or occupational, involving repetitive stressing of the wrist extensors makes an individual susceptible to lateral epicondylitis. In addition to tennis, other sports implicated in the development of lateral epicondylitis include baseball and associated sports involving throwing activities, fencing, martial arts, and swimming. Occupations particularly prone to development of lateral epicondylitis often involve tedious repetitive activities requiring manual dexterity such as electronics repair, plumbing, and tailoring. Medial epicondylitis is more common than lateral epicondylitis in golfers.Correct Answer: Golfers

         

         

        3108. (2892) Q7-3393:

        What is the approximate lifetime prevalence of lateral epicondylitis of the elbow in avid tennis players:

         

        1) 10%

        3) 50%

        2) 30%

      2. 95%

  4. 70%

     

    Although the overall incidence of lateral epicondylitis in the general population is unclear, approximately 50% of avid tennis players will experience this condition during their lifetime. The condition usually occurs during the fourth or fifth decade, but can occur across a broad spectrum of ages, and affects men and women indiscriminately.Correct Answer: 50%

     

     

    3109. (2893) Q7-3394:

    Which of the following is the most commonly injured joint in childhood sports:

     

    1) Ankle

    3) Hip

    2) Elbow

    5) Knee

    4) Shoulder

     

    The knee is the most commonly injured joint in childhood sports. With the increasing participation of children in competitive athletics, midsubstance disruption of the anterior cruciate ligament (ACL) in skeletally immature individuals is becoming more common. Additionally, increasing awareness of this injury and improved imaging modalities have made ACL injuries more readily recognizable in the pediatric population.Correct Answer: Knee

     

     

    3110. (2894) Q7-3395:

    Subacromial impingement syndrome typically affects patients in which decade of life:

     

    1. 1st

      3) 5th

      2) 3rd

      5) 9th

      4) 7th

       

      Patients with subacromial impingement syndrome are usually older than 40 years of age with a chief complaint of shoulder pain. Patients often recall no specific injury to their shoulder, but onset of pain may be related to a recent increase in overhead activity following a period of relative inactivity. Pain initially only occurs with overhead use of the arm but may progress to persistent pain with any use of the arm or even nighttime pain in extreme cases. The dominant extremity is more often affected by virtue of increased utilization.Correct Answer: 5th

       

      Which of the following is the most commonly dislocated joint in the body:

       

      1. Elbow

        3) Hip

        2) Shoulder

        5) Ankle

        1. Knee

           

          The shoulder is the most commonly dislocated major joint in the body, with the incidence of traumatic anterior shoulder dislocation approaching 2%. Traumatic recurrent anterior shoulder instability can also take the form of recurrent subluxation or simply a chronically painful shoulder without a defined history of dislocation.Correct Answer: Shoulder

           

           

          3112. (2896) Q7-3397:

          What percentage of patients with recurrent anterior shoulder instability has an identifiable abnormality on plain radiography:

           

          1) 10%

          3) 75%

          2) 25%

        2. 95%

        4) 85%

         

        Plain radiography can yield osseous abnormalities in up to 95% of shoulders with chronic traumatic anterior shoulder instability, including 75% with humeral head impaction fractures and 85% with a bony glenoid lesion. Bony glenoid lesions include a frank fracture, which can be identified on an anteroposterior view or a glenoid profile view; loss of normal contour of the anterior glenoid rim caused by multiple microimpaction fractures of the anterior glenoid; or a complete bony deficiency of the anterior glenoid caused by resorption of a fracture fragment. Osseous abnormalities are most common in shoulders that have a history of prior dislocation and least common in shoulders that are simply painful with no established history of dislocation or subluxation.Correct Answer: 95%

         

         

         

        3113. (2897) Q7-3398:

        What percentage of patients with recurrent anterior shoulder instability has an identifiable humeral head abnormality on plain radiography:

         

        1) 10%

        1. 75%

      2. 25%

  5. 95%

  1. 85%

     

    Plain radiography can yield osseous abnormalities in up to 95% of shoulders with chronic traumatic anterior shoulder instability, including 75% with humeral head impaction fractures and 85% with a bony glenoid lesion. Bony glenoid lesions include a frank fracture, which can be identified on an anteroposterior view or a glenoid profile view (Figure 8); loss of normal contour of the anterior glenoid rim caused by multiple microimpaction fractures of the anterior glenoid; or a complete bony deficiency of the anterior glenoid caused by resorption of a fracture fragment. Osseous abnormalities are most common in shoulders that have a history of prior dislocation and least common in shoulders that are simply painful with no established history of dislocation or subluxation.Correct Answer: 75%

     

     

    What percentage of patients with recurrent anterior shoulder instability has an identifiable glenoid abnormality on plain radiography:

     

    1) 10%

    1. 75%

      1. 25%

  2. 95%

  1. 85%

 

Plain radiography can yield osseous abnormalities in up to 95% of shoulders with chronic traumatic anterior shoulder instability, including 75% with humeral head impaction fractures and 85% with a bony glenoid lesion. Bony glenoid lesions include a frank fracture, which can be identified on an anteroposterior view or a glenoid profile view; loss of normal contour of the anterior glenoid rim caused by multiple microimpaction fractures of the anterior glenoid; or a complete bony deficiency of the anterior glenoid caused by resorption of a fracture fragment. Osseous abnormalities are most common in shoulders that have a history of prior dislocation and least common in shoulders that are simply painful with no established history of dislocation or subluxation.Correct Answer: 85%

 

 

 

3115. (2899) Q7-3400:

Which of the following represents the most common scenario of injury resulting in athletic pubalgia:

 

  1. Repetitive hyperextension of the trunk in association with hyperabduction of the thigh pivoting on the anterior pelvis and pubic symphysis in a female athlete

3) A single forced hyperextension of the trunk in association with hyperabduction of the thigh pivoting on the anterior pelvis and pubic symphysis in a female athlete

2) Repetitive hyperextension of the trunk in association with hyperabduction of the thigh pivoting on the anterior pelvis and pubic symphysis in a male athlete

5) A single forced hyperflexion of the trunk in association with hyperabduction of the thigh pivoting on the anterior pelvis and pubic symphysis in a male athlete

4) A single forced hyperextension of the trunk in association with hyperabduction of the thigh pivoting on the anterior pelvis and pubic symphysis in a male athlete

 

The rectus insertion on the pubis with or without the origin of the adductor longus tendon appears to be the primary site of pathology. Athletic pubalgia has been described primarily in high-level athletes and almost exclusively in men. The proposed mechanism of injury involves repetitive hyperextension of the trunk in association with hyperabduction of the thigh pivoting on the anterior pelvis and pubic symphysis.Correct Answer: Repetitive hyperextension of the trunk in association with hyperabduction of the thigh pivoting on the anterior pelvis and pubic symphysis in a male athlete

 

 

3116. (2935) Q7-3438:

Medial tibial stress syndrome most commonly occurs in which of the following locations:

 

1) Lateral tibia

3) Superior and middle thirds of the posteromedial tibia

2) Superior third of the posteromedial tibia

5) Medial malleolus

4) Middle and inferior thirds of the posteromedial tibia

 

Medial tibial stress syndrome most commonly occurs over the middle and inferior thirds of the medial tibia and is generally localized to an area 4 cm proximal to the medial malleolus, extending to the junction of the middle and superior aspect of the posteromedial tibia.Correct Answer: Middle and inferior thirds of the posteromedial tibia

 

 

3117. (2936) Q7-3439:

The skeletal pathology most often associated with medial tibial stress syndrome is:

 

1) Varus alignment of the knee

3) Forefoot pronation

2) Valgus alignment of the knee

5) Cavus deformity of the foot

4) Forefoot supination

 

Medial tibial stress syndrome is generally associated with activity changes or overuse. However, some evidence points to an increased incidence in patients who have excessive forefoot pronation. Orthotics can be used to reduce forefoot pronation.Correct Answer: Forefoot pronation

 

3118. (2937) Q7-3440:

Which of the following is the most common theory for the pathophysiology of medial tibial stress syndrome:

 

1) Medial tibial stress syndrome is a variant of chronic compartment syndrome.

3) Medial tibial stress syndrome is a stress fracture.

2) Increased pressure in the anterior compartment causes pain.

5) Medial tibial stress syndrome is a variant of chronic regional pain syndrome.

4) A stress response in the bone results in osteoclastic remodeling that can cause pain.

 

Medial tibial stress syndrome is thought to be a stress response in the bone. The bone is resorbed by osteoclasts in the process of remodeling. Resorption is a direct response to excessive stress applied by overuse. The bone is then replaced by osteoblastic activity.Correct Answer: A stress response in the bone results in osteoclastic remodeling that can cause pain.

 

 

3119. (2938) Q7-3441:

Which of the following is a major finding on physical examination for an athlete with medial tibial stress syndrome:

 

1) Increased pressure in the anterior compartment 5 minutes after exercise

3) Pain with passive ankle range of motion

2) Pain with passive knee range of motion

5) Tenderness to palpation over the anteromedial border of the tibia

4) Tenderness to palpation over the posteromedial border of the tibia

 

Patients with medial tibial stress syndrome often have most of their pain over the posteromedial border of the tibia. Pain is elicited with direct palpation. Palpation of other portions of the tibia should not cause pain. Occaionally, patients will have pain with resisted ankle range of motion, but not with passive ankle or knee motion.Correct Answer: Tenderness to palpation over the posteromedial border of the tibia

 

 

3120. (2939) Q7-3442:

What is the most common radiographic finding for a patient with medial tibial stress syndrome:

 

1) Normal radiographs

3) Traction osteophytes of medial malleolus

2) Osteopenia of the mid to distal tibia

5) Diffuse thickening of the tibial cortex circumferentially

4) Lucent horizontal line extending from the medial cortex

 

The most common radiographic finding in a patient with medial tibial stress syndrome is a normal radiograph. Occasionally, hypertrophy of the posteromedial tibial cortex may be present. This hypertrophy is not circumferential. Lucent lines are indicative of a stress fracture. Osteopenia and traction osteophytes do not play a role in medial tibial stress syndrome.Correct Answer: Normal radiographs

 

 

3121. (2940) Q7-3443:

What are the main findings of medial tibial stress syndrome on a bone scan:

 

1) Delayed uptake of tracer and nonfocal uptake over the posteromedial tibial border

3) Focal uptake of tracer in early phase only

2) Lack of uptake of tracer in all phases

5) Nonspecific uptake of tracer in all phases

4) Focal uptake of tracer in delayed phases

 

Medial tibial stress syndrome has a characteristic finding on bone scan of uptake only in the delayed phase. Uptake is nonfocal and is along the posteromedial border of the tibia. These findings contrast with a stress fracture, which has focal uptake in the early phase.Correct Answer: Delayed uptake of tracer and nonfocal uptake over the posteromedial tibial border

 

The primary intervention for patients with medial tibial stress syndrome is:

 

1) Surgery with release of the posteromedial fascia

3) Viscosupplementation of the knee

2) Cortisone injections at the insertion of the soleus on the posteromedial tibia

5) Increasing activities until pain subsides

4) Activity modification, icing after activities, and nonsteroidal anti-inflammatory medication

 

Most cases of medial tibial stress syndrome resolve with relative rest and icing after activities. Nonsteroidal anti-inflammatory medications may also be useful. Cortisone injections and viscosupplementation do not play a role in medial tibial stress syndrome. Surgery for medial tibial stress syndrome includes release of the soleus fascia; however, surgery is rarely indicated.Correct Answer: Activity modification, icing after activities, and nonsteroidal anti-inflammatory medication

 

 

3123. (2942) Q7-3445:

What is the most common site of pelvic avulsion fracture in a skeletally immature athlete:

 

1) Anterior inferior iliac spine (AIIS)

3) Anterior superior iliac spine (ASIS)

2) Ischial tuberosity

5) Pubic symphysis

4) Iliac crest

 

The most common site of a pelvic avulsion fracture in a skeletally immature athlete is the ischial tuberosity. A pelvic avulsion fracture in a skeletally immature patient is the equivalent of a proximal hamstring injury in an adult. The second most common injuries are to the AIIS and ASIS. Injuries also occur to the pubic symphysis and iliac crest, but they are less common.Correct Answer: Ischial tuberosity

 

 

3124. (2943) Q7-3446:

Which of the following muscle origins is responsible for an avulsion injury of the anterior inferior iliac spine (AIIS) in a skeletally immature athlete:

 

1) Sartorius

3) Rectus femoris

2) Adductor magnus

5) External oblique

4) Semimembranosus

 

The rectus femoris originates from the AIIS and is responsible for avulsion injuries to the AIIS in skeletally immature athletes. The sartorius originates from the anterior superioe iliac spine. The adductor magnus originates from the pubic symphysis and infrapubic tubercle. The semimembranosus originates from the ischial tuberosity. The external oblique inserts on the iliac crest.Correct Answer: Rectus femoris

 

 

3125. (2944) Q7-3447:

What are the muscle actions of the sartorius on the hip and knee joints:

 

1) Hip extension, knee flexion

3) Hip extension, knee extension

2) Hip flexion, knee extension

5) Hip flexion, knee flexion

4) Hip external rotation, knee extension

 

The sartorius crosses the hip and knee joints, crossing the hip anteriorly and the knee posterior to its axis of rotation. Therefore, the sartorius flexes the hip and flexes the knee.Correct Answer: Hip flexion, knee flexion

 

Which myotendinous junction is the most common site for proximal hamstring injury:

 

1) Semimembranosus

3) Biceps femoris

2) Semitendinosus

5) Gracilis

4) Rectus femoris

 

The biceps femoris proximal musculotendinous junction is the most commonly injured muscle in proximal hamstring injuries. The other hamstring muscles that take their origin from the ischium are the semitendinosus and semimembranosus. The rectus femoris and gracilis take their origins from the anterior inferior iliac spine and anterior superior iliac spine, respectively.Correct Answer: Biceps femoris

 

 

3127. (2946) Q7-3449:

Greater trochanteric bursitis is most common among:

 

1) Female runners

3) Female tennis players

2) Male runners

5) Male baseball players

4) Male skiers

 

Trochanteric bursitis is common in female runners, likely due to their broad-based pelvis as compared to their male counterparts. Some runners may identify a causative factor in their symptoms such as an increase in their mileage or an increase in the level of difficulty of their training course. Additionally, if running is done on the road, often one leg will be affected. This leg is generally the leg on the outer side of the road and is affected by the drainage slope incorporated into the design of roads. Other provocative activities include lying on the affected side.Correct Answer: Female runners

 

 

3128. (2953) Q7-3458:

The most common cause of an indirect injury to the Lisfranc joint occurs through which mechanism:

 

1) Compression

3) Axial load of a plantarflexed foot

2) Hyperdorsiflexion

5) Pronation and adduction

4) Supination and external rotation

 

The indirect mechanism of injury involves axial loading of a plantarflexed foot. This type of mechanism is commonly cited in soccer, basketball, and gymnastics. The most frequent pattern in the indirect mechanism is failure of the weaker dorsal tarsometatarsal ligaments in tension with dorsal dislocation of the metatarsals.Correct Answer: Axial load of a plantarflexed foot

 

 

 

 

 

Slide 1

A 19-year-old college gymnast fell off the balance beam during competition, sustaining an injury to her right foot. She presents to your office, and a radiograph (Slide) is obtained. What is the preferred method of treatment to optimize her result:

 

1) Closed reduction and short leg non-weight bearing cast

3) Closed reduction and percutaneous pinning

2) Closed reduction and short leg walking cast

5) Observation

4) Open reduction and internal fixation

 

According to investigators, a displacement of more than 2 mm requires open reduction and internal fixation to avoid a poor outcome, especially in athletes. While some orthopedists prefer closed fixation with percutaneous K-wires, others report that this method does not hold anatomic reduction and fixation. An alternative method involves the use of open reduction and internal fixation with AO screw fixation. An open surgical field allows easier removal of fragments or soft tissue that may be hindering reduction of the dislocation.Correct Answer: Open reduction and internal fixation

 

 

3130. (3175) Q7-3994:

The triceps tendon insertion has two well-defined components. The triceps proper inserts:

 

1) Directly onto the posterior 40% of the olecranon

3) On to the entirety of the olecranon tip

2) On to the anconeous muscle

5) None of the above

4) On to the anterior aspect of the olecranon

 

A thorough understanding of the insertional anatomy of the extensor mechanism of the elbow is essential in understanding the treatment principle for chronic triceps tendonitis in throwers. Approximately 20 cm proximal to the olecranon tip, the triceps tendon originates from within the body of the triceps muscle. Two components of the triceps extensor mechanism form as the tendon approaches its insertion on the olecranon tip. The triceps thickened proper inserts directly onto the posterior 40% of the tip of the olecranon. The relatively thin triceps expansion inserts distally and laterally primarily through the anconeus. The triceps decussation, which is well defined in 50% of specimens, is a well-defined interval located between the triceps proper and the triceps expansion.Correct Answer: Directly onto the posterior 40% of the olecranon

 

 

 

3131. (3432) Q7-4348:

Which of the following is necessary for optimal tendon-bone healing:

 

1) No sling postoperatively

3) Open technique

2) Accelerated physical therapy

5) Loose approximation of tissues

4) Decreased motion at the bone-tendon junction

 

Factors involved in optimal tendon-to-bone healing include footprint coverage, contact pressure, suture strength, loop and knot security, and decreased motion at the bone-tendon junction. Gerber and colleagues suggested that the ideal repair provides high initial fixation strength, minimizes gap formation, and maintains mechanical stability until sufficient healing has occurred.Correct Answer: Decreased motion at the bone-tendon junction

 

Postoperatively, an intact rotator cuff correlates with which physical examination finding:

 

1) More pain

3) Less stiffness

2) More strength in external rotation and forward elevation

5) Better patient satisfaction

4) Less stability

 

Previous studies showed no significant difference in functional or pain scores based on tendon integrity, but strength in forward elevation and external rotation were significantly correlated with an intact repair. Patient satisfaction is not significantly associated with an intact tendon repair.Correct Answer: More strength in external rotation and forward elevation

 

 

3133. (3465) Q7-4401:

An 18-year-old soccer player sustains an injury to his foot. He described the injury as axial loading of a plantarflexed foot. He presents with a swollen and painful foot and is unable to put weight on his limb. After an initial radiograph, what is the next study you would obtain:

 

1) Computed tomography scan

3) Oblique radiograph

2) Sress radiographs with the forefoot in dorsiflexion and abduction

5) Bone scan

4) Magnetic resonance image

 

Standing radiographs are ideal but often times too painful. A good alternative is stress radiographs with the forefoot in dorsiflexion and abduction. On the radiographs, dislocation of the tarsometatarsal joint is indicated by the following: loss of in-line arrangement of the lateral margin of the first metatarsal base with the lateral edge of the medial cuneiform, loss of in-line arrangement of the medial margin of the second metatarsal base with the medial edge of the middle cuneiform in the weight bearing anteroposterior view, and the presence of small avulsed fragments (fleck sign), which are further indications of ligamentous injury and probable joint disruption.Correct Answer: Sress radiographs with the forefoot in dorsiflexion and abduction

 

 

3134. (3478) Q7-4417:

L-shaped tears differ from crescent shaped tears because L-shaped tears:

 

1) Violate the anterior (or posterior) attachment of the supraspinatus

3) Are always more mobile

2) Must involve at least 2 tendons

5) Are never acute

  1. Are less likely to retract more medial to the glenoid

     

    Crescent, L-shaped, and reverse L-shaped tears comprise approximately 90% of all tears, and although they may become large, they are usually mobile and rarely retract medial to the glenoid. The crescent tear is different from an L- or reverse-L shaped tear because it does not violate either the anterior or posterior attachment of the supraspinatus tendon to the greater tuberosity. L-shaped tears can have either an anterior or posterior flap (reverse-L) and require similar surgical considerations to the U-shaped tear.Correct Answer: Violate the anterior (or posterior) attachment of the supraspinatus

     

     

    3135. (3483) Q7-4423:

    Approximately how much of the native supraspinatus footprint can be restored using single-row suture anchor fixation:

     

    1) 5%

    3) 50%

    2) 20%

  2. 95%

4) 80%

 

In vivo examination of the footprint showed that with a single lateral row of suture anchors, 47% of the footprint was reestablished and, with the addition of a medial row, the entire footprint was reestablished.Correct Answer: 50%

 

The Lisfranc ligament passes between which two bony structures:

 

  1. Medial cuneiform and second metatarsal

3) Medial cuneiform and first metatarsal

2) Middle cuneiform and first metatarsal

5) Middle cuneiform and second metatarsal

4) Lateral cuneiform and first metatarsal

 

The proximal end of the second metatarsal is recessed between the medial and lateral cuneiform. This mortise configuration locks the tarsometatarsal complex. No ligament binds the first and second metatarsal. The main stabilizer of this relationship is the Lisfranc ligament. The Lisfranc ligament is a strong oblique ligament that extends from the plantar-lateral aspect of the medial cuneiform and inserts into the plantar medial second metatarsal.Correct Answer: Medial cuneiform and second metatarsal

 

 

3137. (3506) Q7-4458:

What has been identified as a significant risk factor for re-tear of a primary supraspinatus tendon repair:

 

1) Original tear >3 cm in size

3) Age ≤50

2) Dominant extremity

5) Male gender

4) Immobilization

 

When stratified based on original tear size, repair of a tear >3 cm in size had a seven times greater likelihood for re-tear.Correct Answer: Original tear >3 cm in size

 

 

3138. (3507) Q7-4459:

The maximal contraction force generated by the supraspinatus muscle is approximately:

 

1) 3 N

3) 300 N

2) 30 N

5) 5000 N

4) 3000 N

 

Based on the average cross-sectional area of the supraspinatus muscle and the maximal contraction of muscle per unit area, Burkhart determined that the maximal contraction force generated by the supraspinatus is equal to approximately 302 N.Correct Answer: 300 N

 

 

3139. (3539) Q7-4493:

What is the main functional deficit after distal biceps tendon ruptures:

 

1) Loss of extension strength only

3) Loss of extension strength and supination strength

2) Loss of flexion strength only

5) Loss of supination strength only

4) Loss of flexion strength and supination strength

 

The main functional deficit after biceps tendon rupture is a 40% loss of supination strength, 47% loss of supination endurance, and a 21% to 30% loss of flexion strength at the elbow when compared to the uninjured extremity.Correct Answer: Loss of flexion strength and supination strength

 

Which of the following patients is the least ideal candidate for surgical treatment of a distal biceps rupture:

 

1) A 45-year-old mechanic

3) A 57-year-old firefighter

2) A 40-year-old professional weight lifter

5) A 53-year-old retiree with rheumatoid arthritis

4) A 28-year-old Olympic pole vaulter

 

Surgical treatment of distal biceps injury re-establishes flexion and supination strength that is essential in laborers, athletes, and weight lifters. A 53-year-old male retiree with rheumatoid arthritis is unlikely to be active enough to benefit from surgical intervention.Correct Answer: A 53-year-old retiree with rheumatoid arthritis

 

 

3141. (3541) Q7-4495:

Which complication is associated with the anterior one-incision surgical approach for distal biceps tendon repair:

 

1) Radial nerve injury

3) Radioulnar synostosis

2) Heterotopic ossification

5) Median nerve Injury

4) Ulnar nerve injury

 

The radial nerve can be injured during the anterior one-incision approach to the elbow. To avoid this injury, maintain supination of the forearm during dissection of the radial tuberosity.Correct Answer: Radial nerve injury

 

 

3142. (3542) Q7-4496:

Which fixation device used during repair of the distal biceps tendon has shown the greatest load to failure:

 

1) Interference screw

3) Transosseous bone tunnel technique

2) EndoButton (Smith and Nephew, Andover, Mass)

5) Suture anchor and interference screw combination technique

4) Suture anchors

 

In a study by Mazzocca and colleagues, the EndoButton had the highest load to failure when compared to interference screws, a bone tunnel technique, and suture anchors.Correct Answer: EndoButton (Smith and Nephew, Andover, Mass)

 

 

3143. (3543) Q7-4497:

What is the main risk associated with the two-incision approach for distal biceps tendon repair:

 

1) Median nerve injury

3) Radioulnar synostosis

2) Radial tuberosity fracture

5) Transient ulnar neuropathy

4) Radial nerve injury

 

The Boyd and Anderson two-incision technique has classically been associated with heterotopic ossification leading to radioulnar synostosis. This approach has been modified to a muscle-splitting approach, limiting periosteal dissection and decreasing the risk of heterotopic ossification.Correct Answer: Radioulnar synostosis

 

 

 

1) Nonanatomically to the underlying brachialis muscle with the arm in neutral rotation

3) Anatomically to the ulnar aspect of the radial tuberosity with the arm pronated

2) Anatomically to the radial aspect of the radial tuberosity with the arm pronated

5) Anatomically to the radial aspect of the radial tuberosity with the arm supinated

4) Anatomically to the ulnar aspect of the radial tuberosity with the arm supinated

 

The distal biceps tendon attaches in a ribbon-like fashion to the ulnar aspect of the radial tuberosity. During the anterior approach, the arm should be held in supination to avoid the radial nerve.Correct Answer: Anatomically to the ulnar aspect of the radial tuberosity with the arm supinated

 

 

3145. (3545) Q7-4499:

A 46-year-old plumber ruptured the distal biceps tendon in his dominant extremity 8 months ago. He was treated initially with 1 week of immobilization followed by a short course of physical therapy. He states he feels very fatigued at the end of the day and that easy tasks are taking him longer than normal to complete. What is the best treatment option for this patient:

 

1) Nonoperative treatment consisting of aggressive physical and occupational therapy focusing on elbow and wrist stretching and strengthening exercises

3) Anatomic primary repair via a two-incision posterior approach

2) Anatomic primary repair via an anterior one-incision approach

5) Anatomic reconstruction using a posterior tibial tendon autograft via a two-incision posterior approach

4) Anatomic reconstruction using a semitendinosus autograft via a one-incision anterior approach

 

This laborer has a chronic distal biceps tendon rupture, which is affecting his job. Reconstruction with autograft (or allograft) supplementation over primary repair is indicated as his biceps tendon is likely retracted and scarred down. The technique of reconstruction of a chronic distal biceps tendon reconstruction with a semitendinosus autograft using a single-incision technique has been described yielding excellent results. Use of a posterior tibial tendon autograft has not been described.Correct Answer: Anatomic reconstruction using a semitendinosus autograft via a one-incision anterior approach

 

 

3146. (3665) Q7-7442:

The most common type of femoroacetabular impingement is:

 

1) Pincer type

3) Combined (both pincer and cam)

2) Cam type

5) Dysplasia

4) Retroversion

 

The most common type of femoroacetabular impingement is the combined cam and pincer type.Correct Answer: Combined (both pincer and cam)

 

 

3147. (3666) Q7-7443:

Which of the following is not an indication for hip arthroscopy:

 

1) Loose body

3) Femoroacetabular impingement

2) Labral tear

5) Infection

4) Osteoarthritis

 

Osteoarthritis is not an indication for hip arthroscopy. While there may be a limited role for arthroscopic intervention in specific patients, generally, advanced osteoarthritis is best treated with arthroplasty options.Correct Answer: Osteoarthritis

 

 

 

1) Anterior paratrochanteric portal

3) Direct anterior portal

2) Posterior paratrochanteric portal

4) Mid-anterior (accessory anterolateral) portal

 

Lateral femoral cutaneous nerve palsy is one of the more common complications of hip arthroscopy and has been attributed to its proximity to the anterior portal.Correct Answer: Direct anterior portal

 

 

3149. (3668) Q7-7445:

The nerve that is most commonly compressed as the result of traction over a perineal post is:

 

1) Sciatic

3) Peroneal

2) Superior gluteal

5) Pudendal

4) Lateral femoral cutaneous

 

The pudental nerve is commonly injured from compression against a perineal post, resulting in sensory loss to the genitalia.Correct Answer: Pudendal

 

 

3150. (3669) Q7-7446:

All of the following are noted advantages of the supine position compared to the lateral position, except:

 

1) Familiar orientation for most surgeons

3) Shorter positioning time

2) Can be performed on standard fracture table

4) Easier for obese patients

 

The first 3 listed statements are true advantages of the supine position, while the ease of arthroscopy in obese patients is a noted advantage of the lateral position during hip arthroplasty, as the panniculus and buttock fall away from the operative field.Correct Answer: Easier for obese patients

 

 

3151. (3774) Q7-7556:

Which physical examination finding is most consistent in a patient with adhesive capsulitis:

 

1) Loss of active range of motion

3) Loss of internal rotation

2) Loss of passive and active range of motion

5) Loss of strength

4) Loss of abduction

 

The physical examination and history are the keys to early diagnosis and treatment. Adhesive capsulitis involves a loss of active and passive range of motion. Patients with rotator cuff tears may lose active range of motion, but passive range of motion is usually preserved.

 

Correct Answer: Loss of passive and active range of motion

 

An intra-articular corticosteroid injection is most effective during this stage of adhesive capsulitis:

 

1) Stage 1 and 2

3) Stage 3 and 4

2) Stage 2 and 3

5) A diagnostic, local anesthetic injection is more useful.

4) An intra-articular corticosteroid injection is never effective in relieving adhesive capsulitis.

 

 

An intra-articular corticosteroid injection prior to the onset of significant fibrosis can alter the progression of adhesive capsulitis. Correct Answer: Stage 1 and 2

 

3153. (3776) Q7-7558:

What is the expected outcome with nonoperative treatment of adhesive capsulitis:

 

1) All symptoms and signs resolve with no residual effects.

3) Patients have pain with overhead activity and subacromial impingement but regain full range of motion.

2) Patients are always pain free but typically lose 50% of their range of motion.

5) Nonoperative treatment is ineffective and arthroscopic or open capsular release is recommended.

4) Approximately 75% to 90% of patients are satisfied with nonoperative treatment and regain about 80% active range of motion compared to their unaffected shoulder.

 

Although the original descriptions of frozen shoulder and adhesive capsulitis suggest the course is benign and self-limiting, studies by Shaffer and colleagues and Griggs and researchers suggest that while most patients do well, 10% to 25% of patients still have some pain or loss of motion.

 

Correct Answer: Approximately 75% to 90% of patients are satisfied with nonoperative treatment and regain about 80% active range of motion compared to their unaffected shoulder.

 

 

3154. (3777) Q7-7559:

Which of the follow groups are expected to have a higher incidence of recurrent adhesive capsulitis and more residual symptoms after nonoperative treatment:

 

1) Men and patients with diabetes

3) Men and idiopathic

2) Women and posttraumatic

5) Women and patients with diabetes

4) Women and hyperthyroidism

 

Men are afflicted with adhesive capsulitis less commonly than women. Diabetes is one of the most common conditions to be found in conjunction with adhesive capsulitis. Studies such as that by Ogilvie-Harris and colleagues suggest that men and patients with diabetes are the most resistant to nonoperative treatment.

 

Correct Answer: Men and patients with diabetes

 

How is adhesive capsulitis differentiated from rotator cuff tears as a cause of shoulder pain:

 

1) The symptoms are identical; magnetic resonance imaging is needed to evaluate the rotator cuff.

3) A subacromial injection allows full range of motion for all stages of adhesive capsulitis.

2) Subacromial injection relieves the majority of pain related to rotator cuff tear, but the patient may still experience weakness if the tear is significantly large.

5) Patients are weak and cannot lift their arm above the shoulder with both adhesive capsulitis and rotator cuff tears.

4) Oral corticosteroids relieve the symptoms of rotator cuff tears.

 

This is perhaps the most critical early clinical diagnostic challenge when treating a patient complaining of a stiff or weak painful shoulder. While an intra-articular injection is helpful in the early treatment of adhesive capsulitis, a subacromial injection (such as that used with the impingement test), can help differentiate the two diagnoses.

 

Correct Answer: Subacromial injection relieves the majority of pain related to rotator cuff tear, but the patient may still experience weakness if the tear is significantly large.

 

 

3156. (3882) Q7-7761:

What is the most common reason for technical failure after anterior cruciate ligament (ACL) reconstruction:

 

1) Loss of fixation

3) Improper tensioning

2) Improper tunnel placement

5) Inadequate notchplasty

4) Insufficient graft harvest

 

Improper tunnel position is a commonly recognized technical error and reason for graft failure after ACL reconstruction. Tunnel malposition may result in limited range of motion, impingement, and excessive graft tension.

Correct Answer: Improper tunnel placement

 

 

3157. (3883) Q7-7762:

Which of the following is a cause for recurrent instability after anterior cruciate ligament (ACL) reconstruction within the first 6 months postoperative:

 

1) Technical error

3) Insufficient graft

2) Unrecognized patholaxity

4) All of the above

 

Insufficient graft, unrecognized patholaxity, and technical error including improper tunnel placement, fixation failure, and improper graft tensioning are common causes for early recurrent instability and poor outcomes.

Correct Answer: All of the above

 

 

3158. (3884) Q7-7763:

What is the most sensitive radiographic tool to assess for tunnel osteolysis preoperatively:

 

1) Standing anteroposterior radiographs

3) Computed tomography

2) Standing lateral radiographs

4) Magnetic resonance imaging

 

Computed tomography (CT) is a useful tool in preoperative planning for revision anterior cruciate ligament surgery. A CT scan helps delineate tunnel position, integrity, and expansion especially in the setting of a previous soft tissue or synthetic graft.

Correct Answer: Computed tomography

 

 

 

 

 

A vertically placed femoral tunnel as shown (Slide) may have what affect on knee biomechanics:

 

1) Translational control only

3) Negative Lachman examination

2) Rotational control only

5) A and C

4) Negative pivot shift examination

 

A vertically oriented graft may resist translation and thereby have a negative Lachman examination. However, vertical graft placement does not adequately control rotation and patients will often experience persistent instability and a positive pivot shift on examination.

 

Correct Answer: A and C

 

 

3160. (3886) Q7-7765:

When planning the tibial tunnel, the variable angle tibial aimer should create what angle with the long axis of the tibia to ensure proper tunnel obliquity:

 

1) 5°

3) 20°

2) 10°

5) 50°

4) 30°

 

A tibial tunnel reamed 30° relative to the long axis of the tibia will create an appropriate medial to lateral oblique orientation, which permits an anatomic femoral tunnel position when using a transtibial technique.

Correct Answer: 30°

 

 

3161. (4002) Q7-8254:

The most important divisions of the acromioclavicular (AC) ligament in the prevention of translation are the:

 

1) Anterior and superior divisions

3) Anterior and inferior divisions

2) Posterior and superior divisions

5) None of the above

4) Posterior and inferior divisions

 

The acromioclavicular (AC) ligament has superior, inferior, anterior, and posterior divisions. The AC ligaments provide stability in all planes when exposed to small forces. However, with larger forces, their major contribution is to resist displacement in the anterior-posterior plane. Klimkiewicz and associates reported that the superior and posterior divisions of the AC ligament are the most important for preventing posterior translation of the clavicle and abutment against the scapular spine.

 

Correct Answer: Posterior and superior divisions

 

Which anterior cruciate ligament graft material has the highest ultimate tensile load:

 

1) Bone-patellar tendon-bone

3) Achilles tendon

2) Four-strand hamstring tendons

5) Anterior tibialis tendon

4) Central third quadriceps tendon

 

Of the listed graft options, quadruple hamstrings possess the greatest ultimate tensile load (4090 N) and stiffness (776 N/mm). Bone-patellar tendon-bone grafts have an ultimate tensile load of 2977 N and a stiffness of 620 N/mm. Quadriceps tendon grafts have an ultimate tensile load of 2352 N and a stiffness of 463 N/mm.

 

Correct Answer: Four-strand hamstring tendons

 

 

3163. (4009) Q7-8261:

Which anterior cruciate ligament graft material will most quickly incorporate into host bone:

 

1) Four-strand hamstring tendon allograft

3) Bone-patellar tendon-bone allograft

2) Four-strand hamstring tendon autograft

5) Achilles tendon allograft

4) Bone-patellar tendon-bone autograft

 

Bone-patellar tendon-bone grafts heal to host bone within 6 weeks, while the other soft tissue grafts may require 8 to 12 weeks for incorporation into host bone. Although autografts and allografts follow the same sequence of incorporation into host bone, allografts have a slower rate of incorporation. Patellar tendon allograft tissue has demonstrated increased vascularity and inflammatory response, decreased cross-sectional area, less collagen remodeling, and increased anteroposterior translation than patellar tendon autograft at 6 weeks and 6 months following transplantation.

 

Correct Answer: Bone-patellar tendon-bone autograft

 

 

3164. (4011) Q7-8263:

A 20-year-old running-back at a Division I university ruptures his anterior cruciate ligament (ACL) during a game. What is the preferred graft material for his ACL reconstruction:

 

1) Bone-patellar tendon-bone allograft

3) Four-strand hamstring tendon allograft

2) Four-strand hamstring tendon autograft

5) Bone-patellar tendon-bone autograft

4) Quadriceps tendon autograft

 

Autograft is the appropriate graft in this patient, as opposed to an allograft. The bone-patellar tendon-bone autograft may offer some advantages over the other options with quicker incorporation and therefore possible quicker return to play.

Correct Answer: Bone-patellar tendon-bone autograft

 

A 40-year-old bricklayer tears his anterior cruciate ligament (ACL) after a fall from a ladder. Which of the following ACL graft options should be avoided in this patient:

 

1) Bone-patellar tendon-bone autograft

3) Four-strand hamstring tendon allograft

2) Quadriceps tendon allograft

5) Achilles tendon allograft

4) Four-strand hamstring tendon autograft

 

This patient is a good candidate for an allograft based on his age and activity level. Also, the bone-patellar tendon-bone should be avoided due to all of the kneeling in his occupation and possible anterior knee pain.

Correct Answer: Bone-patellar tendon-bone autograft

 

 

3166. (385) Q8-517:

What is the most common complication of interlocked cephalomedullary nail fixation of unstable intertrochanteric fractures when the piriformis fossa is intact, but the lesser trochanter is detached from the proximal fragment:

 

1) Non-union

3) Varus malreduction

2) Distraction of the fracture

5) Implant failure

4) Osteonecrosis of the femoral head

 

Subtrochanteric fractures account for approximately 15% of all proximal femur fractures. Some of the highest biomechanical stresses in the body occur in the subtrochanteric area (1200N).

Forty-five Russell-Taylor type IB subtrochanteric fractures were stabilized with interlocked cephalomedullary nails. Sixty-one percent of fractures were reduced in at least 5° of varus based on comparison of neck-shaft angles of the involved compared to the uninvolved side. All fractures went to union with no implant failure or report of osteonecrosis.

 

Nail insertion is technically demanding, requiring accurate nail length, extremity rotation, entry portal position, and proximal screw placement for good results. The posteromedial cortical buttress can be reconstructed to minimize the risk of varus displacement.Correct Answer: Varus malreduction

 

 

 

3167. (388) Q8-520:

During fixation of a periarticular fracture, a key fragment of bone falls to the operating room floor. If this piece of bone is to be placed back in the wound, the most appropriate method of sterilization is:

 

1) Saline rinse

3) Povidone-iodine scrub/thimerosal immersion

2) Povidone-iodine scrub/saline rinse

5) Povidone-iodine scrub/povidone-iodine immersion

4) Chlorhexidine gluconate scrub/antibiotic solution immersion

 

Contaminated fragments of bone may need to be replaced in the wound for a number of reasons. To evaluate the best method of sterilization, contaminated bovine metatarsal segments were treated with one of the following:

 

Saline rinse

 

 

Povidone-iodine scrub with saline rinse Povidone-iodine scrub/autoclaving

 

 

 

Povidone-iodine scrub/thimerosal immersion Povidone-iodine scrub/povidone-iodine immersion Chlorhexidine scrub/antibiotic immersion

 

The specimens were then cultured. Only chlorhexidine scrub/antibiotic immersion and povidone-iodine scrub/autoclaving were 100% effective in eliminating bacterial growth.Correct Answer: Chlorhexidine gluconate scrub/antibiotic solution immersion

 

 

When placing a transfixion wire in the proximal tibia from lateral to medial, the wire should be kept at least how far from the subchondral margin to remain extra-articular:

 

1) 5 mm

3) 20 mm

2) 14 mm

5) 30 mm

4) 25 mm

 

During proximal tibia transfixion pin placement, wires and pins should be kept outside of the boundary of the knee joint capsule to minimize the risk of developing an intra-articular infection. In a recent study, 35 cadavers were dissected to determine the location of capsular attachment to the proximal tibia. Arthroscopic measurements, as well as magnetic resonance imaging measurements, were compared to these findings. Four zones of attachment were defined:

 

 

Zone 1 (anterior) the capsule inserts 6 mm below the joint.

 

 

Zone 2 (posteromedial) the capsule inserts 12 mm below the joint. Zone 3 (posterior) the capsule inserts 30 mm below the joint.

 

Zone 4 (posterolateral) the capsule inserted on average 12 mm from the joint

Transfixing wires and half pins can be safely placed in the proximal tibia without capsular penetration if they are kept more than 14 mm from the subchondral line.Correct Answer: 14 mm

 

 

3169. (392) Q8-525:

The most significant predictor of fixation failure following internal fixation for displaced subcapital hip fractures is:

 

1) Initial displacement of fracture fragments

3) Preoperative comorbidity

2) Community ambulator

5) Varus reduction

4) Previous fixation failure on the contralateral side

 

Approximately 90% of hip fractures in the elderly occur from a simple fall. A total of 108 patients with displaced subcapital hip fractures treated with internal fixation were evaluated. When comparing preoperative radiograph information, clinical information, and postoperative radiograph information, the most important predictors of failure of fixation included varus reduction and perceived difficulty by the surgeon on achieving a reduction.

 

If a patient had a varus reduction or the surgeon had difficulty achieving a satisfactory reduction, fixation was 4.3 times more likely to fail. If both components existed together, fixation was 13.6 times more likely to fail.Correct Answer: Varus reduction

 

 

 

3170. (394) Q8-527:

Which of the following parameters of intramedullary reamers results in increased intramedullary pressure values:

 

1) Sharp reamer

3) Small shaft diameter relative to the reamer head

2) Shallow flutes

5) Slow advancement of the reamer

4) Multiple passes with each reamer

 

Intramedullary pressure values for 5 commercially available reaming systems were measured using 9.5-mm and 13-mm reamers at a constant speed. The system with the thinnest shaft was found to produce the lowest pressure values and vice versa. To avoid excessive pressures in the medullary canal, reamers should be kept sharp, have deep flutes, and have small shafts relative to the diameter of the reamer head. In addition, the reamer should be advanced slowly and multiple passes should be made with each reamer.Correct Answer: Shallow flutes

 

 

Which of the following factors is not a predictor of 1-year mortality following hip fracture in the elderly population:

 

1) American Society of Anesthesiologists rating of operative risk 3 or 4

3) Pre-injury dependency in basic activities of daily living

2) Development of 1 or more in-hospital postoperative complications

5) Patients older than 65 years of age

4) History of malignancy other than skin cancer

 

Aharonoff and Koval identified and prospectively evaluated 612 patients who were older than 65 years of age, previously ambulatory and home dwelling, and had femoral neck or intertrochanteric hip fractures of a nonpathologic origin. All patients received operative treatment and followed a similar postoperative protocol.

 

In multivariate analyses, the factors predictive of increased 1-year mortality were: Patients older than 85 years of age

 

 

 

 

Pre-injury dependency in basic activities of daily living A history of malignancy other than skin cancer American Society of Anesthesiologists rating of 3 or 4 Development of 1 or more postoperative complications

 

Preoperative comorbidities and prefracture living circumstances were not shown to be significantCorrect Answer: Patients older than 65 years of age

 

 

 

3172. (398) Q8-531:

During a posterolateral approach to the elbow for fixation of a proximal radius fracture, the posterior interosseous nerve (PIN) is safest with the patientâs elbow and forearm in what positions:

 

1) Extension and supination

3) Extension and neutral

2) Extension and pronation

5) Flexion and pronation

4) Flexion and supination

 

The posterolateral approach between the anconeus and extensor carpi ulnaris is frequently used during open reduction and internal fixation of proximal radius fractures.

This approach was performed in 32 cadaveric specimens to evaluate the position of the posterior interosseous nerve (PIN). The distance from the capitellum to the point where the PIN crossed the radial shaft was measured with the forearms in pronation and supination. Pronation allowed for safe exposure of the proximal 38 mm of the lateral aspect of the radius (average 52 mm).

Supination decreases this proximal safe zone to as little as 22 mm (average 33.4 mm).

The distance from the radiocapitellar joint to the origin of the PIN was found to average 1.2 mm, with 98% of nerves remaining completely within the supinator muscle.Correct Answer: Flexion and pronation

 

 

 

3173. (401) Q8-534:

Criteria for nonoperative management of an acetabular fracture includes all of the following except:

 

1) Roof arc measurement of greater than or equal to 45°

3) Stability demonstrated by dynamic stress radiographs

2) Unbroken computerized tomography arc at 10 mm from subchondral bone

5) Congruence of the femoral head with the unaffected acetabular roof on the anteroposterior and Judet views

4) Femoral head subluxation of 3 mm

 

The outcome of nonoperative treatment of acetabular fractures depends on the stability of the hip, the concentricity of the head of the femur under the roof of the acetabulum, and the condition of the roof itself. Nonoperative criteria include roof arc measurement of at least 45°, unbroken subchondral computerized tomography arc of 10 mm, stability of the joint on stress radiographs, and congruence off the femoral head with the unaffected acetabular roof on all three views, anteroposterior and both Judet radiographs. Any subluxation reduces the likelihood of a good result.Correct Answer: Femoral head subluxation of 3 mm

 

 

A patient presents to the emergency department with a segmental tibia fracture as a result of a gunshot injury with a 1-cm entrance wound. After appropriate irrigation and debridement, the wound measures 4 cm, there is no arterial injury, and the wound edges are easily approximated. Based on the Gustillo-Anderson classification, this wound is graded as:

 

1) Type I

3) Type IIIA

2) Type II

5) Type IIIC

4) Type IIIB

 

Open fractures must be graded after the first debridement. Preoperative classification of open fractures often underestimates the degree of bone and soft tissue compromise. The Gustillo-Anderson classification divides open fractures into 5 groups:

 

Type I fractures have an open wound less than 1 cm in length.

 

Type II wounds measure greater than 1 cm but less than 10 cm without contamination, and the wounds can be closed without flap coverage.

 

Type IIIA fractures have an open wound greater than 10 cm that can be closed with delayed primary techniques. Segmental fractures and gunshot injuries are also graded IIIA.

 

Type IIIB fractures require rotational of free flap wound coverage.

 

Type IIIC fractures are any open fractures with an associated vascular injury that requires repair.

Correct Answer: Type IIIA

 

 

3175. (405) Q8-538:

A 10-year-old boy sustains an extension-type posterolaterally displaced supracondylar humerus fracture. The nerve most commonly injured in this fracture pattern without a compartment syndrome is:

 

1) Musculocutaneous nerve

3) Ulnar nerve

2) Anterior interosseous branch of the median nerve

5) Posterior interosseous branch of the radial nerve

4) Superficial radial nerve

 

Supracondylar humeral fractures are the most common fracture about the elbow in children. Extension-type fractures account for 98% of humerus fractures. Complications include:

 

 

Cubitus varus Vascular injury

 

 

Compartment syndrome Nerve palsy

Injury to the anterior interosseous branch of the median nerve is most commonly associated with posterolateral displacement. Radial nerve injury is most commonly seen with posteromedial displacement.

Most nerve injuries recover spontaneously over 6 to 8 weeks after treatment and do not need to be acutely explored.Correct Answer: Anterior interosseous branch of the median nerve

 

 

A 33-year-old patient presents to the emergency department with a low-velocity gunshot wound through the knee. There is no significant soft tissue injury and no fracture noted by radiograph. Treatment should include:

 

1) Observation

3) Local wound debridement

2) Admission for intravenous (IV) antibiotics

5) Arthroscopic irrigation and debridement, and admission for IV antibiotics

4) Local wound debridement and IV antibiotics

 

Gunshot wounds are becoming increasingly more common. Thirty-three patients with low-velocity gunshot wounds through the knee, no significant soft tissue injury, and no fracture necessitating repair were reviewed. All patients were treated with arthroscopic irrigation and debridement.

 

Five chondral injuries and 14 meniscal injuries were found that were not seen on preoperative radiographs. Seventy-one percent of patients who had no radiographic evidence of debris, loose bodies, or bullet or bone fragments were found to have debris and meniscal damage.

 

Patients who sustain low-velocity gunshot wounds through the knee have soft tissue injuries not visible on plain radiographs in most cases. Operative management of these injuries is warranted.Correct Answer: Arthroscopic irrigation and debridement, and admission for IV antibiotics

 

 

 

3177. (408) Q8-541:

A patient presents with a complete ulnar collateral ligament rupture of the thumb. The structure responsible for a Stener lesion in this injury is the:

 

1) Extensor pollicis aponeurosis

3) Volar plate

2) Abductor pollicis brevis aponeurosis

5) Flexor pollicis longus aponeurosis

4) Adductor pollicis aponeurosis

 

In 1962, Stener described a unique anatomic derangement in 25 of 39 cases of complete rupture of the ulnar collateral ligament of the thumb. At the time of operation, he found the adductor pollicis aponeurosis interposed between the distally avulsed ligament and its insertion onto the base of the proximal phalanx. He concluded that without contact at the site of the rupture, ligament healing would be imperfect and result in a lax ulnar capsule regardless of the period of immobilization.Correct Answer: Adductor pollicis aponeurosis

 

 

 

3178. (409) Q8-542:

The main deforming force in a Bennett fracture-dislocation of the thumb is due to the pull of:

 

1) Abductor pollicis longus

3) Extensor pollicis longus

2) Abductor pollicis brevis

5) Flexor pollicis brevis

4) Extensor pollicis brevis

 

The mechanism of injury in a Bennett fracture-dislocation is an axial blow directed against the partially flexed first metacarpal. The fracture line characteristically separates the major part of the metacarpal from a small volar lip fragment. The volar lip fragment is anchored by the anterior oblique ligament and the remainder of the metacarpal is pulled dorsally and radially by the abductor pollicis longus. The distal attachment of the adductor pollicis levers the base further dorsally.Correct Answer: Abductor pollicis longus

 

 

The optimal level for a below knee amputation is:

 

1) 15 cm below the tibial tubercle

3) At the level of the tibial tubercle

2) 10 cm below the patella

5) 5 cm below the patella

4) 5 cm below the tibial tubercle

 

The preferred method is tibial transection at 12 cm to 15 cm distal to the tibial tubercle with a posterior myocutaneous flap. The metabolic cost of walking, as well as functional outcomes, are directly related to the length of the residual limb (with an adequate soft tissue envelope) and the number of usable joints preserved. The ability to use the posterior compartment muscles for coverage determines length of the stirrup.Correct Answer: 15 cm below the tibial tubercle

 

 

 

3180. (411) Q8-544:

A patient with a traumatic brain injury will heal a closed tibial diaphyseal fracture:

 

1) Slower than a normal patient

3) No differently than a normal patient

2) Faster than a normal patient

5) With less abundant callus formation than a normal patient

4) With more abundant callus formation than a normal patient

 

It has generally been believed that fractures heal faster and with âexuberant callusâ in traumatic brain injured patients. However, the results in large studies of both tibial and femoral fractures have contradicted this notion. Garland reviewed 47 tibial fractures in head trauma patients and failed to demonstrate either an early rate of union or an abnormally high union rate. Furthermore, exuberant callus was not observed roentgenographically. Tibia fractures in adults with head injuries appeared to heal in a fashion similar to fractures in normal patients.Correct Answer: No differently than a normal patient

 

 

 

3181. (412) Q8-545:

Contraindications to retrograde nailing of a femur fracture include which one of the following:

 

1) Skeletal immaturity

3) Morbid obesity

2) Pregnancy

5) Ipsilateral hip fracture

4) Ipsilateral tibia fracture

 

The recent clinical and experimental literature supports the use of retrograde femoral nailing. Revised techniques and newer implants appear to have minimized the risk of major complications while maintaining the perceived advantages of retrograde nailing.

 

Most investigators have defined specific indications and contraindications for retrograde nailing. Contraindications include skeletal immaturity and a history of knee joint sepsis. Indications include:

 

Multisystem injury

 

 

Trauma involving multiple extremity fractures Ipsilateral vascular injury

 

Fracture in the morbidly obese

 

 

Isolated fracture above a preexisting total knee prosthesis or below a hip prosthesis Ipsilateral hip/acetabular fracture

 

Spine injury or uncleared spine

A relative indication is pregnancy as it reduces the amount of radiation directly to the abdomen.Correct Answer: Skeletal immaturity

 

 

A patient with a closed, spiral diaphyseal humerus fracture is noted to have a radial nerve palsy. After the fracture is reduced and splinted in the emergency department, the radial nerve palsy persists. Treatment should be:

 

1) Remanipulation and cast application

3) Observation and surgical exploration if function does not return within 1 year

2) Immediate surgical exploration of the radial nerve

5) Immediate electromyogram followed by surgical exploration within 1 week of fracture

4) Observation and surgical exploration if function does not return within 3 to 4 months

 

Up to 18% of humeral shaft fractures have an associated radial nerve injury. Although the Holstein-Lewis fracture (oblique, distal one-third) is best known for its association with neurologic injury, radial nerve palsy is most commonly associated with middle third humeral fractures. Most nerve injuries are a neuropraxia or axonotmesis and 90% will resolve within 3 to 4 months. An electromyogram after 4 weeks is helpful in many cases.Correct Answer: Observation and surgical exploration if function does not return within 3 to 4 months

 

 

 

3183. (414) Q8-549:

A pure radiocarpal dislocation associated with a radial styloid fracture indicates injury through which of the following:

 

1) Scapholunate joint

3) Lesser arc

2) Greater arc

5) Space of Poirier

4) Inferior arc

 

The typical fall on an outstretched hand results in wrist dorsiflexion, ulnar deviation, and intercarpal supination. These forces enter the radiocarpal unit from the lateral or radial side then progress toward the ulnar, thus demonstrating a âtransverseâ instability pattern. The pathologic forces typically follow 1 of 3 paths known as the greater, lesser, and inferior arcs. The paths of the greater and lesser arc injuries are similar because in both injuries the lunate remains reduced in the lunate fossa of the radius. The distinction between the 2 injuries lies in the fact that the injury complex involves fracture through the scaphoid (greater arc) or rupture of the intercarpal ligaments (lesser arc).

 

Inferior arc injury occurs when the forces are transmitted from radial to ulnar at the radiocarpal level. Often, this injury is accompanied by fractures of the radial and ulnar styloids. The most severe injury in the inferior arc spectrum is radiocarpal dislocation.Correct Answer: Inferior arc

 

 

 

3184. (415) Q8-550:

Six weeks after open reduction and internal fixation of a talar neck fracture, an anteroposterior radiograph of the ankle reveals a lucency deep to the subchondral surface of the talar dome. This indicates:

 

1) Malreduction of the fracture

3) Collapse of the dome of the talus

2) Osteonecrosis of the talus

5) Revascularization of the talus

4) Associated tibial plafond impaction

 

The talus is composed of 7 articular surfaces covering 60% of the bony surface. Vascular access to the bone is limited to the nonarticular areas. An anastomotic sling of vessels provides the blood supply to the body of the talus that include: laterally, the artery of the tarsal sinus; medially, the artery of the tarsal canal, and additional arteries that enter dorsally through the neck and on the medial surface of the body.

 

The likelihood of talar body osteonecrosis increases with the severity of the injury. The diagnosis of osteonecrosis is routinely made radiographically by the absence of a Hawkin sign. This lucency deep to the subchondral surface of the dome of the talar dome on an anteroposterior radiograph of the ankle obtained 6 to 8 weeks after injury is an indication of revascularization.Correct Answer: Revascularization of the talus

 

 

The risk of developing avascular necrosis of the femoral head after a Pipkin type II hip fracture-dislocation is:

 

1) Equal to a simple dislocation

3) Lower than a simple dislocation

2) Higher than a simple dislocation

5) Equal to a patient with an associated femoral neck fracture

4) Equal to a patient with an acetabulum fracture without a femoral head fracture

 

Approximately 7% of posterior dislocations of the hip have associated fractures of the femoral head or neck. Pipkin categorized these into 4 types:

 

 

 

Type I â Fracture of the femoral head caudad to the fovea Type II â Fracture of the femoral head cephalad to the fovea Type III â Type I or II associated with a femoral neck fracture

 

Type IV â Type I, II, or III associated with a fracture of the acetabulum

Patients with these types of fracture-dislocations are at a high risk for developing osteonecrosis and post-traumatic degenerative arthritis. The prognosis for these injuries varies. Pipkin types I and II have the same prognosis as a simple dislocation. Pipkin type IV injuries roughly have the same prognosis as acetabular fractures without femoral head fracture. Pipkin type III injuries have a poor prognosis with a 50% rate of post-traumatic osteonecrosis.Correct Answer: Equal to a simple dislocation

 

 

 

3186. (417) Q8-553:

A 35-year-old construction worker who presents after a fall is found to have a fracture involving 65% of the coronoid process and an anatomically reduced elbow joint. Treatment should include:

 

1) Immediate range of motion

3) Long arm cast immobilization

2) Excision of the fragment

5) Static external fixation

4) Open reduction and internal fixation

 

Fractures of the coronoid process of the ulna are commonly seen in approximately 10% of elbow dislocations. Regan and Morrey divided these injuries into 3 types:

 

Type 1 â Avulsion of the tip of the coronoid process

 

 

Type 2 â A fracture less than 50% of the coronoid process Type 3 â A fracture more than 50% of the coronoid process

Closed treatment and early range of motion is recommended for types 1 and 2 fractures because these fractures do not result in anteroposterior elbow instability. However, type 3 fractures have a poor outcome with immobilization. Reduction and fixation is important because the base of the coronoid also provides attachment for the insertion of the medial collateral ligament and part of the anterior capsule, as well as providing a physical block to posterior subluxation of the proximal ulna.Correct Answer: Open reduction and internal fixation

 

 

Malunion resulting in loss of radial bow after open reduction and internal fixation of a diaphyseal radius and ulna fracture will most often result in:

 

1) Loss of elbow motion

3) Early wrist arthritis

2) Loss of grip strength

5) No significant loss of function

4) Early elbow arthritis

 

Diaphyseal fractures of the radius and ulna present specific problems not encountered with fractures of the shafts of other long bones. The recovery of function after fracture of the forearm depends on the return of rotation of the forearm, the maintenance of a functional range of motion of the elbow and wrist, and recovery of grip strength. Malreduction and malalignment can result in poor function, particularly with loss of forearm rotation and grip strength.

 

Schemitsch and Richards evaluated 55 patients with both bone forearm fractures treated with plating at an average of 6 years after injury. Ninety-eight percent of the fractures had healed. A good functional result (more than 80% of normal rotation of the forearm) was associated with restoration of the normal amount and location of the radial. Similarly, the recovery of grip strength was associated with restoration of the location of the radial bow toward normal.Correct Answer: Loss of grip strength

 

 

 

3188. (419) Q8-556:

Injury Severity Score (ISS) is defined as:

 

1) The square of the Glascow Coma Scale

3) The sum of the squares of AIS grades for all body regions

2) The sum of the highest Abbreviated Injury Scale (AIS) grades in the 3 most severely injured body regions

5) The sum of the squares of the highest AIS grades in the 3 most severely injured body regions

4) The sum of the squares of the AIS grades for spine, upper extremity and lower extremity

 

The Injury Severity Score (ISS) was designed to summarize the severity of multiple injuries. The ISS is the sum of the squares of the highest Abbreviated Injury Scale (AIS) grades in the 3 most severely injured body regions. The 6 body regions of injuries used in ISS are head/neck, face, chest, abdominal or pelvic contents, extremities or pelvic girdle, and external.

 

The score ranges from 1 to 75. A patient with an AIS of 6 in any region is assigned an ISS of 75. Generally, multiple trauma patients are defined as patients with an ISS greater than or equal to 18. An ISS less than 30 is usually indicative of a good prognosis, unless associated with a severe head injury. An ISS more than 60 is usually fatal. The accuracy of severity scoring decreases in the geriatric population.Correct Answer: The sum of the squares of the highest AIS grades in the 3 most severely injured body regions

 

 

 

3189. (420) Q8-557:

When treating a tibial shaft fracture with a unilateral external fixator, all of the following increase the stiffness of the construct except:

 

1) Increasing pin diameter

3) Increasing pin number

2) Wide pin spread within a segment

5) Double-stacking connecting bars

4) Increasing distance between the connecting bar and the bone

 

The stiffness of external fixator constructs has been widely studied and depends on multiple factors. Each variable has an incremental contribution to overall stiffness. For a unilateral frame, the key features that determine the performance of the frame are:

 

 

 

 

Pin diameter Pin number Pin spread

 

Number of connecting bars

 

Distance from the connecting bars to the loading axis

When pin diameters are too small, local stresses can lead to micromotion and ultimate pin failure. Multiple pins with a wide pin spread in each fracture segment will substantially increase stiffness. Double-stacking connecting bars as well as a decrease in the distance from the connecting bar to the bone have also been shown to have significant effects on construct stiffnessCorrect Answer: Increasing distance between the connecting bar and the bone

 

 

3190. (421) Q8-558:

A diminished risk for cutout failure of a screw in the femoral head can be predicted by a tip-apex distance less than 25 mm. This distance is determined by:

 

1) The distance from the tip of the greater trochanter to the apex of the femoral head

3) The distance from the tip of the implant to the apex of the femoral head on the lateral radiograph

2) The distance from the tip of the implant to the apex of the femoral head on the anteroposterior radiograph

5) The sum of the distances from the tip of the implant to the apex of the femoral head on both the anteroposterior and lateral radiographs

4) The distance from the tip of the trochanter to the apex of the implant

 

The location of a screw in the femoral head historically has been believed to be an important variable in the strength of the bone-implant construct. A simple way to define screw position, the tip-apex distance has been shown to be a strong predictor of failure by cutout. The distance from the tip of the implant to the apex of the femoral head is determined on the anteroposterior view and added to the value of a similar measurement obtained from the lateral view to generate the tip-apex distance. The risk of failure approaches 0 with values less than 25 mm but increases rapidly as the screw becomes more peripheral or shallowly placed.

Intraoperative estimation of screw position using this technique has been shown to reduce cutout failures.Correct Answer: The sum of the distances from the tip of the implant to the apex of the femoral head on both the anteroposterior and lateral radiographs

 

 

 

3191. (422) Q8-560:

Displaced scapular body fractures are most often associated with injury to:

 

1) Ipsilateral lung

3) Myocardium

2) Contralateral humerus

5) Cervical spine

4) Trachea

 

Fractures of the scapula are relatively uncommon injuries, representing 3% to 5% of shoulder fractures. The scapula is a relatively mobile structure and is well protected by a significant muscle mass.

Thompson reviewed 56 patients with 58 scapular fractures secondary to blunt trauma. The patients averaged 3.9 major injuries excluding their scapular fractures. The injury pattern associated with blunt scapular fracture is unique. Patients with scapular fracture have a high incidence of injury to the ipsilateral lung and chest wall and to the ipsilateral shoulder girdle and its contained structures. Scapular fractures provide the trauma surgeon with a reliable clinical clue that the patient is at inordinate risk to have associated injuries of major consequence to the ipsilateral lung and chest wall; the ipsilateral shoulder girdle; and the ipsilateral subclavian, axillary or brachial artery, and brachial plexus.Correct Answer: Ipsilateral lung

 

 

 

3192. (423) Q8-561:

A Tillaux fracture of the distal tibia is the result of what mechanism of injury:

 

1) Supination

3) Dorsiflexion

2) Pronation

5) Internal rotation

4) External rotation

 

Tillaux originally described a special fracture occurring in older adolescents. The mechanism of injury is an external rotational force with stress placed on the anterior tibiofibular ligament, causing avulsion of the distal tibial physis anterolaterally. This occurs after the medial part of the physis has closed but before the lateral part closes. The resultant fracture through the physis runs across the epiphysis and distally into the joint, creating a Salter-Harris type III or IV fracture. Open reduction and internal fixation are indicated if the fracture is displaced. If left untreated, nonunions may result.Correct Answer: External rotation

 

 

Inability to achieve a closed reduction of a lateral subtalar joint dislocation is most commonly due to interposition of what structure:

 

1) Tibialis anterior tendon

3) Deltoid ligament

2) Tibialis posterior tendon

5) Extensor digitorum brevis

4) Extensor retinaculum

 

Dislocations of the subtalar complex involve disruption of both the talocalcaneal and talonavicular articulations. The clinical deformity is usually apparent, but an anteroposterior view of the foot is helpful to show the position of the talonavicular joint and evidence of impaction before attempted reduction. In closed dislocations, timely reduction is important to minimize vascular compromise to the overlying soft tissues. Patient sedation, knee flexion, and firm longitudinal traction are required for successful closed reduction.

 

With lateral subtalar dislocations, the most common obstruction to closed reduction is the interposed posterior tibial tendon. In medial subtalar dislocations, the head of the talus may become trapped by the capsule of the talonavicular joint, the transverse fibers of the cruciate-crural ligament, or the extensor digitorum brevis muscle.Correct Answer: Tibialis posterior tendon

 

 

 

3194. (425) Q8-563:

A 27-year-old sustains a closed long finger proximal interphalangeal (PIP) joint fracture-dislocation. The fracture is intra-articular and involves 30% of the dorsal articular surface of the middle phalanx. Treatment should consist of:

 

1) Open reduction and internal fixation

3) Closed reduction and casting

2) Closed reduction and percutaneous pinning

5) Static external fixation

4) Primary arthrodesis

 

Dorsal dislocations can be classified into 3 types:

 

Type I (hyperextension) dislocations are characterized by avulsion of the volar plate from the middle phalanx and a minor longitudinal split in the collateral ligaments. The articular surfaces remain intact.

 

Type II (dorsal dislocation) results in an avulsion of the volar plate accompanied by a major bilateral split in the collateral ligament system. There is no contact between the articular surfaces.

 

Type III (fracture-dislocation) is divided into stable and unstable.

In the stable pattern, there is a fracture through the base of the middle phalanx involving less than 60% of the articular arc. The dorsal portion of the collateral ligaments remain intact to the middle phalanx which renders these injuries inherently stable and able to be treated nonoperatively.

 

Unstable fracture-dislocations involve more than 60% of the articular surface. Because the majority of the collateral ligament-volar plate complex inserts on the volar 40% of the articular surface, this results in instability that necessitates operative treatment.Correct Answer: Closed reduction and casting

 

 

An acute anterior sternoclavicular joint dislocation is closed-reduced in the emergency department, however, as the arm is brought back to the patientâs side, the joint redislocates. Treatment should consist of:

 

1) Repeat attempts until a stable reduction is achieved

3) Open reduction and internal fixation

2) Closed reduction and percutaneous pinning

5) Medial clavicle excision

4) Immobilization with a sling for 6 weeks

 

The sternoclavicular (SC) joint is injured infrequently due to its medial location. However, indirect forces transmitted from the shoulder can result in sprains, fractures, and dislocations. The vast majority of SC joint dislocations occur anteriorly, and most occur as the result of motor vehicle accidents or athletic injuries.

 

Open reduction or percutaneous pinning is rarely indicated for anterior dislocations and most techniques have been associated with an unacceptably high incidence of redisplacement and complication. Closed reductions are often unsuccessful with immediate redisplacement of the medial clavicle. If the medial clavicle remains unstable and redislocates, a sling is recommended for definitive treatment to support the arm until symptoms resolve. Most patients with chronic SC joint dislocations remain asymptomatic with little functional limitationCorrect Answer: Immobilization with a sling for 6 weeks

 

 

 

3196. (427) Q8-565:

At the time of open reduction and internal fixation of a depressed medial tibial plateau fracture, a peripheral medial meniscus tear is identified. Treatment should consist of:

 

1) Total menisectomy

3) Delayed arthroscopic debridement of the meniscus after fracture union

2) Immediate repair of the meniscus

5) Meniscal replacement

4) Partial menisectomy

 

Tibial plateau fractures encompass a broad spectrum of both osseous and soft tissue injury. Anatomic restoration and maintenance of both joint alignment and surface congruity has been shown to yield the best results. Approximately 50% of plateau fractures have been found to have meniscal injuries. Most of the tears occur in the vascular region of the posterior half making them amenable to repair, and the tears always occur on the side with the fractured condyle. The significance of preserving the meniscus cannot be overstated. Higher rates of post-traumatic arthritis after only 70 months were reported by Lachiewicz in those patients who underwent menisectomy at the time of their open reduction compared to those with preserved menisci.Correct Answer: Immediate repair of the meniscus

 

 

 

3197. (428) Q8-566:

Which of the following glenohumeral dislocations is associated with the highest incidence of vascular compromise?

 

1) Anterior

3) Inferior

2) Posterior

5) Anterosuperior

4) Superior

 

Inferior glenohumeral dislocations, or luxatio erecta, may be produced by a hyperabduction force that causes impingement of the neck of the humerus against the acromion process which levers the head out inferiorly. Severe soft tissue injury or fractures about the proximal humerus occur with this dislocation.

 

Mallon et al reviewed 80 cases of luxatio erecta and also discussed 6 additional cases that they treated. The literature shows that either a fracture of the greater tuberosity or a rotator cuff tear was associated with this injury in 80% of patients; 60% of the patients reviewed sustained some degree of neurologic compromise, most commonly to the axillary nerve. These injuries usually resolved and the time for recovery varied from 2 weeks to 1 year. Only 3.3% of the cases demonstrated significant vascular compromise, but this is the highest incidence for any shoulder dislocation. Doppler studies of the affected arm or observation of the patient overnight are recommended because of the potentially disastrous complications of vascular insufficiency. If there is any indication of a vascular problem, immediate arteriogram is indicated.Correct Answer: Inferior

 

 

The length of hospital stay for an elderly patient with isolated pubic rami fractures sustained after a fall is dependent upon which of the following pre-injury criteria:

 

1) Requirement of a cane or walker for ambulation

3) Nursing home residency

2) Dependence in activities of daily living

5) History of falls

4) Coronary artery disease

 

Pelvic fractures resulting from low-energy mechanisms are usually fractures of the individual bones of the pelvic ring that do not damage the true integrity of the ring structure. This class includes domestic falls frequently seen in the elderly population, and avulsion injuries seen in young patients.

 

Koval et al reviewed a series of elderly patients with pubic rami fractures. Sixty out of 63 patients (95%) required hospitalization for pain control and progressive mobilization. The hospital length of stay for the 60 admitted patients averaged 14 days; patients who had 3 or more associated medical comorbidities or required use of a cane or walker for ambulation prior to fracture were more likely to have been hospitalized >2 weeks.Correct Answer: Requirement of a cane or walker for ambulation

 

 

 

3199. (430) Q8-568:

Which of the following criteria is not included in the Mangled Extremity Severity Score (MESS)?

 

1) Skeletal/soft tissue injury

3) Fracture pattern

2) Limb ischemia

5) Age

4) Shock

 

Timely amputation is indicated for a severely injured limb that, after reconstruction, would be less functional than a prosthesis. The Mangled Extremity Severity Score (MESS) has been used to predict necessity of amputation after lower-extremity trauma. Points are assigned for skeletal/soft tissue injury (1-4), ischemia time (1-6), age of the patient (0-2), and shock defined by hypotension (0-2).

 

In both prospective and retrospective studies, a MESS score equal or greater than or equal to 7 has had a 100% predictable value for amputation. This relatively simple, readily available scoring system of objective criteria was highly accurate in several studies in acutely discriminating between limbs that were salvageable and those that were unsalvageable and better managed by primary amputation. However, further work has demonstrated low specificity and sensitivity for amputation in multicenter trials. For this reason, any of the scoring systems should be used only as a guideline.Correct Answer: Fracture pattern

 

 

 

3200. (431) Q8-569:

A 25-year-old nonsmoking laborer presents with a closed, highly comminuted, displaced tibial plafond fracture and no other injuries. This is best managed with:

 

1) Immediate open reduction and internal fixation with lag screws

3) External fixation spanning the ankle and delayed internal fixation

2) Immediate open reduction and internal fixation with a buttress plate

5) Intramedullary nail

4) Closed reduction and casting

 

Pilon fractures are most commonly the result of an axial loading injury. Complications have been reported to be as high as 40% and include wound slough, infection, malunion, and post-traumatic arthritis. The safest approach is to delay definitive surgery until the soft tissue envelope is capable of tolerating an incision, demonstrated by resolution of blisters or the absence of tense, shiny skin. During the wait, the limb should be maintained in correct alignment and length, elevated, and protected. A temporary external fixator from the tibia to the hindfoot permits approximate reduction by ligamentotaxis, provides stability, and allows the use of foot pumps which can reduce swelling.Correct Answer: External fixation spanning the ankle and delayed internal fixation

 

 

After sustaining a twisting injury, a lateral ankle radiograph reveals a fibular fracture line from anteroinferior to posterosuperior. This injury can be classified as:

 

1) Supination external rotation

3) Supination adduction

2) Pronation external rotation

5) Supination internal rotation

4) Pronation abduction

 

The 2 most widely used ankle fracture classification systems are the Danis-Weber system, which is based on the level of the fibular fracture, and the Lauge-Hansen system, which is based on experimentally created injury mechanisms.

Lauge-Hansen emphasized the influence that the position of the foot had on the injury pattern and correlated this position with the direction of the deforming forces. The pattern of the fibula fracture can be used to determine the type of fracture:

 

 

 

 

Supination adduction results in a transverse fracture at or below the level of the joint Supination external rotation results in a spiral fracture from anteroinferior to posterosuperior Pronation external rotation results in a spiral fracture from posteroinferior to anterosuperior Pronation abduction results in a transverse fracture at or above the level of the joint

 

Correct Answer: Supination external rotation

 

 

3202. (433) Q8-571:

To confirm a suspected complete Achilles tendon rupture, a Thompson test should be done with the patient in what position:

 

1) Standing on the contralateral leg

3) Supine with feet extended over the table

2) Sitting with knees at 90° of flexion

5) Lateral decubitus with affected side down

4) Prone with feet extended over the table

 

Achilles tendon ruptures occur most commonly in healthy men in their third to fifth decade of life who were participating in recreational sports activities at the time of the injury. In acute cases, physical examination will reveal a palpable depression over the area of the tendon rupture, weakness of plantar flexion, and positive results on Thompsonâs test. Thompsonâs test is performed with the patient prone on a table with the feet extended over the end of the table. The calf muscles are then squeezed between the examinerâs thumb and forefingers in the place of widest girth. A positive test occurs when there is no plantar movement of the foot, indicating a complete rupture of the heel cord.Correct Answer: Prone with feet extended over the table

 

 

 

3203. (434) Q8-572:

When comparing a nonbridging external fixator to external fixation spanning the wrist for treatment of unstable distal radius fractures, a bridging frame has been shown to:

 

1) Diminish finger range of motion

3) Improve wrist range of motion

2) Improve grip strength

5) Maintain carpal alignment

4) Maintain volar tilt

 

McQueen performed a randomized, prospective study on 60 patients with unstable fractures of the distal radius to compare bridging with nonbridging external fixation using pins placed in the distal fragment of the radius. The radiological results showed significant improvement in the nonbridging group at all stages of review. In particular, normal volar tilt and carpal alignment were regained and maintained.

 

The functional results at 6 weeks, 3 months, 6 months, and 1 year showed statistically better grip strength and flexion in the nonbridging group at all stages of review. Other ranges of movement showed an early advantage in the nonbridging group. Nonbridging external fixation is the treatment of choice for unstable fractures of the distal radius that have sufficient space for the placement of pins in the distal fragment.Correct Answer: Diminish finger range of motion

 

 

Early fixation devices for femur fractures in skeletally immature patients included all of the following except:

 

1) The Lane plate

3) Compression plates

2) Beef bone plate

5) External fixation

4) Intramedullary plugs made of ivory or bone

 

Early fixation devices for femur fractures in skeletally immature patients included all of the following except compression plates. The advent of the compression plates took place in the 1960s, heralded by the ankle orthosis group in Europe.Correct Answer: Compression plates

 

 

3205. (616) Q8-854:

An "immediate" hip spica cast is applied

 

1) On admission

3) Within the first 8 hours of injury

2) Within the first 4 hours of injury

5) Within the first 24 hours of injury

4) Within the first 12 hours of injury

 

An "immediate" hip spica cast is applied within the first 24 hours of injury.Correct Answer: Within the first 24 hours of injury

 

 

3206. (617) Q8-855:

Apex posterior malalignment of a femur fracture is poorly tolerated because:

 

1) The bone has less remodelling capacity in the anterior / posterior plane.

3) The risk of overgrowth is greater in patients with an apex posterior malalignment than other angular malalignments.

2) Apex posterior malalignment creates hyperextension in the knee at heel strike, which creates pain with ambulation.

4) Apex posterior malalignment following a femoral shaft fracture can create a traction injury to the sciatic nerve.

 

Apex posterior malalignment of a femur fracture is poorly tolerated because an apex posterior malalignment creates hyperextention in the knee at heel strike which creates pain with ambulation. This can produce pain with ambulation. The pain can be so severe that it creates dysfunction in the lower extremity.Correct Answer: Apex posterior malalignment creates hyperextension in the knee at heel strike, which creates pain with ambulation.

 

 

3207. (618) Q8-856:

When do children have the greater capacity for overgrowth following a femoral shaft fracture?

 

1) Birth to 2 years of age

3) Six to 9 years of age

2) Two to 5 years of age

5) Between 9 years of age and closure of the distal femoral physis

4) Nine to 12 years of age

 

Children have the greatest capacity for overgrowth following a femoral shaft fracture between 6 and 9 years of age. Because of this, more shortening can be accepted in a femur fracture in this age range than at other times.Correct Answer: Six to 9 years of age

 

An "early" spica cast is applied:

 

1) Within the first 24 hours

3) In the second week after injury

2) Between the first and seventh day after admission

4) In the third week after injury

 

An "early" spica cast is applied between one and seven days after a child sustains a diaphyseal femur fracture. An "early" spica cast is normally applied to a younger child who is treated with split Russell's traction prior to cast application. Children older than 6 years of age are treated with 90/90 skeletal traction until a cast can be applied. These children often have to remain hospitalized in traction until the second week after injury.Correct Answer: Between the first and seventh day after admission

 

 

3209. (620) Q8-858:

One of the most common complications following the use of an external fixation device for the treatment of diaphyseal femur fracture is:

 

1) Shortening

3) Malrotation

2) Angulation

4) Refracture

 

One of the most common complications following the use of external fixation devices for the treatment of a pediatric diaphyseal femur fracture is malrotation. The short external rotators outwardly turn the proximal femoral fragment, creating an internal rotation deformity at the fracture site. If the pins are applied parallel to one another and the floor at the time the frame is constructed, the child will have an internal rotation malalignment at union. To prevent this from occurring, the surgeon must direct the proximal pin cluster from slight posterior to slight anterior, compensating for the effects of the short external rotators on the proximal fragment.Correct Answer: Malrotation

 

 

3210. (621) Q8-859:

The middle layer of lateral extra-articular stabilizing structures of the knee include all of the following except:

 

1) Fibular collateral ligament

3) Arcuate ligaments

2) Fabello-fibular ligament

4) Lateral capsule of the knee

 

The lateral capsule of the knee is the deep stabilizing layer eventing a varus stress of the knee.Correct Answer: Lateral capsule of the knee

 

 

 

 

 

Figure 2A

A 45-year-old, otherwise healthy female, was admitted to the trauma service after a snowmobile accident. Examination showed that she had pain in and deformity of her left thigh, with an open wound over her posterolateral buttock. A x-ray of her femur is shown in Slide 2A. What should be done next?

 

1) Immediate surgery.

3) Computed tomography scan.

2) Radiographs of spine and pelvis.

4) Magnetic resonance imaging.

 

The next thing that should be done is a screening view of the pelvis, which in this case showed an associated anterior column acetabular fracture. Radiographs that show the femoral neck well must also be obtained in order to rule out associated femoral neck fracture.Correct Answer: Radiographs of spine and pelvis.

 

 

3212. (623) Q8-864:

 

 

 

Figure 2B Figure 2C

A 45-year-old, otherwise healthy female, was admitted to the trauma service after a snowmobile accident. Examination showed that she had pain in and deformity of her left thigh, with an open wound over her posterolateral buttock. Following evaluation by the trauma team, the patient was stable. The next step in the management of her fracture should be:

 

1) Immediate surgery to debride her wound, with placement of skeletal traction and delayed nailing of the femur.

3) Immediate surgery to debride the wound and nailing of the femur fracture.

2) Immedicate surgery to debride the wound and external fixation of the femur fracture.

4) Immediate surgery to debride the wound and repair of the femur fracture and acetabular fracture.

 

Open fractures must always be operated on immediately with aggressive irrigation and debridement. Once this is done, immediate internal fixation is appropriate. In this case, treatment included placement of tobramycin beads in the wound, with delayed closure. Her acetabular fracture was repaired by an anterior approach several days later. Anteroposterior (AP) and lateral radiographs of the femur are shown in Slides 2B and 2C.Correct Answer: Immediate surgery to debride the wound and nailing of the femur fracture.

 

 

 

 

 

Figure 2A Figure 2D Figure 2E

A 45-year-old, otherwise healthy female, was admitted to the trauma service after a snowmobile accident. Examination showed that she had pain in and deformity of her left thigh, with an open wound over her posterolateral buttock. A x-ray of her femur is shown in Slide 2A. The patient's femur healed, but she complained of external rotation of her left foot. Her radiograph is shown in Slide 2D. The likely explanation of this is:

 

1) Malreduction of her acetabular fracture.

3) Heterotopic ossification.

2) Contracture of the external rotator muscles around her hip.

5) Contracture of the abductor muscles.

  1. Malunion of her femoral shaft fracture.

     

    Rotational deformity following femoral nailing should always be checked by an examination of the relative arcs of hip rotation following surgery. In this patient, a computed tomography (CT) scan of both legs was done Slide 2E, showing a 55° external rotation deformity of the proximal femur.Correct Answer: Malunion of her femoral shaft fracture.

     

     

    3214. (625) Q8-866:

     

     

     

    Figure 2A Figure 2F

    A 45-year-old, otherwise healthy female, was admitted to the trauma service after a snowmobile accident. Examination showed that she had pain in and deformity of her left thigh, with an open wound over her posterolateral buttock. A x-ray of her femur is shown in Slide 2A. The incidence of rotational malunion after femoral nailing is:

     

    1) 2%

    3) 12%

    2) 7%

  2. 25%

4) 20%

 

Rotational deformity following femoral nailing occurs in about 7% of cases. Because of continued complaints of gait disturbance and discomfort, the patient underwent rotational osteotomy and repeat nailing (Slide 2F).Correct Answer: 7%

 

 

 

 

 

 

Figure 3A Figure 3B Figure 3C Figure 3D

A 32-year-old female with schizophrenia was found lying below a broken sixth floor window early one morning, by construction workers. She had severe open wounds above both knees and obvious fractures. She was conscious and did not have any neck or back pain. On arrival at the hospital, she was hypothermic but otherwise stable. Radiographs of her legs are shown in Slides 3A, 3B, 3C, and 3D. The patient was found to have no other injuries. The next step in her management is:

 

  1. Observation in the Intensive Care Unit for arrhythmias.

3) Immediate debridement and stabilization

2) Application of skeletal traction and antibiotic therapy

5) Delayed internal fixation.

4) Immediate internal fixation with bone grafting.

 

As for all open fractures, immediate debridement and stabilization are mandatory. The need for bone grafting is much less when careful indirect reduction techniques are utilized, and when necessary, is delayed until the soft tissue wounds have stabilized.Correct Answer: Immediate debridement and stabilization

 

 

 

 

 

 

Figure 3A Figure 3B Figure 3C Figure 3D

 

 

 

Figure 3E Figure 3F

A 32-year-old female with schizophrenia was found lying below a broken sixth floor window early one morning, by construction workers. She had severe open wounds above both knees and obvious fractures. She was conscious and did not have any neck or back pain. On arrival at the hospital, she was hypothermic but otherwise stable. Radiographs of her legs are shown in Slides 3A, 3B, 3C, and 3D. The patient was found to have no other injuries. What is the best option for internal fixation?

 

1) Cannulated lag screws.

3) Retrograde intramedullary nail.

2) Antegrade intramedullary nail.

5) Condylar buttress plate.

4) Dynamic condylar screw.

 

Dynamic condylar screw fixation is the best way to avoid rotational deformity. Rotational deformity following femoral nailing should always be checked by an examination of the relative arcs of hip rotation following surgery. In this patient, a computed tomography (CT) scan of both legs was done (Slide 2E), showing a 55° external rotation deformity of the proximal femur.Correct Answer: Dynamic condylar screw.

 

 

 

 

 

 

Figure 3A Figure 3B Figure 3C Figure 3D

A 32-year-old female with schizophrenia was found lying below a broken sixth floor window early one morning, by construction workers. She had severe open wounds above both knees and obvious fractures. She was conscious and did not have any neck or back pain. On arrival at the hospital, she was hypothermic but otherwise stable. Radiographs of her legs are shown in Slides 3A, 3B, 3C, and 3D. If the leg had valgus instability immediately after fixation, what is the likely cause?

 

1) Midclavicular injury.

3) Unstable fixation.

2) Occult tibial plateau fracture.

5) Infection

  1. Inadequate external fixation.

     

    In cases of severe metaphyseal comminution, varus/valgus instability is usually due to inadequate fixation. Stability should always be checked during surgery. If after lateral plate fixation, instability is present, the medial external fixator or a separate medial plate placed through a limited medial incision may be done.Correct Answer: Unstable fixation.

     

     

    3218. (629) Q8-870:

     

     

     

    Figure 1A Figure 1B

    A 29-year-old, otherwise healthy male, was seen in the emergency room after a motorcycle accident. Examination showed that his left leg was abnormally shortened and internally rotated. There were no neurologic or vascular deficits. An anteroposterior xray of his pelvis is shown in Slide 1A. An attempt at closed reduction was performed 4 hours after his injury. The post-reduction x-ray is shown in Slide 1B. This patient's risk of avascular necrosis and degenerative arthritis, respectively, are:

     

    1) 15%, 30%

    3) 50%, 75%

    2) 15%, 75%

  2. 25%,50%

4) 80%, 30%

 

Avascular necrosis results in 15%, and is usually evident within the first year (but may appear up to 3 years after injury). The risk of avascular necrosis depends on the time to reduction. Reduction after 8 hours from injury markedly increases the risk of avascular necrosis. Degenerative arthritis may be the result in up to 75% of patients at long-term follow-up.Correct Answer: 15%, 75%

 

 

 

 

 

 

Figure 1A

A 29-year-old, otherwise healthy male, was seen in the emergency room after a motorcycle accident. Examination showed that his left leg was abnormally shortened and internally rotated. There were no neurologic or vascular deficits. An anteroposterior xray of his pelvis is shown in Slide 1A. Following the closed reduction, the next step in management should be:

 

  1. Observation

3) CT

2) Bone scan

5) Chart x-ray

4) MRI

 

CT scans are recommended to determine if there are any osteochondral fragments within the joint, any evidence of posterior wall fracture, or any femoral head fracture. If intra-articular debris is noted, or if the reduction is not congruent, then arthrotomy and irrigation of the joint is necessary.Correct Answer: CT

 

 

3220. (631) Q8-872:

 

 

 

Figure 1A Figure 1C

A 29-year-old, otherwise healthy male, was seen in the emergency room after a motorcycle accident. Examination showed that his left leg was abnormally shortened and internally rotated. There were no neurologic or vascular deficits. An anteroposterior xray of his pelvis is shown in Slide 1A. This patient's CT scan is shown in Slide 1C. The patient should be managed with:

 

1) Activities as tolerated

3) Protected weight bearing with crutches for 1 year

2) Protected weight bearing with crutches for 6 weeks

5) Protected weight bearing with crutches for 6 months.

4) Surgery

 

Following uncomplicated reduction, the patient is treated with protected weight-bearing for 4 to 6 weeks. Operative intervention is necessary if congruent closed reduction cannot be obtained, if an associated posterior wall fracture greater than 25% of the wall is identified, or if osteochondral debris is present within the joint space.Correct Answer: Protected weight bearing with crutches for 6 weeks

 

A 41-year-old, otherwise healthy male, was seen in the emergency room after falling off his bicycle. Examination showed that he was unwilling to bear weight on his left leg, and had pain with any attempted movement of his hip. He preferred to keep his leg externally rotated. There were no neurologic or vascular deficits. The next step in his evaluation should be:

 

1) Observation and crutches for pain

3) Technetium bone scan

2) Radiographs

4) MRI

 

Plain anteroposterior and lateral radiographs are indicated as the primary imaging method to evaluate suspected hip trauma. Although femoral neck fractures are less common in this age group, they must be considered in anyone with groin, thigh, or knee pain. Observation alone may be considered in a patient who is able to weight-bear comfortably and tolerates passive internal and external rotation without pain. Bone scans are generally ordered to evaluate the perfusion of the femoral head in a patient known to have a fracture, and have also been recommended for the evaluation of occult femoral neck stress fractures. However, MRI has proven superior for both of these indications. Plain radiographs in two planes are required for the initial diagnosis of hip trauma, and can differentiate from dislocation, acetabular fracture, femoral neck fracture, or pelvic ring trauma, or other lesions (infectious, neoplastic).Correct Answer: Radiographs

 

 

 

3222. (633) Q8-874:

 

 

 

Figure 2A Figure 2B

Anteroposterior and lateral radiographs are shown in Slides 2A and 2B. This patient should be managed with:

 

1) Observation and crutches for pain

3) Internal fixation

2) Skeletal traction

4) Arthroplasty

 

Femoral neck fractures always require internal fixation, except in the most debilitated of patients. Anatomic reduction is the prerequisite to internal fixation, and requires open reduction if closed manipulation is not successful. Once reduced, multiple (three or four) cannulated screws are now considered the method of choice for subcapital and transcervical fractures. Basilar neck fractures are usually treated with a sliding hip screw and additional derotation screw.Correct Answer: Internal fixation

 

 

 

 

 

Figure 2C Figure 2D

Postoperative radiographs are shown in Slides 2C and 2D. This patient risk of avascular necrosis is:

 

1) 5%

3) 50%

2) 20%

4) 80%

 

Avascular necrosis complicates approximately 20% of displaced femoral neck fractures. In patients considered to be at risk, MRI can be performed early to confirm the diagnosis. Protected weight-bearing is then recommended in order to lessen the risk of femoral head collapse.Correct Answer: 20%

 

 

3224. (635) Q8-876:

 

 

 

Figure 3A Figure 3B

An 85-year-old female was admitted after a presumed fall at her nursing home. Examination showed that the patient was demented, with pain in her right leg, which was shortened and externally rotated. Radiographs of the patients hip are shown in Slides 3A and 3B. The patients fracture is classified as:

 

1) Type I

3) Type III

2) Type II

4) Type IV

 

The anteroposterior and lateral radiographs show an unstable fracture in which the posteromedial buttress, containing the lesser trochanter, has fractured. This represents the Type III fracture of Kyle and Gustilo.Correct Answer: Type III

 

 

 

 

 

Figure 3A Figure 3B

An 85-year-old female was admitted after a presumed fall at her nursing home. Examination showed that the patient was demented, with pain in her right leg, which was shortened and externally rotated. Radiographs of the patients hip are shown in Slides 3A and 3B. This fracture is best managed with:

 

1) Multiple screws

3) Internal fixation with sliding hip screw and 6-hole side-plate

2) Internal fixation with sliding hip screw and 4-hole side-plate

4) Arthroplasty

 

The sliding hip screw concept has made the treatment of unstable intertrochanteric fractures predictably successful, providing that the technique is performed properly. This includes center head placement of the lag screw within 5 mm of subchondral bone. Four screws are generally sufficient to anchor the side-plate in normal bone, but 6 screws are necessary in osteoporotic individuals.Correct Answer: Internal fixation with sliding hip screw and 6-hole side-plate

 

 

3226. (637) Q8-878:

 

 

 

Figure 3C Figure 3D

Postoperative radiographs are shown in Slides 3C and 3D (note that despite the recommendation above, the surgeon chose a 4-hole side-plate for this patient). This patient's risk of avascular necrosis is:

 

1) <5%

3) 50%

2) 20%

4) >80%

 

Avascular necrosis is rarely seen after intertrochanteric fracture.Correct Answer: <5%

 

 

 

 

 

Figure 3E Figure 3F

Follow-up radiographs taken 4 weeks later are shown in Slides 3E and 3F. Comparison of these films with the postoperative x-rays shows the fracture has collapsed since the fixation was performed. The surgeon's assessment and plan should be:

 

1) Impending failure, plan to restrict weight bearing and observe

3) Impending failure, plan to perform total hip replacement

2) Impending failure, plan to revise the internal fixation

4) Successful fixation, allow weight bearing as tolerated

 

The so-called sliding hip screw is designed to allow impaction of the fracture along the axis of the lag screw, allowing the fracture to settle into a stable position. This concept has revolutionized the treatment of intertrochanteric fractures by reducing the complications of cut-out and hardware breakage seen previously with fixed devices.Correct Answer: Successful fixation, allow weight bearing as tolerated

 

 

3228. (639) Q8-880:

 

 

 

Figure 4A Figure 4B

A 32-year-old, otherwise healthy male, was seen in the emergency room after a motorcycle accident. Examination showed obvious fractures of both femurs. The patient was not wearing a helmet, and had a mild closed head injury. Formal evaluation by the trauma service did not identify any other injury. There were no gross neurologic or vascular deficits. Anteroposterior x-rays of both femurs are shown in Slides 4A and 4B. The patient has bilateral subtrochanteric femur fractures. According to the Hennepin County Medical Center, Hennepin County classification, the right and left femur fractures would be classified as:

 

1) Both Type I

3) Right Type I, Left Type II

2) Both Type II

4) Right Type II, Left Type I

 

The classification system presently used at Hennepin County Medical Center is based on treatment and divides subtrochanteric fractures into two types (Slide 13). Type I, or high subtrochanteric fractures, have a fracture into the lesser trochanter. In Type II, or low subtrochanteric fractures, the lesser trochanter remains intact and a first generation interlocking nail is used. Both types of fracture may be either simple or comminuted with shaft extension. The right femur is a Type II, or low subtrochanteric fracture.

By definition, Type II fractures are below the lesser trochanter, and standard intramedullary nails may be used. The left leg has a Type I, or high subtrochanteric fracture.Correct Answer: Right Type II, Left Type I

 

 

 

 

 

Figure 1

Appropriate treatment of the acute injury depicted in Slide 1 is:

 

1) Open reduction, internal fixation

3) Closed reduction and casting

2) Traction

4) Primary midfoot arthrodesis

 

 

 

 

 

 

 

 

N/ACorrect Answer: Open reduction, internal fixation 3230. (641) Q8-884:

 

Figure 4A Figure 4B Figure 4C

Slide 4A shows the anteroposterior view of the left foot of a 52-year-old female who was involved in a high speed motor vehicle accident and suffered a closed head injury and a left foot injury. How would you treat this injury?

 

1) Cast treatment

3) Open reduction and internal fixation of the navicular fracture and anterior calcaneus impaction fracture

2) Open reduction and internal fixation of the navicular fracture only

 

Medially, open reduction and internal fixation was performed to reattach the displaced posterior tibial tendon avulsion fracture, as shown in Slide 4B. Laterally, the depressed articular surface of the calcaneus was elevated, and cancellous bone from the calcaneus was used to support the elevated fragment, thereby restoring the length of the lateral column and the congruity of the calcaneocuboid joint (Slide4C).Correct Answer: Open reduction and internal fixation of the navicular fracture and anterior calcaneus impaction fracture

 

 

3231. (642) Q8-885:

The ankle is a complex joint with the tibia and fibula forming a mortise containing the underlying talus. The joint is stabilized by three groups of ligaments including the medial collateral, lateral collateral, and syndesmotic ligaments. Thirteen tendons, two vascular bundles, and five nerves cross the ankle joint. Biomechanically, this anatomy functions to provide a mobile, stable base for ambulation. For normal ankle function during walking, the ankle must have at least:

 

1) 30° dorsiflexion, 30° plantarflexion

3) 20° dorsiflexion, 10° plantarflexion

2) 10° dorsiflexion, 20° plantarflexion

5) 30° dorsiflexion, 0° plantarflexion

4) 0° dorsiflexion, 30° plantarflexion

 

For normal ankle function during walking, the ankle must have at least 10% dorsiflexion and 20% plantarflexion. In normal ankle walking, the reported sagittal plane rotation has generally ranged from 10 degrees to 15 degrees of dorsiflexion and from 15 degrees to 30 degrees of plantarflexion.Correct Answer: 10° dorsiflexion, 20° plantarflexion

 

In performing a lateral approach to the calcaneus for open reduction, internal fixation, the structure at risk is:

 

1) Lateral plantar artery

3) Dorsalis pedism artery

2) Lateral plantar nerve

5) Superficial peroneal nerve

4) Sural nerve

 

The sural nerve is the most likely structure to be at risk when performing a lateral approach to the calcaneus to perform an open reduction and internal fixation. Care must be taken to protect the small saphenous vein and sural nerve lying immediately posterior to the incision.Correct Answer: Sural nerve

 

 

3233. (644) Q8-887:

 

 

 

Figure 2A Figure 2B

After reduction of the dislocated knee (Slide 2a and Slide 2b), the patient still has slightly decreased dorsalis pedis and posterior tibial pulses in her foot but has no other signs of ischemia. The next appropriate step in management of this injury is?

 

1) Observe vascular status of the limb and plan operative fixation of plateau fracture in the morning.

3) Obtain an arteriogram of limb.

2) Obtain a computed tomography scan to better evaluate the tibial plateau fracture.

4) Proceed to the operating room to explore the limb for a vascular injury.

 

In this patient, the only physical evidence of possible vascular injury is diminished pulses. Diminished pulses have been shown to be unreliable in predicting vascular injury. The next step should be to either obtain an immediate arteriogram or check ankle brachial pressures. If ankle brachial pressures are used, then the patient should have an ABI of 0.9 or greater. In addition, the patient should be closely monitored for 48 to 72 hours. If the patient has absent pulses, cold limb, or an open injury with obvious vascular injury, then the patient should be taken directly to the operating room for exploration and repair of the vessel. An arteriogram is not necessary in this situation since the location of the injury is known.Correct Answer: Obtain an arteriogram of limb.

 

 

 

3234. (645) Q8-888:

 

 

 

Figure 3A Figure 3B

A 30-year-old driver was involved in a motor vehicle accident. She has an isolated, closed injury shown in Slide 3a and Slide 3b. She is neurovascularly intact. What fracture type is this?

 

1) Schatzker I.

3) Schatzker III.

2) Schatzker II.

4) Schatzker IV.

 

This is a split depression injury of the lateral compartment (Type II).Correct Answer: Schatzker II.

 

3235. (646) Q8-889:

 

 

 

Figure 3A Figure 3B

A 30-year-old driver was involved in a motor vehicle accident. She has an isolated, closed injury shown in Slide 3a and Slide 3b. She is neurovascularly intact. What is the most appropriate management of this injury?

 

1) Closed treatment with a cast brace.

3) Open reduction internal fixation with bone grafting.

2) Open reduction internal fixation without bone grafting.

4) Closed reduction, percutaneous screw fixation.

 

The fracture is a displaced split depression (Schatzker II) fracture. The best treatment option is open reduction internal fixation using lag screws and a lateral buttress plate. The depressed fragment cannot be reduced by closed means using ligamentotaxis to facilitate the reduction. Bone grafting will hasten union.Correct Answer: Open reduction internal fixation with bone grafting.

 

 

3236. (647) Q8-890:

 

 

 

Figure 3A Figure 3B

A 30-year-old driver was involved in a motor vehicle accident. She has an isolated, closed injury shown in Slide 3a and Slide 3b. She is neurovascularly intact. What is the most appropriate postoperative plan?

 

1) Long leg cast for 4 weeks then aggressive physical therapy to improve range of motion and non-weightbearing for 3 months.

3) Range of motion beginning on postoperative day 2 and full weight bearing at 3 months.

2) Range of motion beginning in the recovery room and weight bearing as tolerated.

4) Long leg cast for 8 weeks and full weightbearing at 8 weeks.

 

After open reduction internal fixation, early range of motion is important. Range of motion should be started as soon as the soft tissues and the fracture fixation permit. Depending on the degree of comminution and the quality of the bone, partial weightbearing can be started after 6 weeks. Full weightbearing should be avoided for 3 months.Correct Answer: Range of motion beginning on postoperative day 2 and full weight bearing at 3 months.

 

Each of the following statements regarding the use of low-intensity ultrasound in fracture healing is correct except:

 

1) Ultrasound influences the inflammatory, reparative, and remodeling phases.

3) Ultrasound has not been shown to be beneficial in patients with comorbidities such as diabetes, vascular insufficiency, and obesity.

2) Animal studies demonstrate that ultrasound exposure results in stronger callus formation.

5) Ultrasound exposure during fracture healing accelerates the process of endochondral ossification.

4) Animal studies demonstrate that ultrasound exposure results in stiffer callus formation.

 

Several mechanisms have been proposed to explain the influence of low-intensity ultrasound on fracture healing. Ultrasound has been shown to stimulate angiogenesis, influencing regional blood flow to bone. In addition, chondrogenesis and chondroplasia have been shown to be positively affected by ultrasound, thus stimulating early endochondral ossification. Ultrasound is useful in patients with comorbid conditions such as smoking, obesity, and vascular insufficiency.

 

Correct Answer: Ultrasound has not been shown to be beneficial in patients with comorbidities such as diabetes, vascular insufficiency, and obesity.

 

 

3238. (1510) Q8-1901:

Which fascial compartment of the lower leg is most commonly affected by compartment syndrome:

 

1) Superficial posterior compartment

3) Lateral compartment

2) Deep posterior compartment

5) Deep anterior compartment

4) Anterior compartment

 

The anterior compartment of the lower leg is most commonly affected by compartment syndrome. The anterior compartment is enveloped by strong fascia and is susceptible to swelling caused by fracture and soft tissue injury.

 

 

The anterior and deep posterior compartments have a higher incidence of compartment syndrome than the superficial posterior compartment.

 

The deep posterior compartment is the most commonly missed compartment syndrome of the lower leg and is second in frequency, following anterior compartment syndromes.

 

The anterior and deep posterior compartments have a higher incidence of compartment syndrome than the lateral compartment.

 

The leg consists of four compartments including the anterior, lateral, superficial posterior, and deep posterior. A deep anterior compartment does not exist.

Correct Answer: Anterior compartment

 

 

3239. (1511) Q8-1902:

The diagnosis of compartment syndrome is most commonly missed in which compartment of the lower leg:

 

1) Superficial posterior compartment

3) Lateral compartment

2) Deep posterior compartment

5) Deep anterior compartment

  1. Anterior compartment

     

    Compartment syndrome is most commonly missed in the deep posterior compartment. Like the anterior compartment, the deep posterior compartment is enveloped in fascia and is immediately adjacent to the tibia; however, the deep posterior compartment is more difficult to clinically assess secondary to its deep location. Associated findings include pain with passive extension of the toes and plantar foot numbness.

     

     

    Compartment syndrome involving the superficial posterior compartment, lateral compartment, and anterior compartment is easy to detect because it is readily palpable on examination.

     

    There is no deep anterior compartment in the leg.

    Correct Answer: Deep posterior compartment

     

    How many compartments exist in the foot:

     

    1) 2

    3) 9

    2) 4

  2. 11

4) 10

 

There are nine separate compartments in the foot including the calcaneal, medial, superficial, lateral, adductor, and four separate interosseous compartments.Correct Answer: 9

 

 

3241. (1513) Q8-1904:

A 32-year-old man is involved in a motorcycle accident and sustains an open tibia fracture. The wound measures 0.5 cm in length. Radiographs demonstrate a transverse midshaft tibia fracture. He is noted to have dysvascular foot and requires repair of his posterior tibial artery. He is also treated with irrigation and debridement of his open fracture and application of an external fixator after vascular repair. Using the Gustilo-Anderson classification system of open fractures, what is the grade of this open fracture:

 

  1. Type I

3) Type IIIA

2) Type II

5) Type IIIC

4) Type IIIB

 

As defined in Gustiloâs and Andersonâs paper, a type IIIC jury includes any open fracture that is associated with an arterial injury that is repaired, regardless of the degree of soft tissue injury.

 

 

A type I open fracture consists of a wound less than 1 cm long with minimal soft tissue damage and no sign of crushing injury. The fracture is usually simple, transverse, or short oblique with little comminution.

 

A type II open fracture is a laceration more than 1 cm long with no extensive soft tissue damage. There is slight or moderate crush component with moderate contamination and comminution.

 

A type IIIA injury includes segmental or severely comminuted fractures from high-energy trauma, regardless of wound size. Soft tissue coverage is adequate despite extensive laceration.

 

A type IIIB injury is associated with extensive injury to or loss of soft tissue, with periosteal stripping and exposure of bone, massive contamination, and severe comminution from high-velocity trauma. After debridement and irrigation are complete, a segment of bone is exposed and a local or free flap is needed for coverage.

 

Correct Answer: Type IIIC

 

A 50-year-old man is involved in a fall from a ladder and sustains an open tibia fracture. The wound measures 0.5 cm in length. Radiographs demonstrate a transverse midshaft tibia fracture. He is noted to have an absent dorsalis pedis pulse. A vascular surgery consult is obtained, and the anterior tibial artery is lacerated and tied off in surgery. The foot remains well perfused.

Using the Gustilo-Anderson classification system of open fractures, what is the grade of this open fracture:

 

1) Type I

3) Type IIIA

2) Type II

5) Type IIIC

4) Type IIIB

 

As defined by the Gustilo-Anderson classification, a simple fracture pattern with a 0.5-cm wound is a type I injury. Because the vascular injury does not require repair, it is classified as a type I fracture.

 

 

A type II open fracture is a laceration more than 1 cm long with no extensive soft tissue damage. There is slight or moderate crush component with moderate contamination and comminution.

 

A type IIIA injury includes segmental or severely comminuted fractures from high-energy trauma, regardless of wound size. Soft tissue coverage is adequate despite extensive laceration.

 

A type IIIB injury is associated with extensive injury to or loss of soft tissue, with periosteal stripping and exposure of bone, massive contamination and severe comminution from high-velocity trauma. After debridement and irrigation are complete, a segment of bone is exposed and a local or free flap is needed for coverage.

 

As defined by Gustilo and Anderson, a type IIIC injury includes any open fracture that is associated with an arterial injury that must be repaired, regardless of the degree of soft tissue injury.

Correct Answer: Type I

 

 

3243. (1515) Q8-1906:

A 50-year-old man falls from a ladder and sustains an open tibia fracture. The wound measures 0.5 cm in length. Radiographs demonstrate a transverse midshaft tibia fracture. He is noted to have an absent dorsalis pedis pulse. A vascular surgery consult is obtained, and the anterior tibial artery is lacerated and tied off in surgery. The foot remains well perfused. What is the most appropriate course of treatment at this point:

 

1) Irrigation and debridement followed by external fixation

3) Irrigation and debridement followed by IM nailing using an unreamed technique

2) Irrigation and debridement followed by intramedullary (IM) nailing using a reamed technique

5) Irrigation and debridement followed by open plating

4) Irrigation and debridement followed by cast immobilization

 

Reamed IM nailing is the correct answer for a type I open tibia fracture. High union rates and low infection rates make this the correct choice.

 

 

External fixation can be used to treat a type I injury, however union rates are higher and final alignment improved with IM nailing.

 

A recent prospective study demonstrated that open tibial fractures randomized into reamed and unreamed groups had a similar time to union a larger nail and locking screws however demonstrates a lower failure rate over time.

 

Cast treatment provides inadequate stability and limit access to the soft tissue.

 

Open plating has fallen out of favor due to stripping of the soft tissue envelope resulting in delayed or nonunion and the associated risk of infection.

Correct Answer: Irrigation and debridement followed by intramedullary (IM) nailing using a reamed technique

 

A 32-year-old man is involved in a motorcycle accident and sustains an open tibia fracture. The wound measures 7 cm in length. Radiographs demonstrate a short oblique midshaft tibia fracture with a large butterfly fragment. Minimal damage to his anterior tibialis muscle is debrided. What is the most appropriate course of treatment at this point:

 

1) Irrigation and debridement followed by external fixation

3) Irrigation and debridement followed by IM nailing using an unreamed technique

2) Irrigation and debridement followed by intramedullary (IM) nailing using a reamed technique

5) Irrigation and debridement followed by open plating

4) Irrigation and debridement followed by cast immobilization

 

This injury must be treated as a Gustilo-Anderson type II injury. Reamed IM nailing is the correct choice.

 

 

External fixation of a type II injury may result in a longer time to union and an increase in final malalignment.

 

Multiple studies demonstrate a similar rate of infection and similar time to union between reamed and unreamed nails for type II and IIIA open fractures.

 

Cast treatment provides inadequate stability and limits access to the soft tissue.

 

Open plating has fallen out of favor due to stripping of the soft tissue envelope, which results in delayed or nonunion or infection.

Correct Answer: Irrigation and debridement followed by intramedullary (IM) nailing using a reamed technique

 

 

3245. (1517) Q8-1908:

A bull rider at the local rodeo has his leg stepped on when dismounting a bull in the rodeo arena. He sustains an open tibia fracture. The wound measures 1 cm in length. Radiographs demonstrate a transverse midshaft tibia fracture with an associated fibula fracture. Following irrigation, debridement, and intramedullary nailing, the most appropriate antibiotic regimen for prevention of infection is:

 

1) Cefazolin

3) Cefazolin, gentamicin, and penicillin

2) Cefazolin and gentamicin

5) Ciprofloxacin

4) Aminoglycoside and penicillin

 

By definition, this is a âbarnyardâ injury, which categorizes it as a Gustilo-Anderson type IIIA injury. A type IIIA injury is appropriately treated with a first-generation cephalosporin and an aminoglycoside. Penicillin is added because it is a barnyard injury.

 

 

 

Cefazolin is appropriate for type I and II open fractures. Cefazolin and gentamicin are used for type III open fractures.

 

An aminoglycoside and penicillin are not appropriate for any category of open fracture.

 

A recent study demonstrated that single-agent antibiotic therapy with ciprofloxacin is effective in treatment of type I and II open fracture wounds. However, ciprofloxacin alone is not recommended when compared to a first-generation cephalosporin and an aminoglycoside in treating type III open fractures.

 

Correct Answer: Cefazolin, gentamicin, and penicillin

 

A 32-year-old man is involved in a motorcycle accident and sustains an open tibia fracture. The wound measures 7 cm in length. Radiographs demonstrate a short oblique midshaft tibia fracture with a large butterfly fragment. He has minimal damage to his anterior tibialis muscle that is debrided. His injury is considered as a Gustilo-Anderson type IIIA based on degree of periosteal stripping. He is treated with irrigation debridement and an intramedullary nail using an unreamed technique. What is the most appropriate antibiotic regiment at this point:

 

1) Cefazolin

3) Cefazolin, gentamicin, and penicillin

2) Cefazolin and gentamicin

5) Ciprofloxin

4) Aminoglycoside and penicillin

 

A type IIIA injury is appropriately treated with a first-generation cephalosporin and an aminoglycoside.

 

 

 

Cefazolin is appropriate for type I and II open fractures. Cefazolin and gentamicin are used for type III open fractures.

 

An aminoglycoside and penicillin are not appropriate for any category of open fracture.

 

A recent study demonstrated that single-agent antibiotic therapy with ciprofloxacin is effective in treatment of type I and II open fracture wounds. However, ciprofloxacin alone is not recommended when compared to a first-generation cephalosporin and an aminoglycoside in treating type III open fractures.

 

Correct Answer: Cefazolin and gentamicin

 

 

3247. (1519) Q8-1910:

A bull rider at the local rodeo has his leg stepped on when dismounting a bull in the rodeo arena. He sustains an open tibia fracture. The wound measures 1 cm in length. Radiographs demonstrate a transverse midshaft tibia fracture with an associated fibula fracture. Utilizing the Gustilo-Anderson classification system of open fractures, what type of open fracture is this:

 

1) Type I

3) Type IIIA

2) Type II

5) Type IIIC

4) Type IIIB

 

An open wound of any size sustained in a barnyard, resulting from a gunshot wound, or a segmental fracture is considered a type IIIA open wound.

 

 

A type I open fracture is <1 cm and clean.

 

 

A type II open fracture is 1 cm to 10 cm and clean and does not have extensive soft tissue damage. A type IIIB open fracture is any size wound requiring a flap for coverage.

 

A type IIIC open fracture is any size wound requiring a vascular repair.

Correct Answer: Type IIIA

 

Tibia fractures in the proximal metaphysis treated with intramedullary nailing are most often complicated by which of the following deformities:

 

1) Varus

3) Procurvatum and varus

2) Valgus

5) Recurvatum

4) Procurvatum and valgus

 

The tendency for a medial and anterior nail entrance angle in a proximal third tibia fracture produces a procurvatum and valgus deformity. In addition, the pull of the quadriceps tendon, which is typically on the proximal fragment, tends to produce an apex anterior deformity.

 

 

 

Varus deformity is not generally a problem with proximal third tibia fractures. Valgus is a common deformity in combination with an apex anterior deformity.

 

 

Although procurvatum is a correct choice, procurvatum and varus is not the typical deformity. The pull of the quadriceps tendon produces procurvatum, not recurvatum.

Correct Answer: Procurvatum and valgus

 

 

3249. (1521) Q8-1912:

Closed tibia fractures with an intact fibula treated with casting or cast bracing are most often complicated by which of the following:

 

1) Nonunion

3) Valgus malunion

2) Varus malunion

5) Pulmonary embolism

4) Infection

 

An intact lateral strut tends to push the fracture into varus angulation.

 

 

 

Although nonunion is a complication of tibia fractures with an intact fibula, varus malunion is more common. The intact fibula pushes the fracture into varus, not valgus.

 

 

Infections are not a common complication of closed tibia fractures. Pulmonary emboli occur but not as frequently as varus malunion.

Correct Answer: Varus malunion

 

 

3250. (1522) Q8-1913:

What type of deformity following closed management of a tibia fracture is most poorly tolerated:

 

1) Valgus

3) Procurvatum

2) Varus

5) ranslation

4) Recurvatum

 

Because of the limited eversion of the ankle and hindfoot, varus deformity is poorly tolerated in healed tibia fractures.

 

 

 

The ankle and hindfoot can invert enough to compensate for some valgus deformity. Dorsiflexion of the ankle will correct minor procurvatum.

 

 

Plantarflexion of the ankle allows the patient with a recurvatum deformity to walk with a flat foot. As long as the fracture heals, translation is fairly well tolerated.

Correct Answer: Varus

 

Following what length of time is a tibia fracture considered a nonunion:

 

1) 3 months

3) 7 months

2) 5 months

5) 12 months

4) 9 months

 

A tibia fracture takes approximately 4 to 5 months to heal. A nonunion is considered to be about twice the normal healing time, although extraneous circumstances (e.g., diabetes and peripheral vascular disease) may extend this time frame.

 

 

In most healthy patients, tibia fractures take at least 4 months to heal and would not be considered a nonunion at 3 months.

 

 

Five months is the average amount of time for a tibia fracture to heal, not a nonunion. Seven months might be considered a delayed union but not a nonunion.

 

A nonhealed tibia fracture at 12 months is considered a nonunion, but in a healthy adult it would be considered a nonunion sooner.

Correct Answer: 9 months

 

 

3252. (1524) Q8-1915:

What is the average healing time for a tibia fracture:

 

1) 3 months

3) 7 months

2) 5 months

5) 12 months

4) 9 months

 

A tibia fracture takes approximately 4 to 5 months to heal, although certain extraneous circumstances (eg, diabetes and peripheral vascular disease) may extend this time frame.

 

 

Although some patients will heal in 3 months, this is not the average.

 

 

Although some healthy patients will take 7 months to heal, this is not the average. Nine months is considered a nonunion if a tibia fracture is not healed.

 

Twelve months is well beyond the normal healing time for a tibia fracture.

Correct Answer: 5 months

 

 

3253. (1525) Q8-1916:

What is the most appropriate soft tissue coverage option for a type IIIB open tibia fracture located in the mid-third of the tibia:

 

1) Gastrocnemius rotational flap

3) Split-thickness skin graft

2) Soleus rotational flap

5) Sural fasciocutaneous rotational flap

4) Rectus free flap

 

The most appropriate soft tissue coverage for an open wound of the mid-third of the tibia is a soleus rotational flap.

 

 

 

A gastrocnemius flap is not likely to cover the mid-third of the tibia. A split-thickness skin graft is not appropriate for exposed bone.

 

 

A rectus free flap will work but is considerably more morbid than a local rotational flap. A sural fasciocutaneous flap will not likely reach the mid-third of the tibia.

Correct Answer: Soleus rotational flap

 

What is the most appropriate soft tissue coverage option for a type IIIB open tibia fracture located in the distal third of the tibia:

 

1) Gastrocnemius rotational flap

3) Split-thickness skin graft

2) Soleus rotational flap

5) Sural fasciocutaneous rotational flap

4) Free flap

 

The most appropriate soft tissue coverage for an open wound of the distal third of the tibia is a free flap.

 

 

 

A gastrocnemius flap is not likely to cover the distal third of the tibia. A split-thickness skin graft is not appropriate for exposed bone.

 

A sural fasciocutaneous flap will not likely reach the distal third of the tibia.

Correct Answer: Free flap

 

 

3255. (1527) Q8-1918:

When reconstructing a tibial pilon fracture with internal fixation, which of the following is the first step in fixation:

 

1) The metaphyseal defect is augmented with bone graft.

3) The fibula is reduced and plated.

2) The articular surface is reconstructed and fixed.

5) A neutralization plate is applied to the tibia.

  1. A buttress plate is applied to the anterior or medial tibia.

     

    As described in the classic article by Ruëdi, the first step is fixation of the fibula to restore length, followed by reconstruction of the articular surface, then stabilization of the metaphyseal defect with bone graft, and finally, restoration of length in the tibia by fixing the metaphysis to the diaphysis with a buttress plate.Correct Answer: The fibula is reduced and plated.

     

     

    3256. (1528) Q8-1919:

    What percentage of tibial pilon fractures progresses to ankle fusion as a late salvage:

     

    1) 10%

    3) 50%

    2) 20%

  2. 90%

4) 75%

 

Most published series report an arthrodesis rate of 10% to 30%. Although there is an excellent rate of fixation and anatomic reduction, a significant number of patients progress to degenerative arthritis.Correct Answer: 20%

 

An individual sustains a both-bone forearm fracture, clavicle fracture, tibia fracture, femur fracture opposite the tibia fracture, and a talar neck fracture. All fractures were fixed appropriately. Which of these fractures will cause the most long-term disability:

 

  1. Both-bone forearm fracture

3) Tibia fracture

2) Clavicle fracture

5) Talar neck fracture

4) Femur fracture

 

Talar neck fractures are the most common fracture of the talus. Because no tendons or muscles attach to the talus, the blood supply is tenuous and there is a high incidence of avascular necrosis (AVN). The risk of AVN is based on Hawkinsâ classification with type I having a 10% risk and type IV having an AVN risk approaching 100%. Other complications include delayed union, nonunion, and malunion.

 

 

Although tibia, femur, and forearm fractures have a risk of nonunion or malunion, they generally heal with minimal longterm complications.

 

Most clavicle fractures heal uneventfully without long-term sequelae (nonunion rate of 0.8%).

Correct Answer: Talar neck fracture

 

 

3258. (1530) Q8-1921:

How much blood is most commonly lost in an isolated closed femoral shaft fracture:

 

1) 200 cc

3) 1,000 cc

2) 450 cc

5) 4,000 cc

4) 2,000 cc

 

Although the range of blood loss from an isolated closed femoral shaft fracture is 500 cc to 2,500 cc of blood, the most common amount of blood loss is 1,000 cc.Correct Answer: 1,000 cc

 

 

3259. (1531) Q8-1922:

A 45-year-old man sustained a femoral neck fracture that was treated with open reduction internal fixation with three cannulated screws. He developed a progressive shortening of the involved limb, and his radiographs are consistent with varus collapse and an obvious nonunion of his femoral neck fracture. The most appropriate next step in the management of this patient is:

 

1) Magnetic resonance image scan

3) Total hip arthroplasty

2) Valgus osteotomy (Pauwels)

5) Unipolar hemiarthroplasty

4) Bipolar hemiarthroplasty

 

The decision as to how to proceed after a femoral neck nonunion is based on consideration of a patientâs age, function, medical history, and bone density. Overall rates of osteonecrosis approach 10% for nondisplaced fractures and up to 40% in displaced fractures. Nonunion rates are higher in displaced femoral neck fractures and range from 10% to 30%. Nonunion is defined as no evidence of healing 12 months post-injury. A magnetic resonance image is the study of choice to look for avascular necrosis (AVN) of the femoral head. The presence or absence of AVN and the degree of head involvement dictates treatment.

 

 

Valgus osteotomy is a good option in a patient with a varus nonunion, even with AVN involving up to 50% of the femoral head. This is the best option in patients younger than 55 years old.

 

Total hip arthroplasty is a good option in a patient with preexisting rheumatoid arthritis or osteoarthritis involving the acetabular component of the hip joint. This would be the next option in this patient if he had AVN in more than 50% of his femoral head.

 

A bipolar hemiarthroplasty is not the best option for this patient. A bipolar hemiarthroplasty would be indicated in a sedentary patient with poor bone stock.

 

A unipolar hemiarthroplasty is indicated for a patient who is nonambulatory with low life expectancy and poor bone stock.

Correct Answer: Valgus osteotomy (Pauwels)

 

A 45-year-old man sustained a femoral neck fracture that was treated with three cannulated screws. He developed progressive shortening of the involved limb, and his radiographs are consistent with varus collapse and nonunion of his femoral neck fracture. A magentic resonance image was obtained by the referring physician and demonstrated avascular necrosis (AVN) of 40% of the femoral head. The most appropriate next step in the management of this patient is:

 

1) Observation

3) Total hip arthroplasty

2) Valgus osteotomy (Pauwels)

5) Unipolar hemiarthroplasty

4) Bipolar hemiarthroplasty

 

In this patient who has an established nonunion after being treated with open reduction internal fixation and subsequent AVN of a portion of the femoral head, valgus osteotomy is a good option even with AVN involving up to 50% of the femoral head. This is the best option in patients younger than 55 years old.

 

 

 

Observation is not a good option because it does not treat the underlying problem and would lead to permanent disability. Total hip arthoplasty is an excellent option in patients with extensive AVN or patients older than 55 years of age.

 

 

Bipolar hemiarthroplasty is indicated in an older patient as the primary form of treatment. Unipolar hemiarthroplasty is indicated in an elderly person who is nonambulatory.

Correct Answer: Valgus osteotomy (Pauwels)

 

 

3261. (4044) Q8-1924:

A 16-year-old girl arrives in your office 3 years after sustaining a comminuted right femur fracture with intramedullary (IM) nail fixation. She is complaining of moderate low back pain. On physical examination, you note that she has a 3-cm leg length discrepancy, and radiographs confirm that the right femur has healed 3 cm short. The most appropriate treatment at this point is:

 

1) Shoe lift on the right

3) Limb shortening of the left femur

2) Limb lengthening of the right femur

5) Left leg epiphysiodesis

4) Observation

 

Leg length discrepancy is a complication associated with pediatric femur fractures. Generally, discrepancies less than 2 cm can be observed and treated with shoe lifts. Discrepancies of 2 cm to 5 cm can be treated with shortening of the long side, usually using an IM saw and fixation with an IM nail.

 

 

A shoe lift on the right is not recommended for a discrepancy more than 2 cm.

 

Limb lengthening of the right femur is an option, but it would not be the first-line treatment in this situation. Limb lengthening would be indicated if the discrepancy were more than 5 cm.

 

In this scenario, a left leg epiphysiodesis would not be the best option because of the lack of remaining growth in this skeletally mature girl.

 

Observation is not a viable option because it is only indicated if the discrepancy is less than 2 cm.

Correct Answer: Limb shortening of the left femur

 

An 18-year-old woman arrives in your office 3 years after sustaining a comminuted right femur fracture treated with intramedullary (IM) nail fixation. She is complaining of moderate low back pain. On physical examination, you note that she has an 8-cm leg length discrepancy, and radiographs confirm that the right femur has healed 8 cm short. The most appropriate treatment at this point is:

 

1) Shoe lift on the right

3) Limb shortening of the left femur

2) Limb lengthening of the right femur

5) Left leg epiphysiodesis

4) Observation

 

The patient in question is at skeletal maturity. The approach to leg length discrepancies (LLD) depends on the length difference between the limbs. Discrepancies of more than 4 cm to 5 cm are treated with lengthening. Distraction is usually at the rate of 1 mm/day and is achieved by using Ilizarov principles, including metaphyseal corticotomy (preservation of the medullary canal and blood supply) and gradual lengthening. Lengthening over an IM nail may decrease the time in a distraction device.

 

 

A shoe lift is reserved for differences less than 2 cm.

 

The patient is at skeletal maturity so epiphysiodesis is not an option.

 

 

Limb shortening of the unaffected limb is reserved for limb discrepancies of 2 cm to 5 cm. Observation is unlikely to change the patientâs symptoms.

Correct Answer: Limb lengthening of the right femur

 

 

3263. (1534) Q8-1926:

An 18-year-old woman arrives in your office 3 years after sustaining a comminuted right femur fracture treated with intramedullary (IM) nail fixation. She is complaining of moderate low back pain. On physical examination, you note that she has a 1.5-cm leg length discrepancy, and radiographs confirm that the right femur has healed 1.5 cm short. The most appropriate treatment at this point is:

 

1) Shoe lift on the right

3) Limb shortening of the left femur

2) Limb lengthening of the right femur

5) Left leg epiphysiodesis

4) Observation

 

Generally, leg length discrepancies less than 2 cm in mature adults are ignored or treated with shoe lifts.

 

 

Limb lengthening is appropriate when the discrepancy is more than 5 cm.

 

 

Limb shortening of the unaffected limb is reserved for limb discrepancies of 2 cm to 5 cm. Observation would not be appropriate due to the patientâs pain.

 

The patient is at skeletal maturity so an epiphysiodesis is not an option.

Correct Answer: Shoe lift on the right

 

A 35-year-old woman presents 6 months after sustaining a fracture of the right ulna when struck by a softball. She was treated in a fracture brace for the past 6 months and continues to complain of pain at the fracture site. Radiographs demonstrate a hypertrophic nonunion with normal findings at the elbow and wrist and no significant ulna deformity. The surgical treatment plan at this point is:

 

1) Intramedullary (IM) nail fixation of the ulna

3) Bone stimulator and continued bracing

2) External fixation of the ulna

5) Compression plating without bone grafting

4) Compression plating with bone grafting

 

Hypertrophic nonunions are rich in callus and have a rich blood supply in the ends of the fragments. They result from unstable situations. Union is obtained by providing a stable environment for fracture healing with compression plating. Bone grafting is not required because hypertrophic nonunions have suitable biology and blood supply for healing.

 

 

 

The IM nail provides only relative stability and not absolute stability necessary for the healing of a hypertrophic nonunion. While compression is possible with an external fixator, it only provides a condition of relative stability and many inherent problems with this device preclude its normal use in the treatment of hypertrophic nonunions without deformity.

 

This patient has failed standard treatment for this fracture over a significant time frame. Continued use of a fracture brace will unlikely produce union. While a bone growth stimulator has potential to promote union, the literature is sparse and the outcome is uncertain when compared to compression plating.

 

Correct Answer: Compression plating without bone grafting

 

 

3265. (1536) Q8-1928:

A 35-year-old woman presents with a fracture of the right ulna when struck by a softball. Radiographs demonstrate a fracture of the ulna at the junction of the middle and distal third. Radiographs of the elbow and wrist are normal. The ulna is 25% translated and has 9% of apex radial angulation with no shortening. The treatment plan at this point is:

 

1) Intramedullary nail fixation of the ulna

3) Open reduction internal fixation (ORIF) with plate

2) External fixation of the ulna

5) Observation

4) Functional bracing

 

Ulna fractures sustained from a direct blow and not associated with radial head subluxation can be treated conservatively. Functional bracing or treatment with casting yields 95% to 98% union rates.

 

 

In cases with displacement of more than 50%, angulation of more than 10° to 11°, or significantly comminuted and shortened, ORIF with compression plating is most appropriate.

 

 

Other forms of fixation are considered less optimal. Observation does little to protect the fracture and control pain.

Correct Answer: Functional bracing

 

A 40-year-old woman falls on her outstretched left arm. She is point tender over her radial head and at her wrist. She is painful to range of motion at her elbow and cannot significantly move her elbow. Radiographs demonstrate a severely comminuted displaced radial head (Mason type III) fracture with significant step off in the joint. The best treatment plan for this patient is:

 

1) Sling and early range of motion

3) Radial head replacement

2) Cast for 6 weeks

5) Excision of radial head

4) Open reduction and internal fixation

 

Radial head replacement is recommended over radial head excision for highly comminuted radial head fractures with injury to medial collateral ligament or distal radioulnar joint.

 

 

Early mobilization should be considered for stable, minimally displaced fractures that involve less than one-third of the articular surface and have no limitation in motion.

 

Nondisplaced fractures without limitation in motion involving more than one-third of the articular surface should be treated with splint/sling for a minimum of 2 weeks followed by protected functional activities for an additional 7 to 10 days.

 

Comminuted fractures of the radial head are more commonly associated with higher energy trauma and, therefore, are rarely amenable to stable internal fixation.

 

Radial head resection is the most commonly performed procedure for comminuted fractures; however, it is associated with instability and late posttraumatic arthritis when performed in the setting of medial collateral ligament or interosseous membrane disruption.

 

Correct Answer: Radial head replacement

 

 

3267. (1538) Q8-1930:

When evaluating a patient for pyogenic flexor tenosynovitis, the diagnosis is often considered when Kanavel signs are present. Which of the following is not a Kanavel sign:

 

1) Fusiform swelling

3) Pain with passive range of motion

2) Purulence on aspiration

5) Digit in a flexed position

4) Pain along the flexor sheath Kanavelâs four signs include:

  1. Flexed position of finger

  2. Symmetric enlargement of whole finger

  3. Excessive tenderness over the course of the sheath but limited to the sheath

  4. Pain with passive extension finger, most marked at the proximal end

 

Aspiration of the tendon sheath is not considered a Kanavel sign.Correct Answer: Purulence on aspiration 3268. (1539) Q8-1931:

When draining pyogenic flexor tenosynovitis of the index finger, in addition to the incision in the palm, the other most appropriate location of the incision is:

 

1) Dorsal digit

3) Lateral ulnar aspect of digit

2) Volar digit

5) Wrist

4) Lateral radial aspect of digit

 

A midaxial incision is made in the finger, usually ulnar side to avoid the tactile portion of the index finger that is used to pinch.

 

 

 

A dorsal incision does not provide easy access to the flexor tendon. A radial side incision is used for the thumb and small finger.

 

An incision at the wrist does not allow for drainage of the flexor tendon sheath of finger.

Correct Answer: Lateral ulnar aspect of digit

 

3269. (1540) Q8-1932:

What bacteriae infection is likely to develop following a nail puncture wound to the sole of a foot through a tennis shoe:

 

1) Staphylococcus aureus

3) Pseudomonas

2) Eikenella corrodens

5) Clostridium perfringens

  1. Escherichia coli

     

    Pseudomonas is the most common cause of infection after a puncture wound through the sole of a shoe, accounting for up to 90% of all infections.

     

     

     

    Staphylococcus aureus is a common cause of orthopedic infections (e.g., cellulitis and osteomyelitis). Eikenella corrodens is associated with human bites.

     

     

    Escherichia coli is not common in orthopedic infections. It may be isolated in postoperative wounds. Clostridium perfringens is found in grossly contaminated traumatic wounds.

    Correct Answer: Pseudomonas

     

     

    3270. (1541) Q8-1933:

    What percentage of individuals who present with avascular necrosis (AVN) of the femoral head with an apparent idiopathic etiology will develop AVN in the contralateral hip:

     

    1) 10%

    3) 50%

    2) 25%

  2. 90%

4) 75%

 

Bilateral AVN is present in 34% to 72% of patients who present with idiopathic etiology.Correct Answer: 50%

 

 

3271. (1542) Q8-1934:

What imaging modality is the most sensitive for diagnosing asymptomatic avascular necrosis (AVN) of the femoral head:

 

  1. Plain radiographs

3) Ultrasound

2) Bone scan

5) Computerized tomography scan

4) Magnetic resonance image

 

Magnetic resonance imaging (MRI) is the most accurate imaging modality used for the diagnosis of AVN of the femoral head. MRI sensitivity is between 88% and 100%.

 

 

Plain radiographs are the first step after the history and physical examination for symptomatic disease. Stage 1 disease shows no changes on radiograph, but patients have clinical symptoms.

 

 

 

Bone scan may be used to evaluate âsilentâ osteonecrosis; however, MRI has a 10% to 20% higher sensitivity. Doppler ultrasound has been used to study blood flow to the femoral head, but this has not been well established. Computerized tomography scan is useful for detecting early stages of disease (II or III) without collapse.

 

Correct Answer: Magnetic resonance image

 

What laboratory test will be most sensitive for determining long-term response to antibiotic therapy in a child with a diagnosis of hematogenous osteomyelitis:

 

1) Magnetic resonance imaging (MRI)

3) Erythrocyte sedimentation rate (ESR)

2) Bone scan

5) White blood cell (WBC) count

4) C-reactive protein (CRP)

 

CRP elevation begins within 4 to 6 hours, peaks at 36 to 50 hours, closely parallels acute response with 4 to 7 hours half life, allowing return to normal 3 to 7 days after stimulus is withdrawn. Serial measurements of CRP are a logical option for routine use in conjunction with other clinical and laboratory data in evaluating children with acute hematogenous osteomyelitis.

 

 

MRI and bone scan are not laboratory tests.

 

 

ESR rises more slowly and may not return to normal for weeks despite clinical improvement. WBC counts are poor indicators of infection.

Correct Answer: C-reactive protein (CRP)

 

 

3273. (1544) Q8-1936:

A 65-year-old man with long-standing osteomyelitis of his right tibia, small vessel peripheral vascular disease, and diabetes presents with increasing drainage and pain from a sinus tract in his mid tibia. He has received antibiotics, but there is no improvement, and drainage and pain are becoming more severe. The best treatment option for this patient is:

 

1) Intravenous (IV) antibiotics and observation

3) Local debridement and IV antibiotics

2) Radical debridement, soft tissue coverage, and IV antibiotics

5) Below the knee amputation

4) Observation

 

Amputation is infrequently performed on patients with osteomyelitis. However, in certain patients it may be preferable to multiple operations that are unlikely to eradicate the infection. Arterial insufficiency, major nerve paralysis, or joint contractures and stiffness that make the limb nonfunctional are indications for amputation. Cases of osteomyelitis associated with malignancy in the sinus tract usually require amputation.Correct Answer: Below the knee amputation

 

 

3274. (1545) Q8-1937:

A 65-year-old woman presents with a complaint of new onset knee pain with no history of trauma. She has a mild effusion, no joint line tenderness, and no instability. During examination, she is afebrile. The most likely diagnosis is:

 

1) Degenerative meniscus tear

3) Avascular necrosis of the femoral condyle

2) Stress fracture

5) Gout

4) Septic arthritis

 

Degenerative tears of the meniscus tend to occur in patients older than 40 years of age with an atraumatic history of mild joint swelling, mild joint line pain, and mechanical symptoms.

 

 

 

Stress fractures are commonly found in individuals who place a high demand on their lower extremities. History usually reveals recent increase in impact loading. Clinically, the patient has pain, swelling, tenderness, and increased warmth. Avascular necrosis of the femoral condyle results in pain and loss of knee motion, and is usually associated with systemic disease.

 

Hallmarks of acute septic arthritis include pain with passive motion, swelling, and erythema. Patients may complain of fever and chills.

 

Gout results in an erythematous, painful joint. The attack is often sudden and involves a single joint. The joint is swollen, hot, and tender.

Correct Answer: Degenerative meniscus tear

 

What type of skin graft is least likely to contract over time:

 

1) Split thickess skin graft of 0.012 inch thickness meshed

3) Split thickess skin graft of 0.012 inch thickness not meshed

2) Split thickess skin graft of 0.018 inch thickness meshed

5) Full thickness skin graft

4) Split thickess skin graft of 0.018 inch thickness not meshed

 

Full thickness skin grafts do not contract a minimal amount when compared to partial thickness grafts.

 

 

The thinner the graft, the more wound contracture.

 

Meshing increases the surface area that can be covered and allows for drainage of hematomas.

Correct Answer: Full thickness skin graft

 

 

3276. (1547) Q8-1939:

A 40-year-old woman falls on her outstretched left arm. She is point tender over her radial head. She is not tender at her wrist. She is painful to range of motion at her elbow but demonstrates no limitation in motion. Radiographs demonstrate a radial head fracture with 1 mm step off involving approximately 30% of the joint surface. The treatment plan at this time is:

 

1) Sling and early range of motion

3) Cast for 3 weeks

2) Cast for 6 weeks

5) Excision of radial head

4) Open reduction internal fixation (ORIF)

 

Early mobilization should be considered for stable minimally displaced fractures that involve less than one-third of the articular surface and have no limitation in motion.

 

 

Nondisplaced fractures without limitation in motion involving more than one-third of the articular surface should be treated with splint/sling for a minimum of 2 weeks, followed by protected functional activities for an additional 7 to 10 days.

 

 

Displaced fractures without associated injury are preferably treated by ORIF. If ORIF is not feasible, then excision of the radial head may be attempted.

Correct Answer: Sling and early range of motion

 

 

3277. (1558) Q8-1950:

Union rates of the femur after antegrade or retrograde reamed intramedullary nailing are:

 

1) Higher for antegrade nailing

3) Identical

2) Higher for retrograde nailing

5) None of the above

4) Dependent on location in the bone

 

Several studies have compared antegrade with retrograde nailing. When controlled for canal fit and degree of reaming, the biological effects of antegrade and retrograde nailing on union are identical. The entry portal is irrelevant to union of the fracture.Correct Answer: Identical

 

Retrograde nailing when compared to antegrad nailing has:

 

1) A higher incidence of knee and hip pain.

3) There is no significant difference in joint function.

2) A lower incidence of knee and hip pain.

5) None of the above

4) A higher incidence of knee pain and a lower incidence of hip pain.

 

The major functional difference between antegrade and retrograde nailing is the incidence of hip vs knee pain. Because researchers often do not prospectively review the incidence of hip vs knee pain in patient outcomes, antegrade and retrograde nailing literature is inconsistent. Knee pain is 36% in retrograde nailing and 9% in antegrade femoral nailing. Hip pain is 4% in retrograde nailing and 10% in antegrade nailing.Correct Answer: A higher incidence of knee pain and a lower incidence of hip pain.

 

 

3279. (4045) Q8-1952:

When treating supracondylar fractures of the distal humerus in adults, the optimal fixation must include:

 

1) Lag screw fixation of the condyles

3) Perpendicular plating of medial and lateral condyles

2) Medial and lateral posterior plates

5) None of the above

4) Routine bone grafting

 

The distal humerus is flat and wide, and the ability of a plate to hold a wide bone is limited. 90/90 plates provide angular control in two planes, which significantly adds to the stability of the fracture construct. This is necessary to allow early motion and minimize the risk of implant fatigue.Correct Answer: Perpendicular plating of medial and lateral condyles

 

 

3280. (1560) Q8-1953:

When performing reamed antegrade femoral nailing of a transverse midshaft diaphyseal fracture, it is appropriate to:

 

1) Insert locking screws only if the fracture appears intraoperatively unstable after nail insertion.

3) Insert proximal and distal locking screws.

2) Avoid extra surgical exposure and insert a nail without locking screws.

5) None of the above

4) Only insert locking screws if the fracture requires them intraoperatively.

 

Brumback and colleagues showed that approximately 10% of fractures have unrecognized comminutions. Comminutions cause fractures to shorten and externally rotate in the immediate postoperative period, which may lead to malunion. Malunion is prevented by the insertion of locking screws.Correct Answer: Insert proximal and distal locking screws.

 

 

3281. (1561) Q8-1954:

The insertion of static locking screws with reamed antegrade femoral nailing is associated with:

 

1) Increased union time

3) No change in malunion rates with stable fractures

2) Delayed weight bearing

5) None of the above

4) No effect on union time or weight bearing.

 

Static locking of femoral shaft fractures does not retard bone union or delay time to full weight bearing. The advantages of static locking screws include prevention of malunion and early patient mobilization.Correct Answer: No effect on union time or weight bearing.

 

Reamed antegrade intramedullary nailing of the humerus is associated with:

 

1) Increased infection rates

3) Increased rates of nonunion

2) Increased rates of iatrogenic comminution

5) None of the above

4) A, B, and C

 

Reaming of the humerus thins the cortex and prevents revascularization, which results in thin bone that does not have ability to heal. Reamed antegrade nailing of the humerus must be avoided and only nonreamed antegrade intramedullary nailing should be attempted.Correct Answer: A, B, and C

 

 

3283. (1563) Q8-1956:

Antegrade femoral nailing in polytrauma patients with chest injuries should be:

 

1) Performed without reaming

3) Avoided acutely

2) Performed with reaming

5) None of the above

4) Delayed until the pulmonary injury has completed resolved.

 

Reamed femoral nailing in a patient with an acute pulmonary injury does not add to pulmonary complications or the incidence of acute respiratory distress syndrome (ARDS). Stabilize the fracture and mobilize the patient with any available technique. Studies from Germany show an approximate 35% incidence of ARDS in polytrauma patients who undergo antegrade reamed femoral nailing. Studies from the United States have not corroborated these results, and show a 5% to 8% incidence of ARDS.Correct Answer: Performed with reaming

 

 

3284. (1564) Q8-1957:

A 95° blade plate should be used with which of the following fractures:

 

1) A comminuted supracondylar femur fracture with intercondylar extension.

3) A supracondylar femur fracture without extension into the knee joint.

2) A comminuted supracondylar femur fracture without extension and 2 cm of intact bone in an elderly patient.

5) A supracondylar femur fracture with intracondylar comminution.

4) A contaminated supracondylar femur fracture without intracondylar extension.

 

Although blade plates are technically demanding, they are useful for fixation of fractures. To correctly use the device, there must be at least 2 cm of intact distal bone for adequate fixation, as well as correct blade placement in three planes. Blade plates are contraindicated in osteopenic bone and fractures with intracondylar comminution. Blade plates are the most stable of the fixation devices and provide excellent varus and valgus alignment.Correct Answer: A comminuted supracondylar femur fracture without extension and 2 cm of intact bone in an elderly patient.

 

 

3285. (1565) Q8-1958:

Which of the following is not required for use of the dynamic condylar screw (DCS) in a supracondylar femur fracture:

 

1) 4 cm of intact distal femur or easily reconstructable distal femur

3) Healthy, nonosteoporotic bone

2) Intact medial condyle

5) Fracture without intracondylar extension

4) Intact lateral soft tissue envelope

 

To obtain adequate fixation using a DCS, at least 4 cm of intact distal femur or easily reconstructable intracondylar femur is necessary as well as an intact medial condyle. If the device is used percutaneously without medial soft tissue stripping, then medial bone grafting is not necessary. Dynamic condylar screws are ideal fixation devices in osteoporotic bone with the use of cement augmentation. The disadvantage to this plate is that the large shoulder can sometimes be prominent on the lateral aspect of the femur.Correct Answer: 4 cm of intact distal femur or easily reconstructable distal femur

 

A 35-year-old man sustains a closed tibial pilon fracture. The patient undergoes open reduction internal fixation of the fracture with supplemental external fixation. Which of the following is the most common complication after open fixation:

 

1) Loss of fixation

3) Osteomyelitis

2) Prominent hardware

5) Nonunion

4) Soft tissue complications

 

Open fixation of severe tibial pilon fractures can lead to all of the above complications, but soft tissue complications are the most common. Soft tissue complications include wound dehiscence and infections, along with complications of the surrounding tendons.Correct Answer: Soft tissue complications

 

 

3287. (1567) Q8-1960:

Which of the following structures is not found in the greater sciatic foramen:

 

1) Superior gluteal nerve

3) Piriform muscle

2) Inferior gluteal nerve

5) Internal pudendal artery

4) Obturator nerve

 

Fourteen structures traverse through the greater sciatic foramen including seven nerves, three sets of vessels, and the piriform muscle. The seven nerves include the sciatic nerve, superior gluteal nerve, inferior gluteal nerve, internal pudendal nerve, posterior femoral cutaneous nerve, nerve to the quadratus femoris, and the nerve to the obturator externus. The three vessels include the superior gluteal artery and vein, inferior gluteal artery and vein, and the internal pudendal artery and vein.Correct Answer: Obturator nerve

 

 

3288. (1568) Q8-1961:

Which of the following devices is contraindicated for fixation of supracondylar femur fractures with intercondylar extension and comminution:

 

1) Blade plate

3) Condylar plate

2) Less invasive internal skeletal stabilization (LISS) plate

5) Immediate modular hinged total knee arthroplasty

4) Medial and lateral plating

 

Many devices can be used in fixation of supracondylar femur fractures with intercondylar extension. Each implant has advantages and disadvantages for use in the distal femur. Various devices include blade plates, compression screws, condylar plates, medial and lateral plating, antegrade or retrograde nailing, and the percutaneous plate. In patients with osteoporosis and preexisting arthrosis, the use of a modular replacement arthroplasty can address both the fracture and the arthrosis and provides the most rapid return of function. Intercondylar comminution can best be addressed with the use of dual plating, a condylar plate, and a LISS plate. A blade plate is technically demanding and can cause intercondylar comminution and does not provide stable fixation for fractures with intercondylar comminution.Correct Answer: Blade plate

 

 

Upon awakening, a 34-year-old alcoholic man experienced pain in his left shoulder. The patient holds his arm tightly against his chest and holds his forearm tightly to the trunk. External rotation and abduction are severely limited and painful. Which of the following injuries best accounts for these findings:

 

1) Luxatio erecta

3) Posterior dislocation of the shoulder

2) Anterior dislocation of the shoulder

5) Proximal humerus fracture

4) Superior dislocation of the shoulder

 

During electrocutions and seizures, there are uncontrolled muscle contractions that cause a posterior dislocation of the shoulder. The patient in this case has signs of a posterior shoulder dislocation and atraumatic injury most likely because of an alcoholic seizure. The dislocation occurs because the internal rotators of the humerus overpower the external rotators. Superior dislocations are rare and often involve associated fractures of the acromion. Luxatio erecta occurs when there is an inferior humeral head dislocation and the humerus is held alongside the head in full abduction with inability to adduct.Correct Answer: Posterior dislocation of the shoulder

 

 

3290. (1570) Q8-1963:

A 27-year-old man sustains a pelvic injury because of a crush. The patient is hemodynamically unstable despite administration of fluid boluses and packed red blood cells. A diagnostic peritoneal lavage is negative. The next step in the management of this patient is:

 

1) Computerized tomography scan of the abdomen and pelvis

3) Angiography

2) Pelvic external fixator

5) Pneumatic antishock garment

4) Exploratory laparotomy with open packing

 

Hemodynamic instability in a patient with blunt polytrauma must be considered a hemorrhage in the abdomen, pelvis, or chest. In the workup of the source of the bleeding, a diagnostic peritoneal lavage (DPL) should be performed to rule out intra-abdominal bleeding. If the DPL is negative and the patient is still hemodynamically unstable, an external fixator must be applied. The external fixator serves several purposes including preventing gross motion, stabilizing the dimensions of the pelvis to aid in tamponade, reducing of cancellous bleeding at the fracture site, and promoting early mobilization.Correct Answer: Pelvic external fixator

 

 

3291. (1571) Q8-1964:

An 11-year-old child sustains a radius and ulna diaphyseal fracture with 40° of volar angulation and complete displacement. Manipulation under hematoma block shows 75% apposition of the fractureâs end with 20° of volar angulation. The next step in management is:

 

1) Long arm cast in pronation

3) Open reduction and internal fixation

2) Long arm cast in supination

5) Closed reduction with percutaneous nailing

4) Remanipulation under intravenous sedation with splinting

 

After manipulation under hematoma block, this fracture remains in unacceptable alignment. In children older than 10 years of age, the amount of acceptable angulation is 15°, although anatomic alignment must be attempted. Indications for open reduction include open fractures, failed closed reductions, soft tissue interposition, and malalignment. If the child is between 6 and 10 years old, then 20° of angulation is acceptable.Correct Answer: Remanipulation under intravenous sedation with splinting

 

Which of the following is an indication for nonoperative fixation of a humeral shaft fracture:

 

1) Open fracture

3) Segmental fracture

2) Floating elbow

5) Polytrauma patient

4) Radial nerve palsy

 

A radial nerve palsy alone is not an indication for operative exploration or internal fixation of a humeral shaft fracture unless it occurs after attempted reduction. Indications for operative intervention include open fracture, polytrauma with multiple extremity injuries, a floating elbow, pathological fracture, and segmental injury.Correct Answer: Radial nerve palsy

 

 

3293. (1573) Q8-1967:

In a patient with a thoracolumbar burst fracture, which of the following factors does not necessitate operative treatment:

 

1) 60% canal compromise without neurological defect

3) Neurological defect

2) 20° of kyphosis

5) Rotational injury

4) Posterior spinal tenderness

 

It is important to differentiate between two-column injuries, which may be treated nonoperatively, and three-column injuries, which usually require operative treatment. It is the opinion of the authors that fractures are stable if they have less than 25° of kyphosis and less than 50% canal compromise with no neurological deficit. Also, posterior spinal tenderness along with a rotational or translational injury is indicative of a three-column injury.Correct Answer: 20° of kyphosis

 

 

3294. (1574) Q8-1968:

A 22-year-old man sustains a low velocity gunshot wound to the thigh resulting in a comminuted midshaft femur fracture. In addition to antibiotics, management must include:

 

1) Debridement of skin edges and plating

3) Wide debridement with delayed intramedullary nailing

2) Debridement of skin edges with immediate intramedullary nailing

5) Wide debridement with plating

4) Wide debridement with immediate intramedullary nailing

 

Most low velocity gunshot wounds do not inflict extensive soft tissue damage nor do they cause a blast-type injury. In the study by Nowotarski and Brumback, intramedullary nailing with underreamed or minial reaming performed immediately does not cause an increase in infection or malunion compared to plating with only superficial debriding of skin edges.Correct Answer: Debridement of skin edges with immediate intramedullary nailing

 

 

3295. (1575) Q8-1969:

A 40-year-old man sustains a posterior hip dislocation with a femoral head fracture below the fovea. After undergoing closed reduction with adequate anesthesia, the hip remains stable. Radiographs and computerized tomography (CT) scans are obtained and reveal a 1-cm displacement of the fracture fragment. The next step in management is:

 

1) Open reduction internal fixation (ORIF) via anterior approach

3) Skeletal traction

2) ORIF via posterior approach

5) Excision of fragment

  1. Mobilization along with protected weight bearing

     

    In Pipkin type 1 and 2 fractures of the femoral head (this patient has a type 1), nonoperative treatment is appropriate if the displacement of the fracture is less than 2 cm on CT scan after reduction. Studies have shown that ORIF or excision in type 1 fractures produces worse results than those treated with simple closed reduction.Correct Answer: Mobilization along with protected weight bearing

     

    After using external fixation, the infection rate reported in the literature is:

     

    1) 1%

    3) 9%

    2) 3%

  2. 35%

4) 20%

 

Pin tract infections are the most common complication of external fixation. Even with vigilance and immaculate maintenance of pin site care, the majority of patients will have at least one pin tract infection. Researchers at Ilizarov's institute summarized the complications associated with 3,669 fixator applications from 1970 through 1975. Analyzing wire-tract infections, the researchers found an 8.3% rate of purulent soft tissue sepsis and a 0.2% rate of wire-hole osteomyelitis.Correct Answer: 9%

 

 

3297. (1658) Q8-2053:

Which of the following external fixation constructs has the best long-term stability for the treatment of a mid-diaphyseal tibia fracture:

 

  1. A unilateral frame with 1 cm of bone loss

3) A delta frame with 1 cm of bone loss

2) A bilateral frame with 1 cm of bone loss

4) A unilateral frame with bone end abutment

 

Any external fixation configuration with bone end abutment provides better long-term stability because some portion of the physiological forces are transmitted, which decreases the forces transmitted through the pins and decreases the rate of loosening.Correct Answer: A unilateral frame with bone end abutment

 

 

3298. (1659) Q8-2054:

The bending strength of a pin is proportional to its radius to the:

 

1) First power

3) Third power

2) Second power

  1. Fourth power

     

    Pins are typically the weakest component of an external fixation system. The bending rigidity of each pin is theoretically proportional to the fourth power of the pin diameter, so increasing the pin diameter from 4 mm to 6 mm increases the bending stiffness of each pin by a factor of five.Correct Answer: Fourth power

     

     

    3299. (1660) Q8-2055:

    An external fixator pin must not exceed what percentage of the diameter of the diaphysis:

     

    1) 10%

    3) 30%

    2) 20%

  2. 50%

4) 40%

 

The diameter of any screw hole should be limited to no more than 30% of the diameter of the diaphysis. Studies show that a hole more than 30% of the diameter markedly increases the risk of fracture. A screw hole equal to 30% of the diameter weakens the torsional strength of a bone by 45%. Over 6 to 8 weeks, the bone will remodel about the implant and restore its strength. Upon removal of the screw, however, the weakening recurs until the bone has remodeled again.Correct Answer: 30%

 

The bending stiffness of an external fixation frame is decreased by:

 

  1. Changing to a hybrid frame

3) Increasing patient weight bearing

2) Axially dynamizing the frame

5) Adding another pin close to the fracture site

4) Increasing the frame-bone distance

 

Increasing the frame-bone distance decreases stiffness; adding another pin will increase the stiffness.Correct Answer: Increasing the frame-bone distance

 

 

3301. (1662) Q8-2057:

Which of the following is the most important factor in increasing frame rigidity:

 

1) Increasing the size of the side bar

3) Moving the pins closer to the fracture

2) Increasing the distance between pins

5) Increasing the size of the pins

4) Moving the pins farther from the fracture

 

Increasing the size of the pin is the most important means to increase frame rigidity.Correct Answer: Increasing the size of the pins

 

 

3302. (1663) Q8-2058:

The primary mode of healing with the use of modern external fixators is primary bone healing.

 

  1. True

  2. False

 

There are two types of fracture healing â union through external periosteal callus and primary bone healing, also known as in situ fracture remodeling. Fracture callus forms in reaction to the disruption of the periosteum and endosteum combined with the interfragmentary strain or motion associated with a bone injury. Callus bridges the fracture fragments and acts as a stabilizing structural framework and the biological substrate that provides the cellular material for union and remodeling. Current external fixation systems are designed to allow micromotion at the fracture site to promote callus formation.Correct Answer: False

 

 

3303. (1664) Q8-2059:

It is important to introduce bending preload when applying external fixators.

 

  1. True

  2. False

     

    At one time, researchers believed that bending preload was beneficial (placing the pins under bend and attaching them to the external fixator frame in this position); however, this causes failure of the pins because of rapid pressure necrosis on the compression side of the preloaded pin. Therefore, bending preload is discouraged.Correct Answer: False

     

     

    3304. (2008) Q8-2424:

    Which of the following constructs produces the strongest biomechanical fixation when treating a displaced talar neck fracture:

     

    1. Two K-wires placed from a posterior to anterior direction

  3. One 6.5-mm screw placed from an anterior to posterior direction

    1. Two 4.5-mm cancellous screws placed from a posterior to anterior direction

    5) Two 4.5-mm cancellous screws placed from an anterior to posterior direction

    1. Two K-wires placed from an anterior to posterior direction

       

      Biomechanical evaluations have shown that screws are superior to wires for fixation. Posterior-to-anterior screws insertion is superior to anterior-to-posterior insertion. It has also been demonstrated that two 4-mm screws are equivalent to a one 6.5-mm screw.Correct Answer: Two 4.5-mm cancellous screws placed from a posterior to anterior direction

       

       

       

      1) 5%

      3) 89%

      2) 13%

    2. 67%

  4. 38%

 

Despite surgeonsâ adherence to the principles of treating open fractures, the infection rate (38%) remains high, which correlates with poor functional outcome.Correct Answer: 38%

 

 

3306. (2010) Q8-2426:

Which of the following is the best predictor of intact vascularity to the talar body:

 

  1. The presence of subchondral sclerosis on anteroposterior view of the ankle at 6 to 8 weeks

3) Varus malunion of the talar neck

2) Subtalar arthritis

5) Subchondral collapse

4) The presence of subchondral lucency on anteroposterior view of the ankle at 6 to 8 weeks

 

The first sign of revascularization in subchondral bone is seen approximately 6 weeks after surgery. At this time, radiographs of the talus demonstrate a Hawkins sign â evidence of disuse osteoporosis under the subchondral bone of the talar dome. This osteolysis is an excellent sign of revascularization and weight bearing status should be progressed.Correct Answer: The presence of subchondral lucency on anteroposterior view of the ankle at 6 to 8 weeks

 

 

3307. (2011) Q8-2427:

The reported rate of osteonecrosis in a 21-year-old healthy male athlete who sustains a displaced fracture of the talar neck with displacement of both the subtalar and ankle joint is:

 

1) Less than 10%

3) Less than 5%

2) Less than 40%

5) 100%

4) Greater than 90%

 

The Hawkins classification is traditionally used to grade talar neck fractures by the amount of dislocation that has occurred between the body fragments and the neck, the ankle, or the subtalar joint. It is also used to predict the amount of avascular necrosis that may be expected. Depending on the severity of the injury, the risk of avascular necrosis in the body ranges from less than 5% to 90% or more.

 

In type I, the talar neck fracture is nondisplaced; the risk of avascular necrosis in the body is less than 10%. In type II, the body is slightly displaced from one of the two joints; the risk of avascular necrosis in the body is less than 40%. In type III, the body is displaced from both the ankle and the subtalar joints; the risk of avascular necrosis in the body is greater than 90%. Type IV, which was added by Canale and Kelly and not part of Hawkinsâ original classification, includes subluxation of the head, dislocation of the body from both sides, and extrusion of the body; the risk of avascular necrosis to the body approaches 100%.Correct Answer: Greater than 90%

 

 

 

3308. (2012) Q8-2428:

Radiographic indicators of an abnormal patellotibial relationship include all of the following except:

 

1) On an anteroposterior radiograph, the distal pole of the patella lies no more than 20 mm above the plane of femoral condyles.

3) On a lateral radiograph, the length of the patella ligament (from the distal pole to the tibial tubercle) approximates that of the patella.

2) On a 90° bent knee flexed lateral radiograph, the proximal pole of the patella lies posterior to the anterior surface of the femoral shaft.

5) Blumensaatâs line projects near the distal pole of the patella.

4) A patella to patella ligament ratio is equal to 0.5.

 

All of the answers above are radiographic indicators of an abnormal patellotibial relationship except that a patella to patella ligament ratio should equal 0.8.Correct Answer: A patella to patella ligament ratio is equal to 0.5.

 

liver laceration, proximal humerus fracture, and a distal both-bone forearm fracture. Early stabilization of the femoral shaft fracture within 24 hours decreases all of the following except:

 

1) Time until fracture unites

3) Risk for developing adult respiratory distress syndrome

2) Hospital cost

5) Length of stay in intensive care unit

4) Risk for developing pneumonia

 

In a prospective randomized study, Bone and coworkers demonstrated the beneficial effects of early fracture stabilization (within the first 24 hours) and early immobilization in the multiple-injured patients. They showed that the incidence of acute respiratory distress syndrome, pulmonary dysfunction, pneumonia, and abnormal blood gases was significantly lowered. The investigators also noted that early fixation and mobilization prevented deep venous thrombosis and decubitus ulcer. The literature does not show that the benefits of early fixation of femur fractures in multiple-injured patients decrease the time until the fracture unites.Correct Answer: Time until fracture unites

 

 

3310. (2014) Q8-2430:

A 48-year-old woman was involved in a motor vehicle accident as an unrestrained passenger in which she sustained a thoracic injury and a transverse midshaft femur fracture. Which of the following factors appears to be the most detrimental to the degree of lung dysfunction that will develop:

 

1) Placing an nonreamed intramedullary nail to stabilize the patientâs femur fracture

3) The severity of the original thoracic injury

2) Placing a reamed intramedullary nail to stabilize the patientâs femur fracture

5) Locking the intramedullary nail

4) Treating with open reduction and plating of the femur fracture

 

Bosse and colleagues have shown that fixation of long bone fractures has no effect on the severity of the thoracic injury. It appears that the severity of the original thoracic injury determines the degree of lung dysfunction that will develop.

 

Heim and colleagues found no difference between reamed or unreamed femoral nails in pathophysiologic pulmonary data in rabbits. They showed that the first instrument inserted (the starting awl, guide wire, and first one or two reamers) generates the highest pressures in the femoral canal. Heim also demonstrated that a solid nonreamed nail generated pressures similar to those of the first two reamers.Correct Answer: The severity of the original thoracic injury

 

 

3311. (2015) Q8-2431:

A 57-year-old woman presents with a 4-week history of right hip pain. She has no history of trauma. Her past medical history is significant for osteoporosis. Radiographs that were obtained demonstrate a haze of internal callus at the inferior neck. Treatment should include:

 

1) Open reduction internal fixation using a compression screw

3) Closed reduction and placement of three percutaneous cannulated screws

2) Closed reduction and placement of a percutaneous cannulated screw

5) Crutch ambulation with toe-touch weight bearing status

4) Hemiarthroplasty

 

Horiuchi and colleagues demonstrated that compression fractures essentially have no risk of displacement without additional trauma and should be treated with crutch ambulation.Correct Answer: Crutch ambulation with toe-touch weight bearing status

 

 

3312. (2016) Q8-2432:

The critical blood supply to the majority of the weight bearing superior portions of the femoral head is supplied by the:

 

1) Terminal branch of the lateral femoral circumflex artery

3) Lateral epiphyseal artery

2) Artery of the ligamentum teres

5) Foveolar artery

4) First branch of the medial femoral circumflex artery

 

The critical blood supply to the majority of the weight bearing superior portions of the femoral head is supplied by the terminal branch of the medial femoral circumflex artery, or lateral epiphyseal artery.Correct Answer: Lateral epiphyseal artery

 

3313. (2017) Q8-2433:

A 77-year-old man who lives in a nursing home fell from his wheel chair and landed on his right hip earlier this morning. He presents to the emergency department with a painful right hip to palpation held in external rotation. The patient reports pain with any attempt to move his right hip. His initial radiograph does not demonstrate any fracture or dislocation. Your next step would be to:

 

1) Obtain a technetium 99 bone scan before the patient leaves the emergency department on the same day

3) Obtain a technetium 99 bone scan within 48 hours of the accident

2) Send the patient back to the nursing home and arrange for follow-up in your office within the next week

5) Obtain a computed tomography image of the right hip while the patient is in the emergency department

4) Obtain a magnetic resonance image of the right hip while the patient is in the emergency department

 

Rizzo and associates showed that an MRI is superior to bone scanning in the detection of occult hip fractures in the first 48 hours.Correct Answer: Obtain a magnetic resonance image of the right hip while the patient is in the emergency department

 

 

3314. (2018) Q8-2434:

Advantages of the use of intramedullary second-generation nails over plates and screws in treatment of unstable subtrochanteric fractures include all the following except:

 

1) Intramedullary nails preserve the periosteal blood supply.

3) Intramedullary nails are twice as strong in terms of biomechanical stiffness and strain.

2) Intramedullary nails are load-sharing devices.

5) Intramedullary nails cause fewer varus malunions.

4) Intramedullary nails cause less femoral head cutout.

 

Kraemer and associates showed that intramedullary devices were superior to plates and screws in that intramedullary devices are load-sharing and biomechanically stronger, and preserve blood supply.

 

Alvarez and colleagues demonstrated that varus deformities occurred more frequently with the use of an intramedullary nail due to entry points placed too far lateral to the tip of the trochanter.Correct Answer: Intramedullary nails cause fewer varus malunions.

 

 

3315. (2019) Q8-2435:

Surgical indications for treatment of a tibial plateau fracture include all the following except:

 

1) Medial tilt

3) Lateral condyle fracture in 3º of valgus

2) Articular incongruity of 5 mm

5) Axial displacement

4) 8 mm of condylar widening

 

An articular step-off of 3 mm or condylar widening of 5 mm has no adverse effects. Functionality deteriorates with medial tilt, whereas lateral or valgus tilt of up to 5º is well tolerated. Bicondylar fractures with axial displacement also should be treated surgically.Correct Answer: Lateral condyle fracture in 3º of valgus

 

 

3316. (2020) Q8-2436:

Benefits of antegrade femoral intramedullary nailing over retrograde femoral intramedullary nailing in treatment of closed midshaft femur fracture include:

 

1) Less blood loss

3) Decreased time to union

2) Lower operative time

5) Less hip pain

4) Distal locking screw pain necessitating removal

 

Ostrum and associates in the largest prospective randomized series of antegrade versus retrograde femoral nailing to date showed that there was no difference in union rates, time to union, knee pain, or range of motion. Blood loss was significantly lower in the retrograde group, and there was a trend toward lower operative time. Hip pain was more common in the antegrade nails, and distal locking screw pain necessitating removal seems to be more common in retrograde nails.Correct Answer: Distal locking screw pain necessitating removal

 

3317. (2021) Q8-2437:

Diabetic patients who sustain rotational ankle fractures are highly likely to have higher complications (infection, long-term bracing, and/or malunion/nonunion, BKA) independent of the treatment approach if at the time of injury the patient had documented at least one of the following except:

 

1) Nephropathy

3) Age older than 55 years

2) Neuropathy

5) Vasculopathy

4) Retinopathy

 

Yelden, Marsh, and Saltzman demonstrated that diabetic patients who sustain rotational ankle fractures have higher risks for developing complications. Diabetic patients were more likely to require or develop infection, long-term bracing and/or malunion/nonunion, BKA, independent of age, gender, fracture-type, and treatment approach. Diabetic patients are significantly more likely to have one of these outcomes if at the time of presentation they had documented nephropathy, neuropathy, retinopathy, or vasculopathy.Correct Answer: Age older than 55 years

 

 

3318. (2022) Q8-2438:

A 37-year-old woman who is an avid jogger sustains a displaced fracture of the talar neck with displacement of both the subtalar and ankle joints. She was treated with immediate open reduction internal fixation through two incisions. At 13 weeks postinjury, the patientâs fracture radiographically appears to be united and well aligned. Radiographs show evidence of disuse osteoporosis under the superior subchondral bone of the talar dome. Treatment at this time should include:

 

1) Protected weight bearing

3) A magnetic resonance image to evaluate the boneâs viability

2) Autograft bone grafting of the talar body

5) Application of a bone stimulator

4) Blair fusion

 

The first sign of revascularization in subchondral bone is seen approximately 6 weeks after surgery. At this time, radiographs of the talus demonstrate a Hawkins sign â evidence of disuse osteoporosis under the subchondral bone of the talar dome. This osteolysis is an excellent sign of revascularization and weight bearing status should be progressed.Correct Answer: Protected weight bearing

 

 

3319. (2023) Q8-2439:

The rigidity of an external fixator can be increased by:

 

1) Decreasing the distance between the bone and rods

3) Decreasing pin diameter

2) Decreasing the number of pins

5) Increasing the distance between fragment pin sets

4) Separating the half pins by less than 30º when applying anterior and anteromedial frames

 

Many factors determine the rigidity of a frame. The pin holes should be less than 30% of the bone diameter to decrease the risk of open section fracture. The bending stiffness of the pin is proportional to half the radius to the fourth power. Therefore, increasing the pin diameter to less than 30% of the bone diameter will increase the rigidity of the external fixator. Tubular rods with an 11-mm diameter are approximately twice as stiff as solid connecting rods with an 8-mm diameter. Increasing the pin spread within each bone segment to 9 cm triples the resistance to anteroposterior bending. Decreasing the bone rod distance to 2.5 cm leads to a threefold increase in resistance to transverse bending. Another way to increase stiffness of the frame is to separate half pins by greater than 45° when applying anterior and anteromedial frames together.Correct Answer: Decreasing the distance between the bone and rods

 

 

 

3320. (2024) Q8-2440:

Clinical variables associated with a poor outcome following calcaneal fractures include all of the following except:

 

1) Age older than 50 years

3) History of heavy labor

2) Increased body weight

5) Decreased body weight

4) On workersâ compensation

 

Paley and associates demonstrated the clinical variables associated with a poor outcome include age older than 50 years, increased body weight, history of heavy labor, and on workersâ compensation.Correct Answer: Decreased body weight

 

3321. (2025) Q8-2441:

Radiographic factors correlating with a poor outcome following calcaneal fractures include all of the following except:

 

1) Subtalar incongruity

3) Increased heel width

2) Arthrosis of the tibiotalar and talonavicular joints

5) Increased Bohler angle

4) Decreased fibulocalcaneal space

 

The radiographic factors correlating with a poor outcome include subtalar incongruity, arthrosis of the tibiotalar and talonavicular joints, increased heel width, decreased fibulocalcaneal space, and a decreased Bohler angle. Loucks and Buckley reported the correlation between Bohler angle and clinical outcome. An increase in initial displacement, regardless of treatment, results in a less successful outcome.Correct Answer: Increased Bohler angle

 

 

3322. (2026) Q8-2442:

Complications of nonsurgical treatment of calcaneal fractures include all of the following except:

 

1) Peroneal and saphenous nerve impingement

3) Short and wide heel

2) Hindfoot varus

5) Relative leg length discrepancy

4) Pes planus

 

Complications of nonsurgical treatment of calcaneal fractures include peroneal and sural (not saphneous) nerve impingement, hindfoot varus, short and wide heel, pes planus, relative leg length discrepancy, and tibiotalar impingement limiting extension.Correct Answer: Peroneal and saphenous nerve impingement

 

 

3323. (2027) Q8-2443:

Which of the following patient-related risk factors is associated with an increased risk for infection after open reduction and internal fixation of calcaneal fractures through standard lateral incision:

 

1) Obesity

3) Use of systemic steroids

2) History of alcohol abuse

5) Extension of the fracture into the calcaneocuboid joint

4) History of smoking

 

Folk, Starr, and Early showed a 90% risk of wound problems when a patient who sustained a calcaneal fracture had diabetes, a history of smoking, and an open fracture.Correct Answer: History of smoking

 

 

3324. (2028) Q8-2444:

The location of the lisfranc ligament is between:

 

1) Middle cuneiform and base of the second metatarsal

3) Medial cuneiform and middle cuneiform

2) Base of second metatarsal and the medial cuneiform

5) Medial cuneiform and base of first metatarsal

4) Base of first metatarsal and middle cuneiform

 

The lisfranc ligament traverses the base of the second metatarsal to the medial cuneiform.Correct Answer: Base of second metatarsal and the medial cuneiform

 

A 22-year-old college basketball player presented with a history of pain along the lateral border of the foot for the past 3 weeks. Radiographs demonstrated intramedullary sclerosis and fracture at the metaphyseal-diaphyseal junction of the fifth metatarsal. Treatment at this point should include:

 

1) Below-the-knee cast and non-weight bearing

3) Below-the-knee cast and weight bearing as tolerated

2) Soft dressing and weight bearing as tolerated

5) Stiff-soled postoperative shoe with partial weight bearing

4) 4.5-mm intramedullary screw placement

 

DeLee and associates reported on the early return of 10 athletes with diaphyseal stress fractures treated with closed intramedullary screw fixation. This is adequate treatment for foot stress fractures.Correct Answer: 4.5-mm intramedullary screw placement

 

 

3326. (2030) Q8-2446:

Which of the following structures causes an avulsion fracture of the tuberosity of the fifth metatarsal:

 

1) Peroneus tertius

3) Lateral band of plantar fascia

2) Peroneus brevis

5) Medial band of plantar fascia

4) Peroneus longus

 

In a cadaveric study, Richli and Rosenthal implicated the lateral band of plantar aponeurosis as the structure more likely to cause tuberosity avulsion fractures.Correct Answer: Lateral band of plantar fascia

 

 

3327. (2031) Q8-2447:

Thirteen weeks ago, a 42-year-old man sustained a displaced fracture of the talar neck with displacement of both the subtalar and ankle joint. Initially, he underwent open reduction and internal fixation with titanium screws and has been non-weight bearing since the injury occurred. Radiographs of the talus reveal complete union. A magnetic resonance image (MRI) reveals signal changes in 40% of the talus body. Treatment at this point should include:

 

1) Progress to weight bearing as tolerated

3) Blair arthrodesis

2) Progress to partial weight bearing

5) Talectomy

4) Continue limited weight bearing and repeat the MRI in the next 3 to 6 months

 

Thordarson and associates used MRI to established criteria for allowing a patient to begin weight bearing. Their study suggested that patients with Hawkins types I and II fractures should be allowed protective weight bearing when radiographic evidence of healing at the fracture is present. None of the patients with a type I or II fracture in this study developed late segmental collapse.

 

For patients with a Hawkins type III or IV fracture, the investigators recommended an MRI at 8 to 12 weeks postinjury to assess for osteonecrosis. The degree of osteonecrosis is then classified based on the percent of talar body affected by osteonecrosis (type A, homogenous bone throughout the body of the talus; type B, signal changes in up to 25 % of the body of the talus; type C, signal changes in up to 25% to 50% of the body of the talus; and type D, signal changes in more than 50 % of the body of the talus). The fracture presented in this question is a type C. Type C fractures are kept non-weight bearing and the MRI is repeated in 6 to 9 months after injury. If no progression of signal changes is noted, protective weight bearing is allowed.Correct Answer: Continue limited weight bearing and repeat the MRI in the next 3 to 6 months

 

 

A 48-year-old man who was involved in a motor vehicle accident as a restrained passenger sustained an isolated split-depression fracture of his left lateral tibial plateau 8 days ago. Clinical signs indicating that the soft tissue will be more tolerant of an open reduction internal fixation procedure include:

 

1) The presence of fracture blisters

3) The ability to palpate bony landmarks

2) The presence of extensive subcutaneous hemorrhage or bruising

5) The presence of a large draining open wound over the knee

4) The absence of skin wrinkles

 

The absence of fracture blisters, the ability to palpate bony landmarks, the presence of skin wrinkles, and the absence of extensive subcutaneous hemorrhage or bruising are clinical signs that help determine the safety of open reduction internal fixation of tibial plateau fractures.Correct Answer: The ability to palpate bony landmarks

 

 

3329. (2033) Q8-2449:

To avoid intra-articular placement of small wires from a hybrid external fixator, tensioned wires or half pins should be kept how far from the articular surface in the knee joint.

 

1) 3 mm from the articular surface

3) 7 mm from the articular surface

2) 10 mm from the articular surface

5) 14 mm from the articular surface

4) 12 mm from the articular surface

 

To avoid intra-articular placement of small wires from a hybrid external fixator, tensioned wires or half pins should be kept more than 14 mm from the articular surface. The proximal tibio-fibular joint sometimes communicates with the knee joint.Correct Answer: 14 mm from the articular surface

 

 

3330. (2034) Q8-2450:

The advantages of treating tibial shaft fractures with intramedullary nailing over closed treatment and casting include all of the following except:

 

1) Time to healing

3) Decreased rate of malunion

2) Rate of healing

5) Increased measures of general health status

4) Decreased functional scores for ankle and knee function

 

Bone and associates reported that time to healing, rate of healing, decreased rate of malunion, increased functional scores for ankle and knee function, and increased measures of general health status are favored with intramedullary nailing of diaphyseal tibia fractures over closed treatment via casting.Correct Answer: Decreased functional scores for ankle and knee function

 

 

3331. (2035) Q8-2451:

Which of the following is the most common complication associated with the treatment of a closed diaphyseal tibia fracture with an intramedullary nail:

 

1) Nonunion

3) Infection

2) Compartment syndrome

5) Knee pain

4) Symptomatic hardware

 

Court-Brown and associates identified anterior knee pain as the most common complication associated with treatment of a closed diaphyseal tibia fracture with an intramedullary nail.Correct Answer: Knee pain

 

A 39-year-old man sustained a grade II open diaphyseal tibia fracture that was treated with irrigation and debridement, external fixation, and delayed wound closure 12 weeks ago. At 9 weeks, the patientâs weight bearing status was increased to partial weight bearing and the patient has since reported slight leg pain. Radiographs indicated a comminuted diaphyseal fracture of the tibia with no signs of callus formation. The radiographs also show no signs of loosening of the external fixator pins. Treatment at this point should include:

 

1) Removal of external fixator and placement of a patellar tendon bearing (PTB) cast, and progression to weight bearing as tolerated

3) Removal of external fixation and intramedullary nailing

2) Irrigation and debridement

5) Placement of coralline hydroxyapatite into the fracture site

4) Autogenous bone grafting

 

In this patient, the soft tissue has healed from his open wound. The patient is progressing toward a delayed nonunion due to lack of callus formation. The next step should include autogenous bone grafting. The progression of his weight bearing status will lead to loosening of his external fixator pins.Correct Answer: Autogenous bone grafting

 

 

3333. (2037) Q8-2453:

In the fixation of a diaphyseal closed tibia fracture with an intramedullary nail, peaks in intracompartmental pressures occur with:

 

1) Nail passage

3) Insertion of the awl at the starting point

2) Placement of distal locking screws

5) Placement of the proximal locking screw

4) Placement of the guide wire

 

Tornetta and associates showed that peaks in intracompartmental pressures occur in the fixation of a diaphyseal closed tibia fracture with an intramedullary nail during fracture reduction and nail passage. However, generally, these peaks are not sustained and return to baseline during unreamed nailing.Correct Answer: Nail passage

 

 

3334. (2073) Q8-2494:

The primary treatment for anterior sternoclavicular dislocation is:

 

1) Open reduction internal fixation

3) Wiring to an adjacent rib

2) Closed reduction

5) Excision

4) Sling

 

Anterior sternoclavicular dislocations are benign injuries that do not affect any significant neurovascular structure. Attempts to reduce anterior sternoclavicular dislocations often result in high rates of redislocation; therefore, these dislocations are best left unreduced. Internal fixation has not been successful. Resection is recommended for chronic pain.Correct Answer: Sling

 

 

3335. (2074) Q8-2495:

The primary treatment for acute posterior dislocation of the sternoclavicular joint is:

 

1) Closed reduction

3) Excision

2) Wiring to an adjacent rib

5) Sling

  1. Open reduction internal fixation

     

    Posterior dislocations of the sternoclavicular joint, unlike anterior dislocations, can be treated by a closed reduction. The tension forces of the clavicle will allow the sternoclavicular joint to generally remain anterior. The closed reduction is sometimes performed with a towel clip.Correct Answer: Closed reduction

     

    The medial physis of the clavicle closes by age:

     

    1) 18

    3) 22

    2) 20

  2. 30

4) 25

 

The medial physis of the clavicle is the last to appear and the last to close. This is an important point because many injuries to the sternoclavicular area are actually physeal injuries, allowing bone-to-bone healing and, potentially, some remodeling.Correct Answer: 25

 

 

3337. (4049) Q8-2497:

Which of the following is not a design consideration in the titanium limited contact dynamic compression plate (LCDCP):

 

  1. Uniform strength throughout the plate

3) The plates are slightly longer

2) Less stress shielding

5) Elongated holes

4) Increased vascularity to the cortex

 

Scallops under the plate allow increased cortical blood flow and elongated holes allow more variation in screw placement in titanium LCDCP design. Stress shielding is not a significant property of plates as was once thought.Correct Answer: Less stress shielding

 

 

3338. (2076) Q8-2498:

The innervation of the brachialis muscle is:

 

1) Radial

3) Split between median and radial

2) Axillary

5) Split between radial and musculocutaneous

4) Musculocutaneous

 

The innervation of the brachialis is divide The lateral half of the brachialis is radially innervated and the medial half of the brachialis is innervated by the musculocutaneous nerve. This is an important point in approaches to the humerus because the muscles can be split to apply a plate.Correct Answer: Split between radial and musculocutaneous

 

 

3339. (2077) Q8-2499:

Which of the following characteristics does not place the radial nerve at particular risk with distal humeral fractures:

 

1) Butterfly comminution

3) Transverse fracture

2) Lateral spike

5) Open

4) Varus

 

The classic Holstein-Lewis fracture is a distal-third fracture with a lateral spike displaced into varus. This allows the radial nerve to slip into the fracture site creating a palsy that can be worsened by closed reduction.Correct Answer: Butterfly comminution